Jump to content

Wikipedia:Reference desk/Science: Difference between revisions

From Wikipedia, the free encyclopedia
Content deleted Content added
→‎biology: new section
Line 761: Line 761:


:: Another way to go is start at [[Brain]] and have a look at [[Electroencephalography]] [[Functional magnetic resonance imaging]] [[Neuron]]. Don't have the time now to dig up pages, but there have been quite a number of studies correlating increased brain activity in certain areas to corresponding induced thoughts. Having a scout around in the science magazine archives should get you some more information. [[User:Lisa4edit|Lisa4edit]] ([[User talk:Lisa4edit|talk]]) 17:17, 16 May 2008 (UTC)
:: Another way to go is start at [[Brain]] and have a look at [[Electroencephalography]] [[Functional magnetic resonance imaging]] [[Neuron]]. Don't have the time now to dig up pages, but there have been quite a number of studies correlating increased brain activity in certain areas to corresponding induced thoughts. Having a scout around in the science magazine archives should get you some more information. [[User:Lisa4edit|Lisa4edit]] ([[User talk:Lisa4edit|talk]]) 17:17, 16 May 2008 (UTC)

== biology ==

what are prions??? what are the possible mechanisms through which they cause diseases??and what are the current techniques being employed to understand them and cure them???

Revision as of 18:18, 16 May 2008

Welcome to the science section
of the Wikipedia reference desk.
Select a section:
Want a faster answer?

Main page: Help searching Wikipedia

   

How can I get my question answered?

  • Select the section of the desk that best fits the general topic of your question (see the navigation column to the right).
  • Post your question to only one section, providing a short header that gives the topic of your question.
  • Type '~~~~' (that is, four tilde characters) at the end – this signs and dates your contribution so we know who wrote what and when.
  • Don't post personal contact information – it will be removed. Any answers will be provided here.
  • Please be as specific as possible, and include all relevant context – the usefulness of answers may depend on the context.
  • Note:
    • We don't answer (and may remove) questions that require medical diagnosis or legal advice.
    • We don't answer requests for opinions, predictions or debate.
    • We don't do your homework for you, though we'll help you past the stuck point.
    • We don't conduct original research or provide a free source of ideas, but we'll help you find information you need.



How do I answer a question?

Main page: Wikipedia:Reference desk/Guidelines

  • The best answers address the question directly, and back up facts with wikilinks and links to sources. Do not edit others' comments and do not give any medical or legal advice.
See also:


May 10

Could magnetism be said to warp space-time in a similar way to gravity?

And spare me “jokes” about luminiferous ether. It’s just that there are obvious similarities between the attractive effects of those two forces on distant objects. (Although there seems to be no repulsive action with gravity). So, can magnetism curve space and slow down time like gravity? And if not, how does a magnet ATTRACT a distant object? Myles325a (talk) 00:19, 10 May 2008 (UTC)[reply]

I don't think so. As I understand it, the electromagnetic force (of which magnetism is one aspect) works by an exchange of photons, rather than a curvature of space. However, a magnetic field does involve energy, and I believe energy does warp spacetime (E=mc2 and all that), but that warping isn't what we feel as magnetic force. Physicists are working on theories that unify the various fundamental forces, so it's possible there are theories in which EM works in a similar way to gravity by curving spacetime, but in the standard theories EM is described by quantum field theory (exchange of particles) and gravity is described by general relativity (curvature of space) - trying to describe one in terms of the other is very difficult and no-one's quite managed it yet. --Tango (talk) 00:31, 10 May 2008 (UTC)[reply]
This is not such a crazy idea. See Kaluza–Klein theory. Nothing has really worked though. Quantum mechanics gets in the way. I have to run though. Sorry for the shoddy explanation. — gogobera (talk) 02:00, 10 May 2008 (UTC)[reply]
Magnetism follows an inverse cube law not an inverse square law like gravity. The electric force would therefore be more similar to gravity. Magnetism is similar to frame dragging though. Em3ryguy (talk) 12:57, 10 May 2008 (UTC)[reply]
It follows an inverse square law. It's just that every north pole is paired with a south pole and vice versa. The further you get, the proportionally closer the two poles are, and the more their effects cancel. — DanielLC 23:18, 10 May 2008 (UTC)[reply]
Its not really that simple. There are no north or south poles. A single electron moving through space produces a magnetic field that does follow an inverse square law but the field is complex. it is strongest perpendicular to the motion of the electron and nonexistant in front of and behind the electron which therefore makes it even less like gravity. If gravity and the electric force are like the stretching of space then magnetism and frame dragging are like the twisting of space. Em3ryguy (talk) 20:38, 11 May 2008 (UTC)[reply]

OP Myles325a back. Ok gogobera you SHOULD be sorry, because throwing Kaluxi—Zippi …Lapis—Laputski …Lupus—Kaluah …Whatever the hell those jokers are called—at me and then saying BYEEEE…. This is just what the other clown was doing. I would hardly be asking THIS question if I could UNDERSTAND anything on that page, would I? Now, instead of throwing someone to the most esoteric edges of physics, how about starting somewhere closer? Isn't it true that Einstein's General Theory received its first big corroboration when it was observed that light rays were deflected by the Sun's gravity? So, let me ask then, does Magnetism similarly deflect light? And it has been demonstrated that time slows down for objects in strong magnetic fields. So is anything like that observed in strong magnetic fields? And wouldn't such effects be much stronger as magnetism is so much stronger than gravity? And don't magnets just work on metals? In that case, how do those particle accelerators bend deflect those particles in a curve? See, just take it from there, and we will all be soaking up Kalitsy—CalvinKlein or whatever in no time at all. Myles325a (talk) 05:35, 12 May 2008 (UTC)[reply]

A magnetic field only has a very small effect on warping space-time. The reason is that a magnetic field contains energy. But you would need a collosal amount of energy in a small space to have a significant effect, so in practice we do not see it. Magentic fields also work on moving charges. The particles in a particle accelerator are charged and are deflected sidewasy by the magnetic field. If you have a CRT monitor or TV you can hold a magnet near the screen and see the distrotion in the picture. A magnetic field also has an effect on atoms, for example the Zeeman effect, because atoms are made from "moving" charged particles. Magentic fields of extreme levels (too strong for humans to make) will make atoms lose their spherical shape and become long and thin stretched out along the magnetic field lines. This may happen on neutron star surfaces. Graeme Bartlett (talk) 05:49, 12 May 2008 (UTC)[reply]

OP myles325a back. Again. Graeme Bartlett gets me confused by saying:

A magnetic field only has a very small effect on warping space-time. The reason is that a magnetic field contains energy. But you would need a collosal amount of energy in a small space to have a significant effect,…

Now, the ambiguity is: Are you saying that the BECAUSE a magnetic has energy it warps space-time, or are you saying that BECAUSE a magnetic field has energy it has ONLY A VERY SMALL EFFECT on space-time? Also, I thought that magnetism was much more powerful than gravity, as evidenced by my fridge magnets staying on my fridge even though the whole world is pulling them the other way. So why does not magnetism have a greater effect on space-time? And could you, with a sufficiently strong magnetic field, slow time down for particles a la neutron stars? Have any relativistic effects through magnetic forces been experimentally observed? Myles325a (talk) 03:37, 15 May 2008 (UTC)[reply]

I'm sure he is saying that the energy has a tiny amount of mass which produces a gravitational field that warps space. now its true that electric and magnetic fields are similar to gravity and frame dragging but there is no reason to think that they are identical. Em3ryguy (talk) 04:24, 15 May 2008 (UTC)[reply]

Dogs and cats

I sometimes see dogs and cats licking their privates. Do they ever do this to the point that they, shall we say obtain their jollies? FairmontMN (talk) 01:54, 10 May 2008 (UTC)[reply]

You may find Animal_sexual_behaviour#Autoeroticism_.28masturbation.29 interesting. --Tango (talk) 01:58, 10 May 2008 (UTC)[reply]

Conditioning

If you reward a behavior irregularly, it will condition that behavior very strongly, even when the rewards disappear. If you punish a behavior irregularly, it will tend to prevent that behavior very strongly, even when the punishments disappear. What happens if you both punish and reward a behavior, both irregularly? Will the animal shoot up a convenience store? Black Carrot (talk) 02:53, 10 May 2008 (UTC)[reply]

The idea behind this question is so good I imagine animal experiments must have been done on it, although I don’t know of any off hand. One would think it would involve how intense the drives to obtain a particular reward versus escaping a specific penalty were. In the case of things like food, where the need to obtain it can be absolutely vital, then perhaps a subject will risk much more than if the reward was less important. In human experience, some people will rob a bank at the risk of incurring long jail terms IF the potential reward is a very great deal of money. I suppose we all unconsciously work out negatives and positives for any action we are likely to take. After all, any action involves a cost in terms of effort, so everything we do is weighed up against the scenario of what would happen if we did not do it. Myles325a (talk) 03:08, 10 May 2008 (UTC)[reply]

Possibly, but the focus of my question isn't really on whether the animal will continue the behavior or not, it's what the side effects will be. See, if you do this to most humans, they'll get incredibly stressed-out and frustrated, whatever they wind up deciding to do about it. Some humans are pushed so far by conflicts like this that they become needlessly and self-destructively violent. I'm wondering whether animals have the same capacity for stress, and eventual mental illness, that people do, or if they respond to it more dispassionately, as merely a stimulus. Black Carrot (talk) 07:28, 12 May 2008 (UTC)[reply]

One of the assumptions of behaviorist school of psychological thought is that the behaviors are inherently dispassionate. The notion of isolating a specific stimulus and a specific response is hardly a complete theory of psychology, for animal subjects or for humans. Our psychology article has a good section on the rise of behaviorist thought and some of the later ideas that it spawned. Maybe this will give you some context - there are definitely realms where the simplistic experimental view of single-stimulus, single-behavior, single-response mappings do not really hold well, and you have described exactly such a case. Nimur (talk) 16:17, 14 May 2008 (UTC)[reply]
What does this sort of pychobabble mean, Nimur? If you are saying that animals feel no pain, then why bother with the animal cruelty laws? And as post-Darwin we all know that humans are animals as well, why should we get upset concerning human suffering? If someone forced your hand into a pot of boiling water and observed your sweating and screaming and face-pulling, would you approve of his dismissal of your extreme, albeit subjective, pain as an unscientific phenomenon? I would have thought this sort of uber-“scientific” rubbish had been thoroughly refuted by now. Here is a self-explanatory par I added to the talk page of Cetacean intelligence. As the page is transcluded now, I am posting both Nimur and OP Black Carrot this note on their talk pages.

With ref to Michele Bini’s comment above, dolphins are NOT the only animals that engage in self-destructive behaviour when panic-stricken or in extreme pain. All the ‘higher mammals’ including horses, cats, dogs and apes can present with human-like symptoms of severe stress. Dogs which lose a much-loved master can show every sign of ‘nervous breakdown’ and clinical depression, both behaviorally and physically. Other mammals will go on rampages, chew their own fur and eat their own excrement, refuse food and howl incessantly. Apes will throw themselves against their cages. In a series of notorious but well-conducted experiments of the 1960s, researchers tormented dogs to the point where they not only had ‘breakdowns’ but showed every sign of having become permanently insane through terror and pain. These were ‘higher’ animals. I have no idea whether you can make a butterfly mad, or drive a snail to distraction. Myles325a (talk) 04:36, 20 February 2008 (UTC)

Retrieved from "http://en.wikipedia.org/wiki/Talk:Cetacean_intelligence"

Myles325a (talk) 04:13, 15 May 2008 (UTC)[reply]

I think you should see the behaviorism and psychology articles if my comments sound like "psychobabble." I did not even discuss the capacity of animals to feel pain or emotion! I don't see where you drew this conclusion from my statements. Nimur (talk) 14:56, 16 May 2008 (UTC)[reply]

Hi Nimur, thanks for your note re” “what you really meant”. Find below my response, which I am also posting to your Talk Page, and to OP Black Carrot.

There are many things in life that make me want to get down on the ground and pull up carpet tacks with my teeth, and one of the main ones is people who write or say something that reads as muddled or dead wrong and then, after they are taken to task for it, aver that the critic has misunderstood what was intended. The ensuing “debate” takes up everyone’s time, and WP talk pages, and archives, and bandwidth, and patience, and produces much more heat than light.

Your contributions, Nimur, are a copybook example of just such a skew-whiff dialogue. As WP is full of people waving hands and employing phrases that could mean any number of things, I will try to show you why writing clear English and saying EXACTLY what you mean in as concise and unambiguous way as you can, will render you far less misunderstood than you are at the moment. Before I continue might I ask you and others to re-read this and the preceding paragraph? You might not AGREE with the content, but is there anything at all there which you find less than crystal clear? Good. Then let's pass on to your recent offerings. I have put your words in bold type and my own comments in plain.

One of the assumptions of behaviourist school of psychological thought is that the behaviours are inherently dispassionate. Now what significance does dispassionate take on here? Let's not beat around the bush. Does this mean that animals don't feel pain, or that behaviourists don't believe they do, or don't care if they do, or don't believe that pain exists, or don't believe that animals feel pain the way humans do, or consider the subject of subjective feelings of pain as a metaphysical hobbyhorse external to scientific research?

And why would you opt for the murky term "dispassionate" rather than write "behaviours are inherently without emotion"? Dispassionate muddies the waters in this context because it can ALSO mean impartial, disinterested and the like. Are you using it as a "weasel word" because while it connotes "without emotion" it also tempers it with a soupcon of "impartial". Are the animal behaviours themselves "dispassionate" or are the scientists "dispassionate", or is it the methodology itself?

And what is the force of "inherently" here? How is "behaviours are inherently dispassionate" different from the shorter "behaviours are dispassionate". The addition of inherently would suggest that while some external observers might perceive certain animal behaviour as "passionate", objectively, that is "inherently", they are not. If this is not the meaning you intended, perhaps you could explain what you DID intend.

Now let us look at there are definitely realms where the simplistic experimental view of single-stimulus, single-behaviour, single-response mappings do not really hold well… Frankly, Nimur, there are NO interesting cases of animal, or human behaviour, which conform to the single-stimulus, single-response case. I spent a year of a University Psych course under a fanatical behaviourist learning about mice pushing levers for food. In the end, I ascertained that the good Professor preferred to record the highly circumscribed behaviour of mice because, as a scientist, he could not neatly explain what it is that HUMANS were doing, and as a border-line autistic, he was only dimly aware of the world of human experience, and cared even less for it.

A mouse pushes a lever and gets a food pellet. Great! Now, Cindy likes going out and often says "Gee, swell! When do you want to pick me up?" when she gets an invite, uh, sorry, the stimulus of a speech segment over a phone to that effect. But today, Cindy got just such a stimulus and replied to the effect that she was doing her hair. Now, we airy-fairy metaphysical types might just say that's because Cindy did not think that the boy who phoned was a "real spunk rat" or a "hunk". But, and I quote my erstwhile teacher on this, scientifically it should be said that there were "intervening variables" between the stimulus and the response, which made Cindy behave differently. What an absolute laff riot!! Everything—but everything—interesting in this episode lies in these "intervening variables". And as for Cindy, so for all humans, and the great bulk of life. It is another example of weasel words for you to say that the single stimulus – single response model does not (in some cases) old really well. Apart from jumping up when you sit on a tack and the like, there is NOTHING in life which can be described by such a mechanism, and it is an absolute indictment of the entire psychological "profession" that it was not laughed out of business when it was first proposed. So, do tell us, Nimur, what does "does not hold really well" mean?

Nimur, if you cannot quantify the stimulus – response model in life, can you at least quantify how often and in what circumstances human responses DO conform to such models? After all, if we are unable to record and quantify human responses, then we should be able to at least quantify the success to failure ratio of its predictions. My own estimate is that, after all those experiments, the success rate is close to zero. Millions of mice and pigeons pecked and pushed at levers and ran down mazes, and in the wash-up, decades later, I think it was B.F. Skinner himself who gave a description of how behaviourist theory can have some practical significance. Some college was having problems with student stragglers coming late for lunch and thus keeping kitchen staff waiting. Skinner suggested ringing a bell that would summon the students and then denying lunch to those who came more than 30 minutes later! Yes, we are indebted to Skinner and the behaviourists for this and many other such breakthroughs.

You end your piece on another weasel note. After noting that you did not say that animals did not feel pain, you finish with I don't see in any way how this has anything to do with the capacity for the animal to feel pain, nor the ethics of animal testing While OP Black Carrot's original post does not directly deal with animal cruelty, it broaches the subject of highly stressful / painful experiments in which an animal is rewarded and punished randomly for the same behaviour, and links it directly will existential pain felt by humans who might be exposed to such treatment in a social setting. It is hard to credit that you really have no idea how any of that can impinge on the broader topic of animal cruelty.

Moreover, I do not see why you simply do not declare yourself and say that what it is that you DO believe in this regard. Why duck the issue of animal (or indeed) human pain by sweeping it under the carpet of "complex behaviours"? And the question of animal pain is not a red herring. Descartes publicly propounded the theory that animals were no more than machines—a prototypically behaviourist notion—and that the noises they made when they were killed were no different in type to those of a creaking wheel. France, which still venerates Descartes has—for a European nation—a backward attitude toward animal suffering precisely for this reason.

But to get back to the main thread. Nimur, if you said exactly what you INTENDED clearly and without ambiguous weasel-words, then you would find yourself misunderstood on fewer occasions. As it is, if I have misunderstood you, then the preceding will give you fair indication why. Myles325a (talk) 04:55, 17 May 2008 (UTC)[reply]

sauna temperature control

How does the copper tubing temperature sensor in a sauna provide feedback to the electrical control box to maintain the temperature?

Will throwing some water on the copper tubing sensor damage the control box?

Will intermittent water squirted on the copper sensor eventually damage the sensor over a period of time? Jaimegonzo (talk) 03:05, 10 May 2008 (UTC)[reply]

I can't say I know exactly what the “copper tubing temperature sensor in a sauna” is, but I imagine that you might start with Bi-metallic strip#thermostats. Maybe that's entirely not what you're wondering about, though?
About the water… it seems likely to depend on the housing for the control box -- is it water resistant? Does the thermostat rely on electrical conductance at all? I would hazard a guess that in sauna temperatures, if “throwing some water on [it]” doesn't cause problems, then “intermittent water squirted on [it] … over a period of time” won't do much either. Of course, “period of time” is a bit vague. :P  — gogobera (talk) 06:02, 10 May 2008 (UTC)[reply]
If you have a real Finnish sauna it will tolerate water, as throwing water on the stove ("kiuas") is an integral part of sauna. A Finnish kiuas that doesn't tolerate water is a nonsensical concept. Check the stove manufacturer's instructions though; other types of steam rooms from other countries might not be designed for water. If the temperature sensor is inside the stove it will be protected from water. The sensor can also be inside a thermometer on the wall, turning the heating elements on and off based on air temperature rather than stove temperature. Weregerbil (talk) 09:43, 10 May 2008 (UTC)[reply]
The copper tube is likely a thermocouple. (A friend of mine recently cut one like this on a hot tub when he assumed it was a grounding wire). -- Flyguy649 talk 14:31, 10 May 2008 (UTC)[reply]
In thermostats, the copper bulb is usually just a gas-filled copper bulb, connected to the thermostat by a very thin copper tube called the capillary. Within the thermostat, the pressure of the gas within the copper bulb bears on a bellows and that presses on a mechanical switch. Heat the bulb enough and the mechanical switch opens the electrical contacts, shuts the gas valve, or what-have-you, shutting off further heating until the copper bulb cools down again.
Atlant (talk) 16:00, 12 May 2008 (UTC)[reply]

Rube Goldberg pendulum

In a Rube Goldberg machine I am building, a marble hits 'something' which causes a pendulum on the top of a tower to swing back and forth a few times before it hits another marble which rolls down the tower. I'm having some trouble deciding what must be used to do what I just described. What should that 'something' I mentioned earlier be? --hello, i'm a member | talk to me! 03:29, 10 May 2008 (UTC)[reply]

In truth I have some ideas, it wouldn't hurt if I get some more. --hello, i'm a member | talk to me! 03:30, 10 May 2008 (UTC)[reply]

No ideas but what you said doesn't really make sense to me. The pendulum will swing the greatest distance on the first swing. Thus it will reach the marble on the other side of the swing on the first, not necessarily the second, third, etc. swing. Dismas|(talk) 03:38, 10 May 2008 (UTC)[reply]
True if the pendulum swings in one plane. If the 'something' can provide an offset that causes the pendulum to oscillate in two dimensions, it can oscillate through several perioed before striking the target. Example: The pendulum bob might be held at an angle tothe vertical by a hook. the 'something' strikes the bob at an angle that dislodges it from the hook and imparts momentum that is not in the plane of the top of the pendulum. -Arch dude (talk) 04:07, 10 May 2008 (UTC)[reply]
This is what is so difficult about creating a pendulum to the marble on top: how do I create the nesscesary torque to cause the pendulum to oscillate like such (so that it eventually hits the marble)? --hello, i'm a member | talk to me! 04:21, 10 May 2008 (UTC)[reply]
Have you considered driving the pendulum by using another pendulum? Suspend two pendula from one crossbar such that both pendula have the same period of oscillation. Set one pendulum in motion. The system will be a bit 'leaky', and energy will be transferred from the moving pendulum to the stationary pendulum, driving it to oscillate. (The effect only works if both pendula have the same period.) TenOfAllTrades(talk) 06:00, 10 May 2008 (UTC)[reply]
Addendum: It's a fairly popular physics demo; here's a YouTube video of the principle in action: [1]. TenOfAllTrades(talk) 06:03, 10 May 2008 (UTC)[reply]
For another idea of how to get torque look at Driving wheel. We may have a page that describes it better, but I don't know where. Maybe s.o. else does. Otherwise just look at a model train. 71.236.23.111 (talk) 13:47, 10 May 2008 (UTC)[reply]

What is a "geographic feature"?

I've just created the article geographic feature, but I'm not sure I've defined the class entirely correctly. I took some of the material from other Wikipedia articles, and I don't know if they were correct.

For example, are countries and other administrative divisions geographic features? What about imaginary lines like borders, the Equator, etc.

I said they weren't (except for settlements), but I'm not 100% sure.

Wikipedia isn't consistent on geographic features, landforms, etc., and it isn't clear what is or is not a geographic feature. The article landform implies that landforms are not geographic features.

I couldn't find a definitive treatment of what is or is not one, so that leaves a bunch of things up in the air:

What about national parks?

Nature reserves?

What about orchards?

And then there's the sea floor, and its features, like trenches, submerged reefs, etc.

I look forward to your edits and comments.

The Transhumanist 07:28, 10 May 2008 (UTC)[reply]

Political boundaries and the like are not geographical features. That's why they are not invariably found on geographic (=physical) maps; they are found on political maps. - Nunh-huh 07:41, 10 May 2008 (UTC)[reply]
In the field of GIS, as well as cartography to some degree, a geographic feature is anything spatial you are representing on a map or in a geodatabase. The usual term is just "feature", but it's not uncommon to hear "geographic feature". I would argue that political boundaries and the like are geographic features. The field of geography includes political geography, not just physical geography. Pfly (talk) 07:59, 10 May 2008 (UTC)[reply]
I agree. It seems bizarre to me to say that political boundaries are not geographical features; maybe this is not as well-defined a concept as you seem to think. And while this is not the place to debate it, I don't see what the point is of having an article about geographical features in the first place. If they're things shown on maps, why shouldn't that article say whatever is to be said about them? --Anonymous, 08:30 UTC, edited 10:27, May 10, 2008.
Here is a great method of finding out what is a geographical feature and what isn't. Position yourself in a landscape, and point at anything that you know is there. Is it visible? If so, you've located a geographical feature. If it is a resident house, it is a very minor geographical feature, and a man-made one at that. If it is a tall mountain, it can be considered a major geographical feature. If it is a river, it is also a geographical feature, notable if it is big, not notable if not so big. What does the geography feature? It may feature a border station with customs, barracks and wire. The border control station is a feature - the political border is not. The wall of wire is a feature (major if big, minor if not), but is entirely disjoint from what is defined to be the end of a geographically defined administrative region. Hereupon lies the source of confusion with regards to wether or not a border is a geographical feature. Political boundaries are not geographical features. The wall the Israelis built just recently is, however, a geographical feature that signifies a political boundary.
I don't wish to take to the refdesk what should otherwise be on the article's talkpage, but you aired a question here, and as such it deserves my best answer. Scaller (talk) 12:19, 10 May 2008 (UTC)[reply]
Note that some non-geographic features are often included on geographic maps simply to help us locate the geographic features. State, provincial, and national boundaries and large cites might be included, for example. In the case of a map of California fault lines, it would be useful to know where Los Angeles and San Francisco are located. StuRat (talk) 20:58, 10 May 2008 (UTC)[reply]
I think what Anonymous said about the terms being only loosely defined is right. I know the word "geographic" is often used for physical things only, which would leave political boundaries out. But this usage is not the only one, nor necessary "correct". I just went and picked up a book I have called "Modeling our World: The ESRI Guide to Geodatabase Design". Here's a quote from page 25: "Geographic features are located at or near the surface of the earth. They can occur naturally (rivers, vegetation, and peaks), can be constructions (roads, pipelines, and buildings), and can be subdivisions of land (counties, land parcels, and political divisions)." Pfly (talk) 21:21, 10 May 2008 (UTC)[reply]
Another example is the USGS Geographic Names Information System. Although they mostly use just the word "feature", as in "feature name", "feature class", etc, they do use the term "geographic feature" on their FAQ page: "GNIS public Web site: Directly queries the database for official geographic feature names, their locative attributes, variant names, and other data..." The geographic features in the GNIS database all kinds of non-physical things, like counties, states, etc. Pfly (talk) 21:49, 10 May 2008 (UTC)[reply]

Screech of Chalk on a Blackboard

Has anyone done a Fourier analysis on the screeching sound which can be made by chalk or fingernails on a blackboard. Does anyone know why this sound is so unpleasant. 90.206.167.42 (talk) 10:49, 10 May 2008 (UTC)[reply]

Yes [2], and surprisingly the answer is the middle frequencies make it horrible, not the high frequencies. There is some suggestion that the sound is similar to primordial alarm calls and this is what makes us sit up and take notice. You can take part online in that sites study into what makes horrible sounds horrible. But be ready to cringe before you turn up the volume! SpinningSpark 13:20, 10 May 2008 (UTC)[reply]
And of course, I should have known, Wikipedia really does have an article on everything.SpinningSpark 13:23, 10 May 2008 (UTC)[reply]

determining the sugar content of grapes

hi, can anybody tell me how i would find the sugar content of grapes using the equipment found in a small lab? —Preceding unsigned comment added by 82.45.96.96 (talk) 10:56, 10 May 2008 (UTC)[reply]

Homework alert! Makey melly (talk) 11:10, 10 May 2008 (UTC)[reply]
Have you another looked at Sugar#measuring sugar ? SpinningSpark 14:48, 10 May 2008 (UTC)[reply]

Aircraft as flying disc

Can an aircraft able to transport a human being have the form of a flying disc? 217.168.4.133 (talk) 12:44, 10 May 2008 (UTC)[reply]

Yes it can, take a look at Avrocar, British Rail flying saucer and also Military flying saucers is interesting. There is also Lenticular Reentry Vehicle although this was not intended as human transport. SpinningSpark 13:03, 10 May 2008 (UTC)[reply]

Accelerated Thought.

The concept of gaining command over the matter, that is the entire nature through the process of accelerated thought demands attention. Is there any model which presents the operative dynamics of "accelerated thought"? —Preceding unsigned comment added by Dr.Ramakrishnan (talkcontribs) 13:22, 10 May 2008 (UTC) Dr.Ramakrishnan (talk) 13:31, 10 May 2008 (UTC)[reply]

Psychokinesis and James Randi --71.236.23.111 (talk) 14:19, 10 May 2008 (UTC)[reply]
I don't think this is about telekinesis. I think he means "How can we improve web-based education?". --Heron (talk) 14:24, 10 May 2008 (UTC)[reply]
Perhaps you are asking about the Future Shock effect, which predicted an ever-increasing rate of technological advance. IMHO this effect has failed to materialize. Can you clarify your question please ? StuRat (talk) 20:50, 10 May 2008 (UTC)[reply]

Photon energies in cosmological redshift.

Asked my physics teacher a rather obvious question after being taught about cosmological redshift the other day: If the energy of a photon is given by E=hf (planck's constant * frequency), yet the frequency decreases over time due to the expansion of the universe, then doesn't this violate conservation of energy? Won't photons from distant stars lose energy before they reach us? Where does the energy go?

I've had four thoughts about this:

1. Is planck's constant really a fundamental constant? Could it change related to the age of the universe so as to conserve photon energies?

2. Is the problem here that cosmological redshift (though not redshift due to relative motion, which I understand has energy conserved in a specific reference frame) is a prediction of general relativity, which is incompatible with quantum mechanics (which is where I assume the E=hf equation comes from)?

3. Moving on from the last point, does general relativity actually include conservation of energy? what about other conservation laws?

4. If it does, then is photon energy somehow conserved in a specific reference frame within general relativity, and the apparent loss of energy due to considering the photon from different reference frames?

Or is it something else altogether?

JMatopos (talk) 13:26, 10 May 2008 (UTC)[reply]

Photons do lose energy as the universe expands, and the simplest explanation is that it becomes potential energy of the gravitational field. But in a broader sense, energy actually isn't conserved in general relativity. There's a good (though somewhat technical) discussion here. On your four thoughts: 1. It's possible that the fundamental constants could change with time, but this has been tested and ruled out to pretty high precision. So this can't explain the loss of energy, and anyway it's predicted by general relativity without changing the constants. 2. There's nothing wrong with E = hf, but you could say the lack of gravitation in the standard model is the problem. 3&4. General relativity does predict differential energy conservation (the energy at a point equals the energy that was there before plus the energy that flowed in from elsewhere minus the energy that flowed out). But this has to be taken with a grain of salt because the "energy" here doesn't include gravitational field energy. Energy from the photons gets transferred to the gravitational field despite this law seeming to say it can't. Defining an "energy of the gravitational field" is surprisingly difficult. The page I linked says more. -- BenRG (talk) 14:46, 10 May 2008 (UTC)[reply]

Proving alcohol

Suppose, I'm filming a glass of vodka (or, generally any alcohol). How can I prove to the audience that this is vodka, not water? A litmus paper? --85.132.14.38 (talk) 13:43, 10 May 2008 (UTC)[reply]

Alcohol is flameable, right? Setting fire to it would prove its not water. Would spoil the drink though. —Preceding unsigned comment added by Makey melly (talkcontribs) 13:48, 10 May 2008 (UTC)[reply]
The simplest way isn't exactly scientific, but take a new, sealed bottle of vodka and open it in front of the camera. Any scientific test is just likely to confuse people, and could probably be faked anyway. I mean, if litmus paper worked, you might just end up trying to prove that it's real litmus paper. If you set fire to it, what's to say it's not just a liquid hydrocarbon or some other flammable substance? JMatopos (talk) 13:51, 10 May 2008 (UTC)[reply]
Well what is to prove you didn't fill a vodka bottle with water and then reseal it? Or change the label on the bottle? You could do all three; open a vodka bottle, drink some, set fire to some etc to prove beyond doubt. Or have a witness to say "yes, that is vodka". Makey melly (talk) 13:58, 10 May 2008 (UTC)[reply]
Yet in my mind, any attempt to 'prove' that it's vodka on camera would immediately raise my suspicions that it wasn't. Removing the foil seal on a new bottle of vodka is believable, natural and easy to do. Not that all these tests are a bad idea; I mean if you really wanted to 'prove' it, you could set some on fire, use a pH meter to show it's very mildly acidic and give some to a Russian who swears on his life it tastes like vodka. Seems like a lot of hassle, though. :-/ Besides, if I were watching this video, there's no way you'd prove to me 'beyond doubt' it was alcohol unless you personally gave me a sample to run through an IR spectrometer, and I figure that isn't an option. :) JMatopos (talk) 14:20, 10 May 2008 (UTC)[reply]
Having someone in the audience take a sip sounds like a best idea to me. --Tango (talk) 14:04, 10 May 2008 (UTC)[reply]
Dropping in some oil might work. It would separate out and float to the top in water. Breathalyzers use IR spectroscopy if you want something fancy. 71.236.23.111 (talk) 14:10, 10 May 2008 (UTC)[reply]
Ah, but how does he prove that the bottle of oil actually contains oil? Really, there's essentially no way to conclusively 'prove' the identity of a liquid in a videotape—virtually any conceivable test could be faked through mislabeling the reagents used, tampering with the instruments, swapping liquids during a cut between cameras, or using special effects. At some point, you have to rely on the trust of your audience.
The question here might be, "What property of the liquid is actually important?"
  • To show that it's nontoxic, just drink some.
  • To show that it's not straight water, use it to dissolve an ink stain. Test in advance to find an ink that will run in 40% alcohol but not (very much) in water.
  • To show it's not very viscous, slosh it around a bit.
To demonstrate much more than that, you're into a difficult place. Those three tests will demonstrate that it's a drinkable liquid of comparable viscosity to water with some hydrophobic solvent properties. The things that fit those criteria (especially the "I'm willing to drink it" test) are mostly distilled beverage spirits: vodka, white rum, gin, etc. Is that sufficient for your purposes? TenOfAllTrades(talk) 16:26, 10 May 2008 (UTC)[reply]
Of course one further problem is that people could easily feel the video may be faked. For example, it wouldn't be that hard to design a video that to the untrained eye looks like a continuous shot but in fact contains different shots and you could use one drink for the non-toxic part, one drink for the ink part and one drink for the not viscious part and one final drink for where you actually perform the final test on the alcohol solution (I guess there's a point to proving something is alcohol). Of course, you could add a billowing curtain or something similar in the background to help make this more difficult and you story more plausible. It would also be a lot more difficult, even without a curtain to ensure your video fools an experts. This really depends on a lot of things. If for example you are attempting a magic trick/illusion, some of the suggestions from TOAT or others combined with your assurances to the audience that it is alcohol and no camera tricks were used may be sufficient for many viewers. Obviously not everyone is going to be satisfied, but these sort of things generally require some degree of trust from the audience anyway Nil Einne (talk) 19:58, 10 May 2008 (UTC)[reply]
The problem to me isn't that you can "prove" it, but that it's illustrated to be Vodka -- whether the demo is faked or real, for the purposes of a video it illustrates your point. If it's to record an experiment then you;d be playing it straight for teaching purposes. Otherwise, I'm thinking advertising e.g. with two glasses, one labelled (our brand), and one (other brand) for example. Viewers know it's a setup but they suspend disbelief operating through the filmic "wall". Actors don't exactly scull drinks with litmus paper in them. Otherwise like comments above, you can also discredit your demo. Btw, if using a glass of vodka with the oil drop, it seems appropriate to have a glass of water + oil to compare results. Apols if this is all too beside the point, Julia Rossi (talk) 02:24, 11 May 2008 (UTC)[reply]
Goldfish. Put goldfish in the alcohol - dead goldfish proves its not water. Of course, it could still be turpentine, so personally I would not drink any on the basis of that proof, but it is definitely not water. SpinningSpark 20:52, 11 May 2008 (UTC)[reply]
give some to a member of the audience. --Shaggorama (talk) 05:56, 15 May 2008 (UTC)[reply]

aurora borealis from space

Can the aurora borealis be seen from space? Mr Beans Backside (talk) 13:55, 10 May 2008 (UTC)[reply]

Yes; see http://en.wikipedia.org/wiki/Image:Aurora_australis_20050911.jpg and http://en.wikipedia.org/wiki/Image:Aurora_Borealis.jpg - Bear in mind that the first image is a composite and has been enhanced, though - it's not really that bright. JMatopos (talk) 14:01, 10 May 2008 (UTC)[reply]
Yes. You can even see aurora on other planets (see Aurora (astronomy)#On other planets). --Tango (talk) 14:03, 10 May 2008 (UTC)[reply]
Absolutely. Here are links to some NASA images: [3], [4]. POES (Polar Operational Environmental Satellite) observes aurorae at the north and south poles; reports are available here, among other places. TenOfAllTrades(talk) 14:09, 10 May 2008 (UTC)[reply]

Why are magpie's tails so long?

What is the explanation for the length of the magpie's tail. It must be an adaptation to something, but to what? Since both sexes have the same tail length it cannot be sexual selection. 217.42.89.22 (talk) 14:17, 10 May 2008 (UTC)[reply]

I disagree that it can't be sexual selection. Both males and females may choose mates that have a long tail, as that's a sign of good health and genes. StuRat (talk) 15:17, 10 May 2008 (UTC)[reply]
Just a thought: Long tails are good for balancing. This may be a factor here. Since there are long and short tailed species (e.g. cats, monkeys), this is not a very selective trait, but it might cause a new species to split off. 71.236.23.111 (talk) 15:19, 10 May 2008 (UTC)[reply]
Every magpie in my area has a certain green-blue tint to their tails, very much like the plumage of a pigeon (bar then the red/purple). I will therefore present the argument that it can have a sexual purpose. Scaller (talk) 17:39, 10 May 2008 (UTC)[reply]
Edit: See this -> http://upload.wikimedia.org/wikipedia/commons/8/8b/Elster_wikipedia2.jpg Ah, what a beautiful picture. Serves my purpose perfectly, even. Scaller (talk) 17:41, 10 May 2008 (UTC)[reply]
See this research paper on the correlation between tail length and general health in black billed magpies. It looks like they are an honest signal on the overall health of the bird to prospective mates in boths sexes. Multiple elements of the black-billed magpie's tail correlate with variable honest information on quality in different age/sex classes. Lumos3 (talk) 22:19, 10 May 2008 (UTC)[reply]

machine that destroyeds itself

Who invented a machine that destroyed itself on purpose? Mr Beans Backside (talk) 14:44, 10 May 2008 (UTC)[reply]

The Manhattan Project. FairmontMN (talk) 14:46, 10 May 2008 (UTC)[reply]
...or for that matter any explosive, used in war or peace, like blasting caps. I suppose an artist might also create such a thing for "artistic reasons". There are also disposable machines, like a disposable camera. Finally, there is "planned obsolescence", whereby a device intentionally fails after some period of time to make you go out and buy another. StuRat (talk) 15:23, 10 May 2008 (UTC)[reply]
Possibly you are thinking of Jean Tinguely [5].--Eriastrum (talk) 19:32, 10 May 2008 (UTC)[reply]
If with "machine" you are referring to the device commonly known as homo sapiens, all you have to do is to find the name of the inventor. --Cookatoo.ergo.ZooM (talk) 23:22, 10 May 2008 (UTC)[reply]
Someone recently reported in the Risks digest that he had been awakened one night by the carbon monoxide detector in his home beeping every 30 seconds, like the way some devices signal a low battery, but in this case it was displaying Err. He located the manual on the Web and learned that the manufacturer said the device should be replaced after 7 years -- and that they enforced this by having it count 7 years and then automatically stop working and go into this error state. (Wouldn't it be fun to be the company's lawyers if someone died of CO poisoning because this happened and then there was a genuine, undetected CO emergency before they could get to a store to replace the detector?) --Anonymous, 09:00 UTC, May 11, 2008.
Outside the US, I don't see how this would be a problem. If the device started to beep once it entered into a deactivated mode, then the person would either put up with it or turn it off. If you've turned off your device or are ignoring the beeping, you can hardly complain to the company that the device didn't detect CO. Outside the US at least, this would probably be far better for the company then if their device were to be ineffective after 7 years but if the device didn't warn you of this fact Nil Einne (talk) 17:18, 11 May 2008 (UTC)[reply]
I believe the person who posted the story was in fact in the US. In any case, I think there is certainly cause for complaint if the device deactivates itself without advance warning (and I don't consider the fact that the manual mentions it to be adequate warning). Even if one accepts that the basic idea is sensible, it could have started beeping a month ahead, for example. --Anon, 04:54 UTC, May 12, 2008.
Who's to say the device wasn't still working anyway? Did someone actually test it? There's a good chance the err mode still would have gone to emergency mode if there as a leak. In any case, any beeping is liable to result in the person with the device either ignoring the beeping (rare, I doubt anyone could stand the intermitent beeping for very long if it's like a smoke alarm) or turning it off, which as I've already stated, defeats the purpose of the device. Note that it's the manufacturer is going to say "REPLACE DEVICE NOW, IT NO LONGER WORKS" the same way they say "REPLACE BATTERIES NOW, IT NO LONGER WORKS" but I'm pretty sure if you actually test most devices, they may still work provided the problem doesn't stop them from working (which is one of the reasons why they tell you it no longer works). It's generally a bad idea to actually make a device completely deactivate since there's always a risk the device may do it without giving the proper warning somehow. Note that if the person wasn't already aware they should replace the device in 7 years or it's going to start beeping, which we can assume they weren't since they didn't replace the device, then they alway likely weren't aware they should replace the device in 7 years because it's likely to be ineffective after then. It is FAR, FAR better that they were actually made aware of the fact that the device was no longer effective then it is for them to go along their merry way with a non-beeping but ineffective device. Moving the time frame forward doesn't actually achieve anything, it just makes the problem worse (besides I highly doubt the device actually started beeping at exactly 7 years). Note that if you're referring to some sort of complex electronic alarm system which warns the user until he/she turns it off 1 month before, then say more urgently 15 days before, then say every day a week before, while this may sound good in practice, in reality something like this is 1) expensive 2) far more likely to go wrong meaning that in pratices it's simply a bad idea. A simple device which goes into 'error' mode when it has a problem (like the person forgot to replace it even though he/she was supposed to) is far, far better. Let's not forget in the end the only reason anyone will ever encounter this is because they were stupid enough to not replace their device when they were supposed to.Nil Einne (talk) 16:05, 12 May 2008 (UTC)[reply]
I think one could go so far as to say that a primitive animal trap like a rock held up by a precarious stick with bait on it is a self-destroying machine. --Shaggorama (talk) 05:59, 15 May 2008 (UTC)[reply]

airport security system x-rays (part I)

Are the airport security system x-rays the same in strength at those at the dentists? If so, why do the dentists run out of the room when they use the x-rays but the airport staff are next to them all day long? Mr Beans Backside (talk) 14:55, 10 May 2008 (UTC)[reply]

The X-rays at airports are contained within the box that checks your luggage by lead, whereas your dentist has no such protection from the X-rays.
Not sure about the relative strength, but the X-ray machines at the airport are shielded by those strips that fall down on either end, while the one at the dentist's office is not shielded, so that makes a big difference. StuRat (talk) 15:14, 10 May 2008 (UTC)[reply]
As a note, there are different grades of X-rays at airports. Checked baggage undergoes more intense x-raying than carry-on baggage, to the point that undeveloped film in checked baggage can end up getting fogged, whereas film in carry-on luggage will not. --98.217.8.46 (talk) 17:30, 10 May 2008 (UTC)[reply]
From what I can tell, this isn't universally true [6] Nil Einne (talk) 17:11, 11 May 2008 (UTC)[reply]

airport security system x-rays (part II)

Does the average airport security system x-ray the contents of your stomach? For example if you were hiding something illegal in there would they find it? Mr Beans Backside (talk) 14:56, 10 May 2008 (UTC)[reply]

Only if you jump on the conveyor belt along with your luggage. The thing you walk through is just a metal detector, not an X-ray machine, so it would only detect large metal objects in your stomach. StuRat (talk) 15:05, 10 May 2008 (UTC)[reply]
Oh, thank you. That clears a few things up. Mr Beans Backside (talk) 15:11, 10 May 2008 (UTC)[reply]
The metal detector would not detect anything other than metal in your stomach. You should probably check out this legal disclaimer and this medical disclaimer though, I seriously wouldn't advise trying it. It's dangerous from a legal and medical point of view. Regards, CycloneNimrodTalk? 15:13, 10 May 2008 (UTC)[reply]

rechargeable batteries

Can i use rechargeable batteries in television remotes? The instruction manual says not to use them. What damage would it cause? Mr Beans Backside (talk) 14:58, 10 May 2008 (UTC)[reply]

Generally it is best to follow instructions for the sake of safety, but in this case i'm pretty sure the only difference would be reduced performance in terms of the life of the battery. Regards, CycloneNimrodTalk? 15:08, 10 May 2008 (UTC)[reply]
I think it's more of an issue of the rechargeable batteries not having sufficient voltage to operate the remote, especially after they've been recharged hundreds of times (they get progressively weaker with each recharge). StuRat (talk) 15:09, 10 May 2008 (UTC)[reply]
Yes, the difference is 300mV per cell (NiMH or NiCd rechargeable against Alkaline). So assuming your remote takes two batteries it will start off 0.6V too low and then steadily deteriorate. Couple that with NiMH do not perform at their best in low drain/infrequent use applications you may find you have to take the batteries out to recharge quite often. On the other hand, disposable batteries in a TV remote would typically last a year or two, close to the shelf life of the battery. Another common problem with rechargeables in this area (though possibly not usual for TV remotes) is that any device with a low battery warning calibrated for alkalines is going to go off way too early. In short, I agree with the first answer - follow the instructions. SpinningSpark 17:08, 10 May 2008 (UTC)[reply]
Rechargeable batteries also have a very noticeable self-discharge phenomenon. That is, even without a load, they discharge themselves over a fairly short period of time (a few months). By comarison, I find that a fresh alkaline battery used in a typical infrared remote control can last several years in that very-light-duty service. This makes the alkaline battery more appealing to me.
Atlant (talk) 15:48, 12 May 2008 (UTC)[reply]

Soil moisture measurement (copied from Misc. desk)

The following is on the misc desk. Doesn't seem to work there, so I copied it here: (don't know how to move stuff)71.236.23.111 (talk) 16:40, 10 May 2008 (UTC) Which method would be more reliable to measure soil moisture: resistance or capacitance? Does different soil type contribute much to the measured resistance/capacitance? Does capacitance method still work at freezing temperatures? I'm working for a automatic watering controller using soil moisture measurements and the accuracy doesn't have to be very high, but I do want it to work reliably between different soil types. --antilivedT | C | G 07:01, 10 May 2008 (UTC)[reply]

Take a look at Frequency domain sensor which seems to give some of the answers to this and Soil moisture#Geophysical methods has links to some other probe types. SpinningSpark 17:22, 10 May 2008 (UTC)[reply]

What's the name of the fish species...

... in which the male is far tinier than the female, and ends up living parasitically inside her vagina (or fishy equivalent)?

thanks, Adambrowne666 (talk) 17:59, 10 May 2008 (UTC)[reply]

Anglerfish. --98.217.8.46 (talk) 18:04, 10 May 2008 (UTC)[reply]
You might note that although the male does become a parasite attached to the female's body, this is not inside her oviduct.--Eriastrum (talk) 19:42, 10 May 2008 (UTC)[reply]
Thanks very much - so am I conflating two species then? - is there a happy, nonparasitic fish that does live in the oviduct? Adambrowne666 (talk) 23:23, 10 May 2008 (UTC)[reply]

Ridiculous genomic databases and their stupid lies

I'm trying to get a gene sequence around its transcription start-site, so I'm using this Japanese website: http://dbtss.hgc.jp/ - I am interested in the TJP1 gene: http://ensembl.genomics.org.cn/Homo_sapiens/geneview?db=core;gene=ENSG00000104067

When, on the Japanese database page, I set the category to Ensemble(ENST), and insert the code ENSG00000104067, I get the message: Sorry, there is no hit data. Your search keyword is 'ENSG00000104067'. Is there an alternative to way? ----Seans Potato Business 20:00, 10 May 2008 (UTC)[reply]

Sorry this may be way off, but would this help? [7] —Preceding unsigned comment added by Lisa4edit (talkcontribs) 22:45, 10 May 2008 (UTC)[reply]
Thanks, Lisa. If I go to the page you linked and click on the "genomic sequence" link on the left panel, it seems to give some potentially-useful information but I'm having trouble figuring out what's being displayed. The 'flanking sequence' form fields seem to have no effect and I can't find the transcription-start site. ----Seans Potato Business 23:09, 10 May 2008 (UTC)[reply]
That would be under "Gene information" and there's a link there for the start, at least I think that's it. --Lisa4edit (talk) 23:38, 10 May 2008 (UTC)[reply]

Personally, I use this site for all of my searches. Wisdom89 (T / C) 17:42, 11 May 2008 (UTC)[reply]

What settings do you use? I used Swiss-PROT/TrEMBL and search for TJP1 and then I get an information page, with links to other pages which include the sequence. But there's no way of telling where the transcription start site is. That ExPASy site seems to be mostly dedicated to proteins. ----Seans Potato Business 21:25, 11 May 2008 (UTC)[reply]
Why don't you export it directly from Ensembl? Find the gene you want, then choose the transcript you want (e.g. [8]). Then note where it says the transcript is found ("Chromosome 15 at location 27,778,863-27,901,974."). Since the gene is on the reverse strand, that means the beginning of the transcript is at 27,901,974bp on C15. The you can export the sequence from Ensembl. On the tab on the left, click on "Export transcript data". [9]. On this page you can choose the sequence you wish to export. If you would like to see 50bp either end of the transcriptional start you should add and subtract 50 from the start value: ("Chromosome 15, Bp 27901924, Bp 27902024") [10]. Choose your output format and, viola... you have the transcriptional start context. Rockpocket 22:21, 11 May 2008 (UTC)[reply]
If I go here: [11] to export a sequence and either a) retain the transcript value and continue (0 upstream and 0 downstream bp added) b) change to gene and enter tjp1 and continue, neither of the presented sequences begin with the start codon. Thinking more about the word 'transcript', I figure that in this context it just means the DNA sequence that is transcribed, so I'd expect a start codon at beginning of this sequence, even if not for the gene (which I suppose includes promotor info?). --Seans Potato Business 00:02, 12 May 2008 (UTC)[reply]
Transcripts don't typically begin with a start codon. They have something called 5'UTR first. If you want to start your sequence with an ATG then you want the cDNA sequence, not the transcript. Rockpocket 01:33, 12 May 2008 (UTC)[reply]

Seagulls

What was the natural habitat of the seagull before humans started interfering with the environment and encouraged them into towns and cities? Are there any seagulls left that still live as nature intended? --62.136.203.112 (talk) 23:30, 10 May 2008 (UTC)[reply]

We have an article on seagulls, you know. Anyway, as the name might suggest, they're seabirds, typically nesting on seasides. Their natural diet includes fish and other sealife, although they're opportunists and will readily exploit other available food sources. —Ilmari Karonen (talk) 23:50, 10 May 2008 (UTC)[reply]
Beat me to it. : Seagull isn't all the same. Our Gull page lists quite a few species. Some fish, some scavenge, some hunt other birds and eat eggs and some steal the catch from others. Most gulls live in costal areas. Only some species live near humans. (I guess if you eat dead tuna or dead duck it doesn't matter much if it has been in a can or trashcan before, or is "fresh" from the beach. :-) Lisa4edit (talk) 23:58, 10 May 2008 (UTC)[reply]


May 11

absolute zero

Does all motion stop at absolute zero —Preceding unsigned comment added by 74.47.169.87 (talk) 01:05, 11 May 2008 (UTC)[reply]

Take a look at Absolute zero, the first couple of lines should answer your question. --Tango (talk) 01:10, 11 May 2008 (UTC)[reply]
Expanding on this a little. All particles with mass would be stationary, but all massless particles would be traveling at the speed of light as they always do. It would be very interesting to see what a neutrino did under such conditions. GameKeeper (talk) 22:47, 12 May 2008 (UTC)[reply]

Why does this microwave have electric-plug-shaped ventilation holes?

The microwave ventilation holes at the bottom are shaped like a variety of electric plugs.

Microwaves have ventilation holes in the rear; but after closely investigating many microwaves, we noticed that there is consistently inclusion of "ventilation holes" which are exactly shaped and sized like various international electric plugs. Any ideas why this exists? We have exhausted a lot of possible explanations to no avail. There is no electric connectivity to these plug holes, they just go through the sheet metal casing. Ideas? Saket (talk) 04:47, 11 May 2008 (UTC)[reply]

Just a wild guess, but it could be because they used those holes to test whether plugs were shaped correctly during some sort of Q&A analysis. Seems a bit weird, though... Titoxd(?!? - cool stuff) 04:54, 11 May 2008 (UTC)[reply]
Here's a random idea - maybe that rear panel of the microwave is used in several different models of microwaves, some of which have power inputs in that location? That could explain the various power-plug shaped holes. Although it seems odd that the female power junction is on the microwave - usually it's just on the wall outlet... it's just a wild guess. --Bmk (talk) 05:02, 11 May 2008 (UTC)[reply]
Or perhaps it's part of some metal-shaping quality control procedure in the factory? Kind of like the colored patches you can find under cardboard flaps on cereal boxes? I can't think what the purpose would be though... --Bmk (talk) 05:03, 11 May 2008 (UTC)[reply]
Is it a place to "park" the plug while in its box? Julia Rossi (talk) 05:24, 11 May 2008 (UTC)[reply]
We thought of all of these possibilities. We should mention that one of our microwaves (not the one in the photo) only has european-style holes in the ventilation panel EVEN THOUGH it is an American model with a US-style plug. This seems to invalidate many of the above possibilities... Saket (talk) 05:31, 11 May 2008 (UTC)[reply]
The one not-yet-mentioned possibility is that this particular model was designed to konfuze WP Reference Desk editors. :) -- Fullstop (talk) 05:35, 11 May 2008 (UTC)[reply]
Aww, my microwave is boring, no plug holes. How about this: a built-in template for the workers to figure out what country the microwave is going to, so they don't get them mixed up in the factory. The workers don't have to be able to read or anything, it's all done with diagrams. For the exception above, that factory is just wiring North America and Europe-style plugs, so they only need one template to check against. Franamax (talk) 06:43, 11 May 2008 (UTC)[reply]
By my count, there are 9 hole patterns in that pic. Thats covers pretty much every type, not just the North Am + European ones. -- Fullstop (talk) 07:01, 11 May 2008 (UTC)[reply]
Yes, that one came from a factory supplying the whole world. The other one mentioned above at 05:31 is from a factory with less scope. Franamax (talk) 07:04, 11 May 2008 (UTC)[reply]
I think it's for packaging purposes; when the microwave was packaged, the plug was perhaps inserted into the relevant holes and excess cable bunched up with a wire tie thing. ----Seans Potato Business 07:51, 11 May 2008 (UTC)[reply]
Packaging maybe not, but assembly more likely. You could feed the terminal end of a pre-assembled cord through a hole in the panel and then store the plug. That way it doesn't dangle around while you move that sub-assembly to the next step. Process wise feeding the cord through is a process (grip, position, feed, pull) that is quite different from fixing the cord and putting the back panel on (grip, position, fix screws). Either a human or a machine would have to change tools, so it would make sense to do it at 2 separate stations. But who knows. Lisa4edit (talk) 09:15, 11 May 2008 (UTC)[reply]
I vote for packaging, I have unpacked items myself that had the plug stored in this way. SpinningSpark 09:47, 11 May 2008 (UTC)[reply]
See reference at 5:31 to a model of a microwave oven with an American plug and only European holes on the back panel. That only makes sense for assembly if the plug was replaced after assembly (say, as part of the importing process), and makes even less sense for packaging. Or did they just screw up with that model? 128.12.187.10 (talk) 11:34, 11 May 2008 (UTC)[reply]
Well it could still make sense for packaging. UK plugs are horrible blocky things with pins sticking out sideways causing problems getting it into the box. You need somewhere for the pins to go. US plugs on the other hand are slimmer and with the pins pointing along the cable axis. Coiling the cable and tieing may be sufficient. Also (not sure if this point applies), are not some US power cords detachable like IEC power cords are? If it is not captive then it is not such a problem packing, it is put in the box after the main item. SpinningSpark 12:14, 11 May 2008 (UTC)[reply]
No reason you can't have an IEC style UK plug. Computers, rice cookers, kettles in the UK and/or Malaysia use them Nil Einne (talk) 16:58, 11 May 2008 (UTC)[reply]
Yes, but that was not the point. The question is, does the US microwave mentioned above have one? I suspect the answer is probably no as it is uncommon to find detachable cords on power devices as it is cheaper for the manufacturer to make the cord captive (kettles are a special case because of the need to pour). SpinningSpark 17:40, 11 May 2008 (UTC)[reply]
No it doesn't.128.12.190.172 (talk) 22:12, 11 May 2008 (UTC)[reply]
But my point was, if the US plug was detachable, then surely the UK plug would be as well? Why make an IEC style detachable plug and then make the UK one fixed and the US one detachable? And if the US/UK one was detachable, then the issue of it not being a problem packing because it's put into the box after the main item equally applies. So it just seems to me it's irrelevant whether or not it was detachable as to an explaination for why it owuld have holes for a UK plug but not a US plug Nil Einne (talk) 15:50, 12 May 2008 (UTC)[reply]
If the plug is there you can pretty much rest assured it is not plugged into the wall. 71.100.14.205 (talk) 15:38, 17 May 2008 (UTC) [reply]
It's not only microwave ovens that have these holes. I have a stereo amplifier with the same holes. I've always considered them as a place to stow the mains plug when carrying the unboxed appliance around. Not something that happens very often I admit, but putting the plug safely in these holes stops it unravelling and tripping you up whilst carrying it. Quite a simple explanation - I'm surprised noone else has suggested this. I could be wrong of course. smiler (talk) 14:30, 30 May 2008 (UTC)[reply]

Concentrate Juice vs Non-Concentrate

<moved from miscellaneous desk>
How much of a difference is there between fruit juices that are "made from concentrate" and those that are "not made from concentrate"? Suppose if Juice A is made from concentrate with just filtered water and juice concentrate as its ingredient and Juice B is not made from concentrate (like Tropicana), how much of a nutrient difference is there? Is the juice not made from concentrate "healthier" for you? Acceptable (talk) 21:31, 10 May 2008 (UTC)[reply]

Well the effective food miles of from concentrate is way lower so you really shouldn't touch the "not from concentrate" stuff. I have tried blind taste off from concentrate with bits versus not from concentrate and they are indistinguishable but real freshly squeezed of course is quite different. --BozMo talk 08:30, 11 May 2008 (UTC)[reply]
I guess you've seen our articles food miles, concentrate and juice? In the last, sugar's a health issue. Julia Rossi (talk) 10:10, 11 May 2008 (UTC)[reply]
I don't understand why "from concentrate" juice should have fewer food miles - could you explain Boz? -Bmk (talk) 15:34, 11 May 2008 (UTC)[reply]
While the concentrate may have travelled just as far, the water has hardly gone anywhere. --Tango (talk) 16:20, 11 May 2008 (UTC)[reply]
Right so the "litre-kilometres travelled" per litre drunk is less. --BozMo talk 20:33, 11 May 2008 (UTC)[reply]

BozMo's point that fresh-squeezed juice tastes different is due, I believe, to the pasteurization process, which heats the juice to the point where some chemicals are altered, resulting in a flavor change. In my experience, juices that have been flash pasteurized more closely approximate the flavor of fresh-squeezed juices.

Assuming that both the concentrate and not-from-concentrate juices have been equally pasteurized, I would say there are minor differences, but they would be indistinguishable. When boiling away the water in a vaccuum at low temperature, other volatiles in addition to water can get boiled away too. This may result in a slightly different chemistry between the concentrate and original, which may result in a detectable difference in aroma if not flavor. =Axlq 06:07, 12 May 2008 (UTC)[reply]

Platinum

<moved from miscellaneous desk>
I have a piece of what apears to be platinum that was melted down in a crucible and was wondering how do I go about finding out what it is?--66.191.106.19 (talk) 08:31, 11 May 2008 (UTC) Probably just as simple as taking it to a jeweler, if that doesn't work, ask said jeweler for any contacts of his. —Preceding unsigned comment added by 121.222.146.142 (talk) 10:32, 11 May 2008 (UTC)[reply]

Well...there is a test, which is that if it isn't attacked by either concentrated nitric acid or hydrochloric acid, but it does react with the combination of the two (aqua regia), then it's probably platinum. But I would take it to a jeweler - if you do it yourself you run the risk of damaging your sample or yourself. --Bmk (talk) 18:18, 11 May 2008 (UTC)[reply]
One way is determine the density. If it looks pure with no bubbles or inclusions, weigh it, and find the volume by water displacement. Platinum is one of the densest metals, so it should be heavy, but there are a few other dense silvery metals too, such as osmium or iridium. A metalurgist may have equipment such as xrays that could determine what it is too. Graeme Bartlett (talk) 21:29, 11 May 2008 (UTC)[reply]
Density may not work particularly well if the sample isn't pure. If it's alloyed with something else, the density could be significantly different from the density you would find in a textbook for pure platinum.. --Tango (talk) 00:17, 12 May 2008 (UTC)[reply]
Oh, and platium is worth a lot (US$2000/once) so get it to a jeweler.118.90.102.125 (talk) 12:33, 13 May 2008 (UTC)[reply]

crystals

where and when were crystals discovered —Preceding unsigned comment added by 76.29.128.29 (talk) 13:34, 11 May 2008 (UTC)[reply]

I imagine they've been known about since pre-historic times. --Tango (talk) 16:17, 11 May 2008 (UTC)[reply]
According to our article on crystals, they were indeed known to (if not understood by) the ancient Greeks, and probably most other ancient civilizations as well. Nicolas Steno, a Danish geologist, is sometimes credited with beginning the modern science of crystallography and minerology when he observed in 1669 that any sample of a single crystalline substance has the same angles between faces. --Bmk (talk) 18:39, 11 May 2008 (UTC)[reply]

Nutrient needs of brain

Are there any nutrients which have been shown to be essential or particularly beneficial to the brain, more than to the rest of the body? Also, how many calories(assuming average calorie intake, which I think is 2000 daily) does the brain alone require every day? Hence, what would be the daily nutritional needs of the proverbial "brain in a vat" which maintains mental functions without a body? —Preceding unsigned comment added by 69.224.182.55 (talk) 14:58, 11 May 2008 (UTC)[reply]

You average calorie intake is unlikely to have much effect on the energy requirements of your brain. (Your average calorie intake doesn't have much effect on the energy requirements of your body either although the heavier you get, the more energy your body needs to move you around) Nil Einne (talk) 16:53, 11 May 2008 (UTC)[reply]
I think the question was what percentage of the daily calorific intake is used by the brain, not how does calorific intake affect what the brain uses. --Tango (talk) 19:01, 11 May 2008 (UTC)[reply]
The brain has an almost absolute requirement for glucose. It does not metabolize fatty acids. However, it will use ketone bodies for energy if glucose is not in abundance. Wisdom89 (T / C) 17:36, 11 May 2008 (UTC)[reply]
The absolute fuel requirement for an adult brain is about 4-6 mg/kg/minute of glucose, where kg is the person's body wt, not brain wt. Glucose provides about 4 cal per gram of glucose. —Preceding unsigned comment added by 159.14.240.230 (talk) 21:08, 14 May 2008 (UTC)[reply]

ionizing radiation

What is the simplest definition of ionizing radiation? Mr Beans Backside (talk) 17:30, 11 May 2008 (UTC)[reply]

Electromagnetic radiation (high energy EM waves/particles) or radioactive particles (beta particles, neutrinos, alpha particles etc..etc..) that strip electrons and produces ions of neutral elements. Wisdom89 (T / C) 17:33, 11 May 2008 (UTC)[reply]

"Any radiation capable of displacing electrons from atoms or molecules, thereby producing ions" - from http://dels.nas.edu/potassium_iodide/glossary.html Regards, CycloneNimrodTalk? 17:45, 11 May 2008 (UTC)[reply]

Dark matter

Why doesn't dark matter reflect light? I was wondering because of the recent story reporting the discovery of a dark planet. I seemed to have forgotten what it was exactly that was discovered, and can't find the story now. Maybe I dreamed it! But I thought it said something about gravitational lensing. Any help, or am I losing it? Mr Beans Backside (talk) 17:31, 11 May 2008 (UTC)[reply]

You're not losing it. Dark matter (see that article, it should answer your questions) either does not emit electromagnetic radiation or doesn't emit enough of it to be detected directly. Instead, we detect it by it's gravitational effects on matter. Not entirely sure why it doesn't reflect light, though.. Regards, CycloneNimrodTalk? 17:43, 11 May 2008 (UTC)[reply]
Light is an electromagnetic wave - only charged bodies interact with it. So (presumably uncharged, certainly the upper limits on its charge are very small) dark matter particles don't interact with it. The electrons in your body are what reflect light - a big brick o' neutrons would reflect very little light indeed (only through virtual particles. WilyD 17:46, 11 May 2008 (UTC)[reply]
How do we know the upper limits on the charge are small? Wouldn't our observations just tell us the upper limits on the net charge? Objects with a zero net charge can still reflect light by being made up of equal numbers of positive and negative charges (eg. atoms). --Tango (talk) 18:59, 11 May 2008 (UTC)[reply]
That's correct. But we can put small upper limits on the charge of each particle. Your body is neutral because it contains equal numbers of electrons and protons, but the negative electrons still reflect light. So DM can't be made of negative and positive particles lumped together like 'normal' (Baryonic) matter is, or we'd see it. Olaf Davis | Talk 20:41, 11 May 2008 (UTC)[reply]
If the charge is not small, then dark matter particles would be created quite often in particle accelerator experiments, where it would show up as missing mass and momentum. Null results from such experiments limit the charge of any unknown particle (this depends on the mass of the unknown particle). This method does not depend on whether or not the particles actually exist.
There are other ways to contrain the charge of dark matter particles. E.g. one can exploit the fact that the millicharged DM background would become polarized in an electric field. You can try to measure the electric field inside a cavity of a conductor when there is a strong elecric field outside, see here. Other possible experiments are discussed here and here. Count Iblis (talk) 21:02, 11 May 2008 (UTC)[reply]
(ec)First of all, no one has proved dark matter exists. Astrophysicists would like it to exist because if it does not there are severe theoretical difficulties with our theories of the universe. There are many ideas of what dark matter might be and why we cannot see it, as Cyclonenim said, look at the article for the various possibilities. I think the claimed discovery you are referring to is a galaxy rather than a planet, also mentioned in the article. SpinningSpark 17:50, 11 May 2008 (UTC)[reply]
I concur with Spinningspark - I'm not sure about the "dark planet" thing, but I guarantee it wasn't a planet made of dark matter. At this point dark matter is just a hypothesized (but as of yet not directly observed) substance introduced to explain gravitational anomalies in the universe. Some physicists have proposed a model of what dark matter might be like if we ever detect it - it's known as a WIMP. Basically, WIMPs are particles that interact only via the gravitational force and weak nuclear force, like neutrinos. So if dark matter exists and is made up of something like WIMPs, that would mean that they would not emit or absorbe any electromagnetic radiation (light), and would be literally "dark". But again, it's all highly hypothetical at this point. --Bmk (talk) 18:45, 11 May 2008 (UTC)[reply]
These are both realistically misrepresentations. Science never proves anything exists - The level of confidence among astronomers that is such that working with ~100 astronomers, I don't know any who take seriously an alternative explanation. But there's no evidence of "dark matter planets" - "dark planet" would presumably mean a planet that's just too dim - a planet discovered through radial velocities or gravitational lensing probably won't be visible, nor would a free-floating planet (you might call it a planemo if you want to avoid the ire of certain astronomers, who'll remain nameless). Dark matter might form compact objects, but it seems unlikely - all the observations we have of it suggest its not very collisional - see bullet cluster, for instance, and maybe read MACHOs. WilyD 18:56, 11 May 2008 (UTC)[reply]
According to the article 'bullet cluster', stars arent very collisional either-"The major components of the cluster pair, stars, gas and the putative dark matter, behave differently during collision, allowing them to be studied separately. The stars of the galaxies, observable in visible light, were not greatly affected by the collision, and most passed right through, gravitationally slowed but not otherwise altered". Em3ryguy (talk) 21:07, 11 May 2008 (UTC)[reply]
That's right. There are actually models (admittedly bit far out) that predict DM analogous to ordinary matter. So-called [mirror matter] is an example and this article actually suggests that some close in extrasolar planets might actually be made of this invisible mirror matter :). See also here. Count Iblis (talk) 21:16, 11 May 2008 (UTC)[reply]

math and physical difference in mental ability

is there any conclusive data that asain people are better at math because of a physical difference in there brain or learning ability that gives them an advantage over other races? —Preceding unsigned comment added by 206.75.0.220 (talk) 17:54, 11 May 2008 (UTC)[reply]

Not all Asian individuals are as intelligent as you imply. However, when they are, it tends to be as a result of how they were schooled rather than a physical difference. Regards, CycloneNimrodTalk? 18:26, 11 May 2008 (UTC)[reply]
If you are pondering why a lot of good engineering is presented by Asian tigerstates, Japan and Asia in general, the answer should not lie quite in the brain. The dominant religion, for instance, is one which stresses subordination to mentors and the importance of learning skills; to strive for a common goal. Now, that's merely the gist of things. Greater political factors have a lot to do with how industry in Asia has gone well after the second world war. Confucianism, Japan and Four Asian Tigers should give you a quick introduction, while better, Asian-wide topics can be reached from these. Merely google something like "industry in asia", or "asian economy". Does this help you? I hope my interpretation was correct, or at least not completely wrong. Scaller (talk) 18:34, 11 May 2008 (UTC)[reply]
I may be heading down a slippery slope here, but see Race and intelligence. SamuelRiv (talk) 19:57, 12 May 2008 (UTC)[reply]

Concentrated Blackcurrant Juice

What would be the effects of "hypothetical person A" drinking too much concentrated Blackcurrant Juice? Would it be enough to kill them? ps this is not medical its science in hypothetical terms. Mr Beans Backside (talk) 18:07, 11 May 2008 (UTC)[reply]

It is hypothethically possible to drink enough of virtually any liquid to kill yourself, but the amounts are really quite excessive. See: water intoxication. Dragons flight (talk) 18:41, 11 May 2008 (UTC)[reply]
Do you mean without diluting it? I'm not sure it's strong enough to harm you, but if it is, it could dehydrate you and cause damage to the liver and kidneys, I expect. I'm just guessing, really, though. I expect someone here knows better than I do. --Tango (talk) 18:54, 11 May 2008 (UTC)[reply]

Why was I born to my parents in my town at a specific time?

moved to HumanitiesDesk

Taste of apple seeds

I've read that apple seeds contain cyanohydrins (see hydrogen cyanide) and that cyanide has a "faint bitter almond-like odor". I've also noticed that apple seeds (at least for Cripps Pink and Golden Delicious and some other varieties I've tried) taste quite strongly of almond. Is this a coincidence, or am I indeed tasting the hydrogen cyanide present in the apple seeds? Eric. 81.157.250.169 (talk) 20:15, 11 May 2008 (UTC)[reply]

Well cyanohydrins are produced by reaction with HCN. Also, the almond taste of such seeds is mostly subjective. For instance, I personally don't taste it. Cyanohydrins are derived from ketones, for instance, which can have a fruity/distinctive smell. This could possibly explain the taste. Wisdom89 (T / C) 20:48, 11 May 2008 (UTC)[reply]
I thought that it was really bad for you to eat apple seeds? Never really thought to question it but I was told as a child that they were toxic and were always to be discarded... --Kurt Shaped Box (talk) 21:37, 11 May 2008 (UTC)[reply]
See Archive May 1 "Cyanide and almonds" same goes for apple seeds Lisa4edit (talk) 00:13, 12 May 2008 (UTC)[reply]
Ah, interesting. So the taste is due to benzaldehyde? I see. Eric 81.157.250.169 (talk) 12:52, 12 May 2008 (UTC)[reply]
I've had around 20 or so apple seeds in one sitting before (chewed, not swallowed whole), so either they're almost harmless or I'm working on being the next Rasputin. Eric 81.157.250.169 (talk) 12:52, 12 May 2008 (UTC)[reply]
Many plants and seeds contain glycosides in varying low does concentrations. Among them such healthy foods as spinach. One of those glycosides is amygdalin which is contained in the pits of stone-fruit and apple seeds. If you add enzymes to amygdalin HCN is split off. HCN is toxic in very low concentrations to plants and animals. Plants contain both the amygdalin and the enzyme, but they are stored separately from each other. When you chew the seed, the enzyme gets mixed with the amygdalin and reacts. Some amygdalin passes through to your digestive tract and reacts with enzymes there. HCN is also further broken down by enzymes into HCNO which is not toxic. But the body only has a limited capacity for that to happen. Some bacteria also process the HCN into HCNO. Individual apple seed have less bulk than bitter almond. I've dug around for concentrations and found one source [12] that states 690-790 ppm (mg/kg). Based on this study another source [13] created this scenario The ADI (acceptable daily intake) described for cyanide has been reported at 0.05 mg/kg. If all amygdalin would be converted into cyanide this would corresponds to an ADI for amygdalin of 0.85 mg/kg. For a person of 75 kg this equals an acceptable intake of amygdalin of 63 mg daily. Assuming that all amygdalin from apple seeds is extracted from the seeds (which will be an overestimation), this equals the consumption of 80 g of apple seeds. Before you start upping your daily consumption to that amount you should consider that the ADI includes intake from all sources. (lima beans, bamboo shoots, juice, cider, and even corn to name just a few). It also presumes that you are in perfect health otherwise and weigh 75 kgs. Lisa4edit (talk) 19:52, 12 May 2008 (UTC)[reply]
Thanks for the thorough research & information -- I considered asking for specific concentration information but thought that was a bit much to hope for. Fortunately 80 g of apple seeds is well beyond my appetite for them. I found your first reference helpful for putting such information in context. Eric 81.157.250.169 (talk) 04:15, 13 May 2008 (UTC)[reply]

Aluminum chloride

Is aluminum chloride the same as aluminum zirconium tetrachlorohydrez glycine? Mr Beans Backside (talk) 20:41, 11 May 2008 (UTC)[reply]

Take a look at aluminium chloride and zirconium + glycine and tell us Nil Einne (talk) 20:50, 11 May 2008 (UTC)[reply]

Hardness of water

Is the hardness of water a consequence of pipe scaling? What technology and major steps are involved in the purification of drinking water and the treatment of waste water? Mr Beans Backside (talk) 20:43, 11 May 2008 (UTC)[reply]

To the first one, er no... As Hard water and Fouling may tell you, it's more likely to be the reverse. For the second one, is there anything in particular that Water treatment doesn't tell you? Nil Einne (talk) 20:56, 11 May 2008 (UTC)[reply]

Pressure and altitude

Could someone explain why pressure decreases but temperature varies as altitude increases? Mr Beans Backside (talk) 20:44, 11 May 2008 (UTC)[reply]

Erm, well as a general rule, temperature decreases with altitude so I'm not entirely sure where you got this impression from. Regards, CycloneNimrodTalk? 21:02, 11 May 2008 (UTC)[reply]
Once you get high enough, that's not the case any more. The Thermosphere can get extremely hot, for example. However, since it's so thin, the actual amount of thermal energy is very low, so it wouldn't feel warm. --Tango (talk) 21:53, 11 May 2008 (UTC)[reply]
You should look at atmospheric pressure for pressure (obviously) and lapse rate for temperature. SpinningSpark 21:21, 11 May 2008 (UTC)[reply]
Different sources of heat become significant at different altitudes, which is what causes the variation in temperature, rather than just a gradual cooling as you might expect. Earth's atmosphere will tell you more. --Tango (talk) 21:53, 11 May 2008 (UTC)[reply]

Energy sources

I know there's electric, solar, wind, hydroelectric, geothermal, coal, oil, natural gas, and nuclear. What other energy source are there, or are those them all? Mr Beans Backside (talk) 20:48, 11 May 2008 (UTC)[reply]

Biomass? Fribbler (talk) 20:57, 11 May 2008 (UTC)[reply]
No, that is not all. There is tide and wave power for a start. There are many more, take a look at List of energy topicsSpinningSpark 21:00, 11 May 2008 (UTC)[reply]
(EC) There are lots and lots of potential energy sources. Energy development has most/all of the ones currently of most interest. List of energy resources has a lot of other things including means of energy storage and energy conservation Nil Einne (talk) 21:01, 11 May 2008 (UTC)[reply]
What about potential energy? User:Yetmotega/1 Yetmotega (talk) 22:00, 11 May 2008 (UTC)[reply]
Potential energy is energy stored in a field. So you need to specify which force of nature we are discussing know the energy type, eg electric, magnetic, gravitational etc. SpinningSpark 22:30, 11 May 2008 (UTC)[reply]
E=mc2 so basically everything is an energy source. GameKeeper (talk) 22:52, 12 May 2008 (UTC)[reply]
The most basic energy sources are nuclear fission/fusion and cosmic motion. Motion of the heavenly bodies causes tidal forces. Natural nuclear fission (decay) of Uranium in the earth's crust cases it to heat up, giving us geothermal energy. Nuclear fusion makes the sun 'burn', which (directly) gives us light, thermal solar energy and photovoltaic energy, but also causes the air and water to heat up, causing wind (thus wind energy) and evaporation, rain and thus hydro-energy. The difference in salinity of this water and the sea water can also be used (blue energy). The oceans heat up unevenly, so the difference in temperature between top and bottom water can be used to produce energy. And of course we can let plants grow on the sunlight, giving us biofuel. Let that rot for a while and you get fossilised fuels (coal, oil, gas). So apart from tidal and geothermal it's all solar energy. And then there is man-made nuclear energy, but why bother when we already have the sun doing that for us, while we have barely sratched the surface of the possibilities that gives? (Sorry, a bit of politics in there. :) )
Note that 'electric', which you mention is an energy form, not an energy source. And while I'm at it, hydrogen is not an energy source, but a way of storing energy.
I'm sure I missed a lot of the resultant energy sources too. And then there are combined technologies like the solar chimney. The possibilities are endless. Too bad we've only recently started lookiong into all these options and are still barely investing in such research compared to the shitloads of money poured into fossil fuels and nuclear fission. DirkvdM (talk) 09:04, 14 May 2008 (UTC)[reply]

Which genes flank TJP1

How do I find out which genes flank TJP1 in humans? I figure one of the many genomic databases would be able to help but they're terribly complicated and intimidating. ----Seans Potato Business 21:04, 11 May 2008 (UTC)[reply]

On the link you provided, scroll down to the section Genomic context. This will tell you the chromosome the gene is present on and it's location. By searching the position before it or after it should give you your answer. Regards, CycloneNimrodTalk? 21:11, 11 May 2008 (UTC)[reply]
Its actually a bit complicated this one. However, 5' of TJP1 appears to be necdin-like 2, while 3' is CHRNA7-FAM7A fusion isoform 1 (which is a very odd locus). There may be other genes in between, but they have not been characterized. Rockpocket 21:53, 11 May 2008 (UTC)[reply]
How did you get those? ----Seans Potato Business 21:59, 11 May 2008 (UTC)[reply]
Okay, this isn't working. I'd like to know in which order the genes TJP1, NDNL2, GREM1, SLC12A6 and SPRED1 occur. I know they're in the same region, but I want to determine if there are any genes between them and in what order the occur. If I search for each gene I get:

ndnl2 15q13.1

slc12a6 15q13-15

tjp1 15q13

grem1 15q13-15

spred1 15q14

Yet none of these shows up on the 'genomic context' diagram of any of the others. How can I interpret this mess? Does SPRED1 exist on an exon inside both GREM1 and SLC12A6 which both exist in exons of each other? I suppose one may be on the antisense strand, but there's going to be overlap somewhere here. ----Seans Potato Business 21:55, 11 May 2008 (UTC)[reply]
According to Ensembl (by searching for each gene on the human genome):
  • Ndnl2.. can be found on Chromosome 15 at location 27,347,645 - 27,349,325 and is anti-sense.
  • Tjp1.... can be found on Chromosome 15 at location 27,778,863 - 27,901,998 and is anti-sense.
  • Grem1.. can be found on Chromosome 15 at location 30,797,497 - 30,814,158 and is sense.
  • Slc12a6 can be found on Chromosome 15 at location 32,309,489 - 32,417,557 and is anti-sense.
  • Spred1. can be found on Chromosome 15 at location 36,331,808 - 36,433,526 and is sense.

Are there intervening genes? Yes in all cases, except between Ndn12 and Tjp1. Rockpocket 22:35, 11 May 2008 (UTC)[reply]

Okay, this is really helpful of you. Thanks. However, now I have a conflict; SPRED1, according to this is at 15q14 while GREM1 according to this is at 15q13-15. If 15q13-15 means 15q13 to 15q15, then why do those numbers you (kindly) provided not overlap? And how did you find out whether they were sense or anti-sense? --Seans Potato Business 22:48, 11 May 2008 (UTC)[reply]
The "q" refers to chromosome regions while the numbers (13-15) refer to cytogenetic bands identified by staining. That nomenclature is an old skool way of positioning genes on chromosomes and is not particularly accurate. Now we have a fully sequenced genome we can precisely identify exact positions of genes. In this case 15q13-15 means that Grem1 resides somewhere within bands 13-15, rather than meaning it spans that huge length. I used Ensembl to find the info, and identified the strand from that. For example, have a look at the diagram on the TJP1 page, under the "transcripts" sub-section. Find the thick blue line that says "DNA (contigs)". Now any gene identified above that line is on the forward strand (sense orientation) and any genes under that line is on the reverse strand (anti-sense). TJP1 is directly underneath the line, therefore it is anti-sense.
If you don't wish to use Ensembl, the NCBI site you were using gives the same info. On the Spred1 page, on the right hand side of the "genomic context" section there is a link to "See SPRED1 in MapViewer" If you follow that link [14], you can see that the Spred1 gene is located on the right hand side vertical line spanning roughly the same distance I provided (36331k to 36433k). If you want exact values, you can always zoom in by clicking on the gene. Note also the little black arrow beside the Spred1 gene name. The arrow points downwards, indicating the gene is on the sense strand. If it was to point upwards, you would know your gene of interest was anti-sense. Rockpocket 01:22, 12 May 2008 (UTC)[reply]

How does something vibrate in 10 dimensions?

An article I read awhile back about an atom's parts said that superstrings are the smallest part of the atom. It went on to say it vibrates in 10 dimensions.

Huh? How do they determine this? If you cant actually see it with a microscope, how can you determine with fancy math that something can exist in 6 dimensions we cant even imagine?--Sam Science (talk) 21:40, 11 May 2008 (UTC)[reply]

There is very little experimental evidence for superstring theory at this point, as far as I know. It's mostly maths and mathematicians can imagine all sorts of things that don't fit in our everyday understanding of the world. If you can work out the maths of something vibrating in 3D, you can work it out for something vibrating in 10D, the maths is pretty much the same, just with different numbers. It turns out that if you do things in just the right way (that's the hard part) those vibrating strings happen to interact with each other in the same way we observe elementary particles interacting. --Tango (talk) 21:47, 11 May 2008 (UTC)[reply]
I think you're a tad confused with what you read in that article. String theory is theoretical, as the name suggests and it is hoped it'll fill in a lot of gaps in physics, but it certainly isn't proven. Also the theories state that the strings vibrate in one dimension, not ten. You're quite right in thinking we can't see them with microscopes, we can't even see atoms themselves properly. As I said, it is not a proven theory, simply something that is being made up to fix gaps in the theories of physics. Regards, CycloneNimrodTalk? 21:50, 11 May 2008 (UTC)[reply]
"Proven theory" is a contradiction in terms. Theories can't be proven, you can just gather evidence for them until it becomes very unlikely that they're wrong. Proof is a mathematical concept, you prove theorems, not theories. And what do you mean by them vibrating in one dimension? They are one dimensional objects vibrating in 10 dimensional space. --Tango (talk) 21:55, 11 May 2008 (UTC)[reply]
Physicists use mathematics to describe the universe, using vectors and matrixes to describe space. In simple models these are 4-element matrixes - 3 space-like and 1 time-like (see Minkowski space). It may be surprising, but this often makes the mathematics a lot simpler - for example, electricity and magnetism join messily (using several equations) using 3 space dimensions and keeping time separate (using maxwells equations), but join simply (using one equation) when using the 4 element vectors (using the electromagnetic tensor). Maxwell's equations and the electromagnetic tensor are equivalent - the only difference is how they're expressed.
Theoretical physicists find that their theories work when these vectors and matrixes have more than 4-elements - generally the theories use either 10 or 11 elements in their vectors and matrixes, and this corresponds to 10 or 11 dimensions. The reason why a new theory is needed is to express the four fundamental forces and lots of fundamental constants more simply. This is cutting-edge physics - a lot of the work on these theories is currently working out how to detect how this is different from the standard model. --h2g2bob (talk) 22:43, 11 May 2008 (UTC)[reply]
The various string theories only work at all in a particular number of dimensions (10, 11, 12 or 26 depending on which variant you're talking about). I don't know what goes wrong otherwise, but it might be a violation of unitarity (meaning, basically, that the theory predicts events happening with a probability greater than one). So if you want to use string theory as a physical theory, you're committed to having those extra dimensions. There's no other reason to think they exist. -- BenRG (talk) 01:17, 12 May 2008 (UTC)[reply]
One of the things that fails is that photons are not massless. More generally, there is a mapping problem. If you assume that all particles are associated with vibrational string modes and that there are X independent classical properties of particles (e.g. charge, spin, color, etc.) then you require at least X extra dimensions to map those properties onto geometric structures. (You can get away with less than X if you assume that they aren't ultimately independent.) Dragons flight (talk) 05:43, 12 May 2008 (UTC)[reply]

Dogs in heat

How many times should a female dog mate while shes in heat to ensure she becomes pregnant? And at what point during her being cycle should she mate for the most success? Mr Beans Backside (talk) 22:07, 11 May 2008 (UTC)[reply]

Chemists: solvents for spot cleaning carpet?

I have some dried emulsion paint (might be called latex paint in the US?) I want to remove from a carpet.

I do not know what the carpet is made of, but I do not think it is wool. Many capets in the UK seem to be made of polypropylene or perhaps nylon.

My questions are - 1) as an alternative to the below, would white spirit, turpentine substitute, or even WD-40 be just as good please? And 2) would Nail Polish Remover dissolve the carpet? I.E. It dissolves the paint, it does not dissolve the carpet!

Rug Doctor "Traffic Lane Cleaner" (carpet pre-washing spray): active ingrediant Ethylene Glycol Monobutyl Ether - has worked well removing dried chewing gum and unknown stains from a carpet. Have not tried it on paint yet.

Nail Polish Remover: Acetone, Glycerine, Cetylacetate, Oleyl Acetate, Acetylated Lanolin Alcohol, Polysorbate 80. Highly recommended for carpet paint removal here http://www.thriftyfun.com/tf991362.tip.html

Stardrops cleaner: Sodium dodecylbenzene sulphonate, Alkyl ether sulphate, Alcohols C9-11, ethoxylated Bitrex (Denatonium benzoate) solution, Bronopol (2-bromo-2-nitropropane-1,3-diol).

Cheaper and to-hand alternatives: white spirit, WD-40.

Thanks. 80.2.195.209 (talk) 22:54, 11 May 2008 (UTC)[reply]

Whatever you do, do a "spot-test" first on an inconspicuous area of the carpet (e.g. part that's always covered). --Seans Potato Business 22:58, 11 May 2008 (UTC)[reply]
I would guess that white spirit will work (it will almost certainly remove the paint, I'm just guessing it won't ruin your carpet). As Seans says, you should definitely test it somewhere out of the way first. If it were me, I would just have a go with white spirit somewhere hidden and see what happens, but you may want to wait for more informed answers (I've never been one to follow instructions!). --Tango (talk) 23:01, 11 May 2008 (UTC)[reply]
Not based on any chemical analysis, but merely OR. Undiluted "Pine Sol" and elbow grease along with a pile of paper towels worked well. A generic "compare to" product didn't. Since you aren't in the US I don't know if you can get it and whether you get the same stuff under that label. We're out and I threw out the old bottle, so I can't tell you the composition. Trying in a hidden area is always a good idea. Lisa4edit (talk) 23:46, 11 May 2008 (UTC)[reply]

Emission line

Where can learn more about the spectral lines of substances? In particular, there is apparently an argon emission line at 750.38 nm and I'd like to know if it belongs to the argon atom or an ion and what the cross section for its activation is as a function of incident electron energy. I imagine there are reference works for this kind of data and would be glad for directions. Thanks. —Bromskloss (talk) 23:09, 11 May 2008 (UTC)[reply]

Atomic spectral line has an explanation and a link at the bottom that should help answer your question. --Lisa4edit (talk) 23:33, 11 May 2008 (UTC)[reply]
Thanks, but I'm not looking for an explanation of how it works. What I need is a data collection where I could look up argon, see a list of all its lines and preferably the corresponding cross section for electron-argon collisions that would result in emission in this line. —Bromskloss (talk) 23:46, 11 May 2008 (UTC)[reply]
I recommend the CRC Handbook of Physics and Chemistry at http://www.hbcpnetbase.com/. It's quite comprehensive. The data you're looking for will probably be found in Section 10: Atomic, Molecular, and Optical Physics. --Bmk (talk) 02:08, 12 May 2008 (UTC)[reply]
Thanks for the tip. That one could be useful for the future. However, I look at the table Line Spectra of the Elements, but there is no mention of an argon line at 750.38 nm. :-( (And no cross secions, as far as I can see.) —Bromskloss (talk) 08:54, 12 May 2008 (UTC)[reply]
No, the Argon I line at 750.3 nm is listed there!! I must defend the honor of the CRC handbook!! :) The lines are listed in Angstroms, rather than nm, so you can find it at 7503.8 A. But no, I'm not sure where you can find the crossection. --Bmk (talk) 14:05, 12 May 2008 (UTC)[reply]
Ah, I didn't see that! It's a good thing for me you came to defend it. :-) —Bromskloss (talk) 09:11, 13 May 2008 (UTC)[reply]
Lol - after your comment I spent a few horrified minutes looking for 750.38 before I realized I was also using the wrong units :) --Bmk (talk) 01:51, 15 May 2008 (UTC)[reply]
A quick google search reveals that it is from ArI - ie non ionized. It is one of the main emmission lines. But this is not a systematic method! Graeme Bartlett (talk) 03:56, 12 May 2008 (UTC)[reply]


May 12

Guinea Pig head bump

I have two guinea pigs, and when i gently place my hand on either's noses, They quickly thrust their heads up. Why is this? Gbgg89 (talk) 04:29, 12 May 2008 (UTC)[reply]

Maybe it is a response to a nearby source of food, so perhaps to bite your finger? Astronaut (talk) 09:29, 12 May 2008 (UTC)[reply]
Guinea pigs are herbivores, so that's unlikely. Given that noses are very sensitive organs in most species, my money is on it being a protective reaction. Since guinea pigs are also social animals, a submissiveness reaction may also be possible. 71.236.23.111 (talk) 16:33, 14 May 2008 (UTC)[reply]

Maps and Time lines of plants, insects, sm. animals, and terrain.

I am looking to assemble a wide verity of information grouped together covering a certain area of land.

Along the border of China and Mongolia. 100 miles south of border to north 200 miles inside Mongolia From western border to eastern border

All from the time of 125 million to 300 million years ago.

Maps and graphs containing:

-Topographical : - Terrain

                - Water levels highs and lows
                - Some research sight (containing programs as to what the place looked like
                  back then

-Time lines of : - insects (paleontology)

                - sm. animals
                - fossils
                - charts graphs

WHY:

First Flower PBS Airdate: April 17, 2007 Professor Sun Ge, from China's Jilin University, is certain that early flowers evolved here, in northern China, and he is determined to find the world's first.

Not far from the border of Inner Mongolia, there is a remarkable fossil site that is revealing what the Earth looked like more than 100,000,000 years ago. —Preceding unsigned comment added by Coffee1030 (talkcontribs) 05:15, 12 May 2008 (UTC)[reply]

ARUN SINGH BAGH

WHAT IS THE NAME OF THE WOODEN BLOCK ON WHICH THE WINNERS IN OLYMPICS STAND WHEN THEY GET FIRST SECOND AND THIRD POSITION —Preceding unsigned comment added by 195.189.142.54 (talk) 06:03, 12 May 2008 (UTC)[reply]

It is called a medal podium. Rockpocket 06:12, 12 May 2008 (UTC)[reply]

telecommunication-microwave survey as used in network establishment

elecommunication-microwave survey as used in network establishment — Preceding unsigned comment added by 61.246.1.108 (talk) 07:30, 12 May 2008

Do you have a question? Astronaut (talk) 09:07, 12 May 2008 (UTC)[reply]
It could be a site survey. Graeme Bartlett (talk) 04:55, 13 May 2008 (UTC)[reply]

Insanity

I don't know a lot about psychology (I got as far as AP psych in high school, so I know the basics), but I've recently gotten curious about how people go crazy. Since I don't have much background in the area, I don't where to go to learn about it. Any ideas? Also, as a subquestion, does insanity universally degrade a person's ability to succeed, or are there cases of people who become smarter, more creative, what-have-you after losing it than they were before? Black Carrot (talk) 07:35, 12 May 2008 (UTC)[reply]

The answer will depend on which specific conditions you group under the umbrella term of insanity - note that "insanity" is no longer considered a medical diagnosis (although it is still a rather loosely defined legal term in the UK and the USA). A fairly generic answer to your various questions might be as follows:
  1. Many psychiatric conditions may be diagnosed in childhood, in adolesence, or appear in later life, possibly due to an event that triggers or exacerbates a previously unnoticed condition.
  2. There are several autobographical and semi-autobiographical accounts of the onset of a psychiatric condition - see Girl, Interrupted, I Never Promised You a Rose Garden, The Bell Jar.
  3. Most psychiatric conditions, as long as they are properly diagnosed and managed, do not prevent a person living a "normal" and indeed successful life. There are several well-known examples of people with a psychiatric condition that appears to be intimately linked to their success in a particular field - see, for example, John Forbes Nash (schizophrenia); Vincent van Gogh (possibly schizophrenia or bipolar disorder); Stephen Fry (bipolar disorder); Richard Borcherds (Asperger syndrome) ... Gandalf61 (talk) 10:25, 12 May 2008 (UTC)[reply]
Try reading up on: biopsychiatry and causes of mental disorders. Re Gandalf61, see creativity and mental illness. --Mark PEA (talk) 20:00, 12 May 2008 (UTC)[reply]

Entropy and Creationism

A common argument for Creationism is that as entropy can only increase there must be a start where everything is in order, which is when God created the universe. What is a rebuttal for this argument? How come entropy was so low after the Big Bang? --antilivedT | C | G 07:38, 12 May 2008 (UTC)[reply]

See Objections_to_evolution#Evolution_violates_the_second_law_of_thermodynamics, Entropy and life and Five Major Misconceptions about Evolution at TalkOrigins. Guettarda (talk) 07:53, 12 May 2008 (UTC)[reply]

OK that makes sense but how come the universe didn't go into heat death as soon as it was created? What had caused the disequilibrium (if that's a word)? --antilivedT | C | G 08:10, 12 May 2008 (UTC)[reply]
"Why was the entropy of the early universe so low ?" is a good question, and one of the big puzzles of cosmology. Rogare Penrose addresses the question at some length in The Road to Reality. There are several candidate explanations:
  1. The universe has a very, very long pre-Big Bang history, and what we think of and observe as the Big Bang was a random fluctuation in which all the matter of the universe happened to arrange itself into an especially low entropy state (remember that processes that reduce entropy are not absolutely impossible, they are just very unlikely, and anything that is not impossible can happen if you wait long enough). In this view, our current state of increasing entropy is a relativley short-lived phase in the mega-history of the universe.
  2. A large number of universes are continually being created, possibly by some sort of quantum "budding" process. Those that start in high entropy states are spectactuarly dull places and do not give rise to life. Only a universe that starts in a low entropy state could eventually give rise to beings that ask the question "why was the entropy of the early universe so low ?". This is a version of the strong anthropic principle.
  3. Some mechanism active only in the early universe, and possibly connected with cosmic inflation, drove the universe into a low entropy state. Such a mechanism would break the second law of thermodynamics, so it cannot be active in the universe as we observe it today.
So there are various possible explanations - we just don't yet know which one is correct. Gandalf61 (talk) 09:25, 12 May 2008 (UTC)[reply]
And note that even if you want to say that God had a hand in this, it still requires an understanding of what naturalistic process God would have used to do it. "God did it" doesn't relieve the need for a physical explanation—even if it was done by some sort of intelligent deity there still would be a corresponding physical expression of that action. --98.217.8.46 (talk) 15:02, 12 May 2008 (UTC)[reply]
The first two options don't work at all, and Penrose should realize this. They're the cosmological equivalent of the 747-from-a-junkyard argument against biological evolution. Darwinian evolution is not a 747-from-a-junkyard theory, and if it were then it would be as useless as the ID people think it is. It would be unfalsifiable and predict nothing. Option 3 isn't an option, it's practically a definition of what it means to explain something. Explaining something doesn't mean showing that the world can be that way, it means showing that it couldn't have been some other way. You have to produce a mechanism that makes the world we see more likely than it appears on the surface. You have to rule out something beyond what you ruled out a priori with your anthropic (or aeronautic) principle.
In principle the idea of anthropic reasoning makes sense. What scientists do is reason from the available evidence to a theoretical model that explains it. One piece of evidence available to us is the existence of homo sapiens. If you reason from that evidence to a theoretical conclusion, you've engaged in anthropic reasoning. But in practice no one ever succeeds in doing that; it's too hard. Progress in natural philosophy began when people started to gather evidence about much simpler things, like the motion of a wheel on an inclined plane. As far as I know there has never been a successful anthropic argument in the history of science. All of its claimed successes are co-opted. Fred Hoyle reasoned from the observed abundance of carbon in the universe to a prediction that carbon must have a particular energy level, which was later found. This was claimed as an anthropic prediction. Why? Because human beings contain lots of carbon. It could just as well have been lithium instead of carbon that went into his argument, and then no one would have called it anthropic. Even worse is the current claims of anthropic reasoning in string theory. They want to narrow down the possible flux compactifications in string theory by requiring that they produce a universe consistent with the one we see. This is known as "fitting the model to the data", and it's what scientists have always done. The data is this case is that which supports the Standard Model and the existence of a small positive cosmological constant. It seriously scares me that prominent string theorists think they're "reasoning anthropically" when they do this. That, more than anything else, makes me worry that they've completely lost sight of reality. -- BenRG (talk) 16:56, 12 May 2008 (UTC)[reply]
Not going to pretend that I understand what triggered that particular rant, but let's clarify something for the record. Penrose doesn't actually come down in favour of any of the above three explanations. In fact, he sets them up in order to knock them down in favour of his own explanation, which is connected with quantum gravity. Gandalf61 (talk) 21:55, 12 May 2008 (UTC)[reply]
The argument that "some supernatural entity must have started the universe in perfect order" is hardly an argument for big-"C" Creationism, since that entity could have been anything. --Sean 13:08, 15 May 2008 (UTC)[reply]
Oh - this is a classic numbskull creationist argument. You have to be careful with the OP's statement about entropy. It is NOT true to say that "entropy can only increase". You missed the first half of the second law of thermodynamics. The correct way to simply state this is: "Within a closed system entropy can only increase." (To quote entropy: Spontaneous changes, in isolated systems, occur with an increase in entropy.) So if we consider the Earth to be a closed system (which it's not - but we'll quietly pretend that it is) - then evolution can create localised pockets of decreasing entropy at the cost of increasing entropy much more steeply elsewhere on the planet. Thermodynamics allows this. For example, take 20 dice in a cup. You can go and painstakingly take each one from the cup, turn itto show a 6 and line them up into a nice neat row. The entropy of the dice decreased - they are now much less random. But it took energy to make that happen. In using your hands to straighten them up - you consumed a few more calories and you might maybe eat a little more as a result. Just toss 20 nicely ordered dice onto the table without going to a lot of effort - and guess what? Entropy kicks in and they're all disordered again. Entropy only decreased when you put energy into the system from outside. Taking nice well-ordered food energy and turning it into disorderly end products increased the entropy of our world by considerably more than straightening up the dice did. Humans are going around forcing local pockets of entropy to reduce all the time by activities such as collecting up scattered amounts of iron ore and making cars out of them...but in the process, we are consuming energy and creating lots of ugly chaos elsewhere in the process. So when animals evolved, their LOCAL entropy went down - but the entropy of the planet went up by more than enough to compensate for that. Entropy LOCALLY decreases in all sorts of natural situations - the growth of crystals in a liquid solution for example. It's only within a closed system that entropy increases. You can't use a misstatement of a law of physics to prove or disprove anything! 66.137.234.217 (talk) 15:12, 17 May 2008 (UTC)[reply]

Michelson Interferometer

If the mirror is moved away then the number of fringes appearing in the microscope would change or not? If so then it is because the length of path is changing and so fringes are closely spaced. —Preceding unsigned comment added by MAQMAQ (talkcontribs) 08:54, 12 May 2008 (UTC)[reply]

Please do your own homework.
Welcome to the Reference Desk. Your question appears to be a homework question. I apologize if this is a misevaluation, but it is our policy here to not do people's homework for them, but to merely aid them in doing it themselves. Letting someone else do your homework does not help you learn how to solve such problems. But here is a hint to help you: consider what the interferometer is doing, how the fringes are created and what happens if one of the mirrors is moved.
Please attempt to solve the problem yourself first. The Michelson interferometer article might help, or you can search Wikipedia or search the Web.
Astronaut (talk) 09:27, 12 May 2008 (UTC)[reply]

Aloe Vera

I have been told that Aloe Vera Gelly will kill ear mites in dogs. Can anyone confirm this? Mr Beans Backside (talk) 10:12, 12 May 2008 (UTC)[reply]

Please ask your veterinarian. -- Coneslayer (talk) 11:47, 12 May 2008 (UTC)[reply]
Petroleum jelly and mineral oil are sometimes used to kill parasites on animals - mainly through suffocation. However, proper application can be tricky, and severe infestations may require additional medical help. Phoning your veterinarian's office is highly recommended - depending on your particular situation, they may be able to help you over the phone, without necessitating an office visit. -- 128.104.112.147 (talk) 22:27, 13 May 2008 (UTC)[reply]

Plants

When do plants use the cyclic electron pathway instead of the noncyclic pathway? Mr Beans Backside (talk) 10:13, 12 May 2008 (UTC)[reply]

You mean, "why do plants use the non-cyclic pathway?" The cyclic pathway has as its sole useful product ATP, whereas the non-cyclic pathway also produces NADPH (in addition to oxygen), which is necessary for driving the calvin cycle. Indeed, the cyclic pathway is considered more primitive; any photosynthetic organism relying solely on that process would have a fine source of ATP only in the daylight, and would need yet another process for using that energy to obtain or build the various molecules it needs. An organism using the non-cyclic pathway can use the NADPH to drive the conversion of CO2 into sugars, which can be used as stable stores of energy, polymerized into structural molecules, or converted into virtually any other molecule the plant needs. Someguy1221 (talk) 15:24, 12 May 2008 (UTC)[reply]

Global warming

What are some simple steps or creative ideas that people can take at home and work to combat global warming? Mr Beans Backside (talk) 11:50, 12 May 2008 (UTC)[reply]

  • Ditch your SUV for something that uses less than 6 litres of petrol per 100km
  • Plant a tree
  • Turn down the temperature on your geyser or get a blanket to insulate it (in SA we use electrically powered geysers and our electricity is generated from coal (mostly) so this may not apply to you) and don't waste energy
  • Make sure your house is well insulated
  • Vote for politicians who are committed to reducing CO2 levels
  • Recycle
I see there are quite a few wiki articles but linking all of them will make you less inclined to read any of them so I'll wait for someone to link the best one.
Zain Ebrahim (talk) 12:49, 12 May 2008 (UTC)[reply]
For those not familiar with British idiom, a geyser in this context is a water heater, and not an erupting hot spring. TenOfAllTrades(talk) 13:52, 12 May 2008 (UTC)[reply]
Is it some dialect? I'm British (North-West) and did not recognise the term, though I did surmise the correct meaning. --Seans Potato Business 14:14, 12 May 2008 (UTC)[reply]
I'm British, South-East, and didn't recognise it either. Zain mentions SA (South Africa, presumably) - I guess it's a term local to there. --Tango (talk) 14:18, 12 May 2008 (UTC)[reply]
Oh Sorry! I've only ever referred to it as a geyser and assumed it was a universal term. It not only heats the water but it also stores the heated water. Does a water heater do that as well or do you guys store the water elsewhere? Zain Ebrahim (talk) 14:42, 12 May 2008 (UTC)[reply]
Modern systems heat the water as its needed. You switch on the tap and the heater (or "boiler") switches on and hot water comes from the tap. This system is more energy-efficient than trying to store a quantity of hot water. Such storage systems do exist in the UK but are comparatively rare in low-usage applications (average household). ----Seans Potato Business 15:16, 12 May 2008 (UTC)[reply]
Interesting. Thanks. Zain Ebrahim (talk) 16:10, 12 May 2008 (UTC)[reply]
In the United States, the systems that Seans Potato Business described are uncommon. The common heater is the kind with a storage tank, like you described. However, the term "geyser" was completely unfamiliar to me. We just call them "water heaters" (or, rather strangely, "hot water heaters"). -- Coneslayer (talk) 16:32, 12 May 2008 (UTC)[reply]
Interestingly enough, the disambiguation page for "geyser" says that is can be "A water heating system, in British English". Although I have lived in the USA all my life, as a self-described Anglophile I understood geyser as used by Zain, so I'm perplexed by our NW and SE British colleagues indicating non-awareness of the term. Perhaps you need to read more Agatha Christie ;-) The various types of heaters and some of the terminology are discussed in the article water heating. --LarryMac | Talk 17:14, 12 May 2008 (UTC)[reply]
Geyser is also the Dutch word for an old-fashioned water heater (nowadays 'everyone' uses boilers). I wonder if this has anything to do with Afrikaans stemming from Dutch. DirkvdM (talk) 08:34, 14 May 2008 (UTC)[reply]
What I know as geysers used to be fairly common in the UK in my younger days - at least up to the 1970s. They were gas-powered devices where the main gas burner would ignite from a pilot light, usually with a terrifying whoomph, when the water was turned on. There was great skill required in getting the rate of water-flow just right: too fast and it wouldn't get hot enough, too slow and the gas would go out. AndrewWTaylor (talk) 19:58, 12 May 2008 (UTC)[reply]
I know these things as "boilers". In my experience, the hot water is usually stored in the UK. You have a boiler and a hot water tank. Heating the water immeadiately before using it sounds hard, since you need to heat it really quickly (certainly more efficient, though). --Tango (talk) 10:58, 13 May 2008 (UTC)[reply]
That kind is called a demand or on-demand or tankless water heater. --Anon, 22:22 UTC, May 13, 2008.

Eat less meat or go vegetarian. 200.127.59.151 (talk) 15:54, 12 May 2008 (UTC)[reply]

He was talking about lowering global temperature, how does eating less meat contribute? Mac Davis (talk) 17:55, 12 May 2008 (UTC)[reply]
200 was probably referring to Attribution of recent climate change#Key attributions#Land use#Livestock and land use. Zain Ebrahim (talk) 18:14, 12 May 2008 (UTC)[reply]
(edit conflict) In theory, less people eating meat would mean less land used for grazing lands. Less rain forests clear-cut to create grazing lands leaves more trees. More trees consume more carbon dioxide. And so on. Of course there's all that methane not being produced by farting cattle as well. --LarryMac | Talk 18:18, 12 May 2008 (UTC)[reply]
It's not in the farts, the cows breathe out (most of) the methane. DirkvdM (talk) 08:34, 14 May 2008 (UTC)[reply]
Change your incandescent lightbulbs to compact fluorescent lamps.118.90.102.125 (talk) 12:38, 13 May 2008 (UTC)[reply]

As it is scientifically proven that humans emit carbon dioxide, the best way to reduce CO2 emissions is to engage in genocide of the human race.--WaltCip (talk) 12:59, 13 May 2008 (UTC)[reply]

Nope, Humans themselves are carbon neutral. All the CO2 you emit ultimately comes from plants who sucked it out of the atmosphere. Some serious suggestions are:
  • Walk short distances and use a bicycle for medium ones. Also good for your health.
  • Use less air conditioning (and lobby for reasonable settings in public buildings). If it's 35 degrees Celsius outside, its nonsense to cool a building to 20. People adequately dressed for outdoors will catch a cold...
  • Preferably buy food that is in season, and from local suppliers if possible.
  • Stop whining about high gas prices. Indeed, lobby for a tax system that taxes products roughly proportional to environmental impact. This motivates people to invest in energy-saving devices.
  • Buy electricity from renewable sources, if available. In this case the market impact is probably more in the signal than in the direct effect (many electricity companies split their existing mix and sell the clean part at a higher price, and the remaining worse mix to people who don't care...), but it still is a signal.
  • A solar warm water system is cost-effective in many regions. Consider this especially if you have a pool or other large consumer.
  • Most important is probably to work towards a general recognition that this is a problem, but that many solutions have upsides that compensate for costs (better quality of live, cleaner environment, less dependency on foreign oil, more jobs in new industries). Existing large players stand to lose (e.g. the oil and coal industry), but the overall economic impact may well be positive. Germany, e.g., is creating a lot of high-tech jobs in the solar and wind energy industries. Pro-environment does not have to be anti-market! --Stephan Schulz (talk) 13:27, 13 May 2008 (UTC)[reply]

I know someone mentioned this above, but I'm not sure it was taken seriously - eating vegetarian is probably the most influential thing a person can do to personally reduce climate change - according to a recent UN report, http://www.un.org/apps/news/story.asp?NewsID=20772&Cr=global&Cr1=environment, "Cattle-rearing generates more global warming greenhouse gases, as measured in CO2 equivalent, than transportation". Pretty impressive. --Bmk (talk) 19:20, 13 May 2008 (UTC)[reply]

It is indeed astounding how little useful information can be found about this. Switching to fluorescent lamps is mentioned quite a lot, but lighting only represents a few percent of CO2-emissions. As I understand it, the really big energy guzzlers are transportation and heating/airco, which together cause almost half the CO2-emissions. Of course, with the temperature control it depends a lot on where you live (hot or cold climate). Wearing a sweater indoors in winter in a cold climate can help a lot. Concerning transportation, options range from keeping your car tires sufficiently inflated to living close to where you work (or vice versa). Most cars are waaaaay to powerful, so get the most energy-efficient one (can still be luxurious - why not?). And don't accelerate too fast. That really makes a big difference and it one of the easiest things to do. And of corse, in concurrence with Zain, the easiest thing you can do is vote for the party that is most focused on solving this problem (both the reduction of energy use and the development of alternative energy sources). I've been doint that for over ten years now. I've become a one-issue voter. DirkvdM (talk) 08:34, 14 May 2008 (UTC)[reply]
I think one of the reasons why fluorescents often come up is because even though the amount they contribute may be small, switching is easy to do, relatively painless for the vast majority of people and generally quite cost effective even in the relatively short term. Improving insulation, particularly in temperate countries, is another common suggestion but it can be expensive. Common methods to try and do this include better building standards, so all new homes at least have a resonable degree of insulation and some sort of tax relief for people carry out renovations to improve their insulation. Over powered cars is a factor, but it's something difficult for governments to deal with directly without too much controversy. But resonable petrol taxes for example, which encourage the efficient use of petrol (and therefore the use of energy efficient i.e. not overpowered cars) can help (they also have the added bonus of reducing the subsidy other people pay to road users and making it easier to subsidise things like public transport which have a far greater benefit then excessive road building). The recent high price of petrol has definitely helped, particularly in the US which has always had very low petrol taxes compared to much of the rest of the world although it hasn't always had the completely desired effect (for example, the growth in hybrid SUVs in the US which will good, it would have been far better if the people had chosen less overpowered vehicles). Stuff like going vegetarian, which while an excellent idea in practice, isn't something you can convince a lot of people to do Nil Einne (talk) 11:42, 19 May 2008 (UTC)[reply]

Protoplast fusion

while performing protoplast fusion we have to chop leaves into very fine peices before proceeding for actual fusion. so why don't the cells die out at that moment?--Kayatheangel (talk) 13:19, 12 May 2008 (UTC)[reply]

Presumably the cellular metabolism continues. Plant cells may not have to be so isolated from their neighbours by an intact cell wall to function in the short term. You would have to keep a large number of the organelles intact. Some of the membranes could reform spontaneously. The cells would die in the natural environment when chopped apart, because they would dry out and lose their root nutrient supply. Graeme Bartlett (talk) 21:46, 13 May 2008 (UTC)[reply]

Finding CpG islands with genomic databases

How do I determine which area of a gene can be designated 'CpG' island? Suppose that I'm interested in this gene; I vcan see that it has a lot of CpGs but can I get a program (preferably online) that defines the CpG island(s)? ----Seans Potato Business 13:34, 12 May 2008 (UTC)[reply]

Widely-distributed, semi-technical history of moon shot?

Was there anything like the Smyth Report for the Apollo program, or for the Apollo 11 mission (moon landing) specifically? The Smyth Report was a government-produced history of the atomic bomb that came out contemporaneously with its use, to explain in more detail what people were seeing on the front pages of newspapers after the bombings of Hiroshima and Nagasaki. Anything like that for the space race? I don't mean official histories that came out years later, or unofficial histories/popularizations. --140.247.242.128 (talk) 17:16, 12 May 2008 (UTC)[reply]

While I can't directly answer the question, bear in mind that the Apollo program, and the related programs preceding it, were carried out in view of the public, in contrast with the secret Manhattan Project. There is not, as a result, the need for a parallel to the Smyth Report. — Lomn 19:45, 12 May 2008 (UTC)[reply]

Oceans, Lakes, and Seas

What is the differance between an ocean,lake,and a sea? —Preceding unsigned comment added by Unclejojo (talkcontribs) 17:43, 12 May 2008 (UTC)[reply]

(I made a new section, if no one minds) An ocean is a large salty body of water that is part of Earth's main body of water, a lake is a body of water surrounded by land, and a sea is a body of salt water surrounded by land and connected to the ocean (however, some salty lakes are called seas). 206.252.74.48 (talk) 18:46, 12 May 2008 (UTC)[reply]
A "sea" can also be a (pretty arbitrary) geographical region of an ocean, for example, the Sargasso Sea, the South China Sea.
Atlant (talk) 19:05, 12 May 2008 (UTC)[reply]
Well, you would know, Atlant! I'll also point out that in addition to the above usages, the Earth's main body of water can be called "the ocean" or "the sea". --Anonymous, 22:27 UTC, May 13, 2008.
:-). It's not a coincidence, by the way.
Atlant (talk) 17:14, 14 May 2008 (UTC)[reply]

So the Salton Sea and the Dead Sea are lakes. Edison (talk) 03:38, 14 May 2008 (UTC)[reply]

Earthquakes

I understand how earthquakes start - the release of pressure from two tectonic places which are pushing against each other or rubbing across each others boundary's - but how do earthquakes stop? What stops the plates from continuing to move? Mr Beans Backside (talk) 18:08, 12 May 2008 (UTC)[reply]

As I understand it, the plates are moving pretty much the whole time. An earthquake happens when the plates get stuck and pressure builds up until it is forcefully released and the plates continue moving again. --Tango (talk) 18:18, 12 May 2008 (UTC)[reply]
Or put another way, the plate portions right at the boundary get stuck again when enough strain energy has been released. Stress starts to build up again, since the plates as a whole are moving, but they are jammed against each other at the boundaries. Rub your fingers hard across a rough surface - you hand is moving continuously, but your skin is stopping and starting. Franamax (talk) 19:16, 12 May 2008 (UTC)[reply]
What you describe are interplate earthquakes. There is another kind Intraplate earthquakes. They are rarer, but generally more destructive. You might also want to have a look at Subduction. Lisa4edit (talk) 20:21, 12 May 2008 (UTC)[reply]

Paleontology

How do paleontologists determine the sex of a fossilized creature? Mr Beans Backside (talk) 18:09, 12 May 2008 (UTC)[reply]

I think it depends on the creature, but there are usually things that differ between the sexes. The total size, the size of certain body parts, the shape of certain body parts, etc. --Tango (talk) 18:16, 12 May 2008 (UTC)[reply]
But all you would have is bones? Does the bone structure differ between the sexes? Mr Beans Backside (talk) 18:24, 12 May 2008 (UTC)[reply]
Bone structures often (although not always) differ by sex. If the creature is extant (still alive somewhere), the skeletons of the male and female can be compared for differences. The specific differences between sexes vary by species. For instance, the presence of a baculum is an indicator of male (although females can have a baubellum) in most mammals; likewise antlers in deer, pelvic structure in humans, etc. might be used. For extinct species the work is mostly analogy to living relatives, although this can be difficult. The use of ancient DNA has also been able to shed light on some of the more recently extinct creatures. Some genetic work on moa (extinct, flightless birds of New Zealand), for instance, was able to show three "species" (classified by bones) were really one species of bird but of different sexes (two sexes, of course, the original classifier, Richard Owen working in the 19th century, had based the distinction largely on size). --TeaDrinker (talk) 18:31, 12 May 2008 (UTC)[reply]
Our Tyrannosaurus article has some interesting bits on this topic. Matt Deres (talk) 20:14, 14 May 2008 (UTC)[reply]

Male to female ratio

Human offspring are roughly balanced 50% male to 50% percent female. Does this balance exist throughout nature, or has evolution resulted in more useful proportions for some species? For example, more male dear because some might die fighting each other. If proportions are unbalanced in some species, how does this work? Mr Beans Backside (talk) 18:45, 12 May 2008 (UTC)[reply]

Have you read sex ratio? -- Coneslayer (talk) 18:49, 12 May 2008 (UTC)[reply]
I'm not sure but I think animals like aphids have females or males which can reproduce asexually during certain seasons. Surely this would create such an 'unbalance'. Regards, CycloneNimrodTalk? 18:51, 12 May 2008 (UTC)[reply]
Actually the human sex ratio is not balanced at birth, since males tend to die faster. Evolution has resulted in this proportion, 50.5:49.5 as a response.
In some other species, the sex-ratio changes in response to food availability, more males in times of abundance, more females in times of scarcity. Franamax (talk) 19:21, 12 May 2008 (UTC)[reply]
More detailed answers: Wikipedia:Reference desk/Archives/Science/2007 October 26#gender ratios in a species. Rockpocket 22:01, 12 May 2008 (UTC)[reply]

Does the universe spin?

The planets, solar systems, star clusters, and galaxies all spin. Does the universe itself spin? Mr Beans Backside (talk) 19:58, 12 May 2008 (UTC)[reply]

We have no evidence that it does and no reason to expect that it would. The systems you mention, from planets to galaxies, are all examples of systems that have coallesced under the force of gravity. Gravitational contraction preserves any initial angular momentum a system may have had at random, and as a result amplifies a system's rate of rotation. However, by contrast, the universe as a whole is not a gravitationally collapsed system, and hence the same dynamics will not apply. Dragons flight (talk) 20:04, 12 May 2008 (UTC)[reply]
Apparently there are a lot of bored theorists, cause it isn't that hard to google papers on rotating universe versions of cosmology. This quotes a limit of 2×10−13 radians/year, or one revolution every 30 trillion years. Dragons flight (talk) 22:17, 12 May 2008 (UTC)[reply]
Hot, thankee. WilyD 13:10, 13 May 2008 (UTC)[reply]

This is making my head spin

Isn't "spinning" defined in relation to surrounding space? But what if there is no surrounding space? How could you tell the Universe is spinning if you have nothing outside the Universe to compare its changing position to? Just curious.  :)  The Transhumanist 21:28, 12 May 2008 (UTC)[reply]
Excellent question, and not an easy one to answer. I'm not sure if there even is a generally accepted answer. Consider this: You have two masses connected by a string, you set them spinning around their common centre of gravity and the string will go taut. What happens if you do it in an otherwise completely empty universe? They're not spinning with respect to anything else, since there isn't anything else, so does the string go taut? --Tango (talk) 21:39, 12 May 2008 (UTC)[reply]
Yes the string would go taut, the physics to calculate this does not involve any other entity. spin is not relative. It is a bit too abstract for me to talk about an entirely empty universe, instead consider a very empty space, very large space, you know the string would be taut in this case. If you want some more interesting physics, Black holes preserve angular momentum as one of the few properties that exists beyond the event horizon. GameKeeper (talk) 22:35, 12 May 2008 (UTC)[reply]
The short answer is "no" - you can tell if something's spinning due to the appearence of interial forces, namely the "centrifugal force". I don't believe this has ever been measured for the universe, which'd give some low upper limit, but I don't know what it is. WilyD 21:57, 12 May 2008 (UTC)[reply]
I'll back up this 'no'. Spin is not relative, it's not measured in relation to the surrounding space but can be absolutely determined due to the accelerations it involves. The maximum spin of one revolution every 30 trillion years (as mentioned above), would mean the universe would have to be 2000 time older than current best estimates before it did one rotation. Something rotating at a maximum of such a rate most definitely would not be described as spinning. GameKeeper (talk) 22:27, 12 May 2008 (UTC)[reply]
You can construct theoretical models of a rotating universe. There's no axis of rotation (much like there's no center to the expansion), but there is a direction (of the angular momentum vector), so these models violate the cosmological principle. This would be visible as an anisotropy in the cosmic microwave background. There was actually a claim a few years ago that such an anisotropy does exist (astro-ph/0502237). I don't know what the current status of this is, but the evidence for it is weak at best. -- BenRG (talk) 23:35, 12 May 2008 (UTC)[reply]
I'm surprised no one has mentioned Mach's principle. -- BenRG (talk) 23:35, 12 May 2008 (UTC)[reply]
Yes of course, that's why I left this thread alone. Basically (if I understand it right), the only way you know for sure you're spinning is because of the presence of distant "fixed" stars. It is the presence of those incredibly distant stars that establishes your own local inertial frame. Put another way, if the whole universe was spinning, how would you know? There would be nothing to measure it against, and you could equally say that the whole non-universe was spinning and the universe was standing still. It would make no difference. Franamax (talk) 03:12, 13 May 2008 (UTC)[reply]
Of course, there's no particular reason to believe Mach's principle is true... WilyD 04:42, 13 May 2008 (UTC)[reply]
One could easily apply Newton's Bucket argument instead - which seems just as reasonable, and experimentally indistinguishable. WilyD 04:44, 13 May 2008 (UTC)[reply]
Newton? That hack couldn't even dodge a falling apple! Reading through that article, beyond the fact that there is an external observer of the bucket, I see "true motion can be understood only in reference to absolute space" - but the notion of absolute space has been pretty throughly destroyed, along with absolute time. We're only left with relativity, and Mach used the relation to fixed/distant stars. The central axis of the bucket is pointing to one particular star, the rotating reference frame of the water is defined by the fact that other distant stars "revolve around the bucket". The point here though is that if there is nothing outside the bucket-universe, "spinning" loses all meaning, you can't define spinning if there's nothing outside to look at. That article could probably use an update to elaborate how the view of absolute measurement has since fallen apart. Franamax (talk) 07:25, 13 May 2008 (UTC)[reply]
Without doing the same experiment in an empty universe, you cannot establish Mach's Principle - there's no result anywhere, either theoretical or observational, to invalidate the bucket experiment - it does, in fact, demonstrate that linear motion is relative, but accelerations are absolute - this is the result of GR - all Mach's Principle suggested is that mass is normalised by the mass of the universe. I would argue GR is suggestive of Bucket, not Mach, but it's really neither here nor there until we get a second universe to test in. WilyD 13:09, 13 May 2008 (UTC)[reply]
It's a curious observation that many people seem naturally predisposed to think of motion as absolute and rotation as relative, when the laws of physics, as deduced over time from detailed observation of how things actually work, say it's exactly the other way around. I suspect it's because, in the everyday world we live in here on Earth, there exists an obvious absolute reference for motion (the Earth itself), but no such reference for heading, the Earth's surface being, to a first order approximation, anisotropic. We then, somehow, are prone to generalize this lack of an absolute reference from heading to rotation; even though centrifugal and Coriolis effects should be familiar to anyone who has ever ridden a carousel, somehow our conviction of the relativity of rotation is so strong that many, if they puzzle over the seeming contradiction at all, end up concluding that these effects must result from some unseen influence of the surrounding, "counterrotating" world.
Perhaps it's because humans tend to be visually oriented creatures, whereas the direct effects of absolute rotation upon our own bodies are only observable through the non-visual (kinesthetic and balance) senses. If you're sitting in a rotating chair in a windowless room, to your eyes it makes no difference if it's the chair or the room that rotates, even though, if the speed is more than a few rpm, the difference would be easy enough to feel. We don't trust our muscles and inner ear as much as we trust our eyes, and if we notice the discrepancy, it's easy enough to suspect that our non-visual senses are somehow detecting the orientation of the Earth through the walls; after all, we sense the Earth's gravity too, don't we? And the same argument can be used to explain away any other means of directly detecting the rotation of our hypothetical room, such as by holding a pendulum and seeing if it precesses or by watching the surface of water in a bucket: maybe the Earth "counterrotation" is also affecting the motion of the pendulum and the water, just as it, so the explanation goes, affects our inner ear.
Of course, counterexamples to these simple theories of "relative rotation" are easy to find, but the catch is that the theory can always be extended, for example by assuming that the rotation we feel isn't actually relative to the Earth, or to any other nearby mass, but to the entire mass of the Universe. And since we can't actually give the entire Universe a spin and see what happens, the theory then becomes unfalsifiable. Of course, at that point Occam's razor should suggest that there are easier explanations, but this does not make a particularly convincing argument if one's intuition says otherwise, particularly if one is not sufficiently familiar with physics to see that the theory without these "Machian" effects really is simpler. —Ilmari Karonen (talk) 18:53, 13 May 2008 (UTC)[reply]

Cloned fingerprints

Would a genetic clone have the same fingerprints as the original? Mr Beans Backside (talk) 20:05, 12 May 2008 (UTC)[reply]

I seem to remember hearing that identical twins have different fingerprints, so I would expect a clone to have different prints too. --Tango (talk) 20:08, 12 May 2008 (UTC)[reply]
Similar but not identical.[16] There are lots of sources on this. Gwen Gale (talk) 20:18, 12 May 2008 (UTC)[reply]
Are fingerprints also dependent on environmental factors? Would that cause a sufficient difference for them to be distinguishable? Mr Beans Backside (talk) 20:24, 12 May 2008 (UTC)[reply]
Yes. Gwen Gale (talk) 20:26, 12 May 2008 (UTC)[reply]
Hi. On the other hand, irises are even more unique than fingerprints. Even identical twins will have different iris patterns. Hope this helps. Thanks. ~AH1(TCU) 23:10, 12 May 2008 (UTC)[reply]
I would expect clones would have an even greater difference then an identical twin since it would have a far greater environmental difference (in the womb) then a twin Nil Einne (talk) 16:34, 14 May 2008 (UTC)[reply]

shortwave radio

hi, I am interested in buying a shortwave radio (not to transmit just to receive) but am interested in hearing ham radio too.

I am confused as to if i need ssb (short sideband??) with also usb, lsb and cw.

can someone clarify? also can i spend $100/£50 rather than a lot , if so please post a link.

thx —Preceding unsigned comment added by 78.144.146.123 (talk) 21:25, 12 May 2008 (UTC)[reply]

According to our article on Single-sideband modulation, you will need it to hear ham radio. See also sideband for more info on bands. As with most things, you get what you pay for, the cheaper radios seem to only receive normal shortwave broadcasts. You might also need a big antenna to pick up ham transmissions. Try googling "buy shortwave radio", there are hits for information and purchasing. Franamax (talk) 22:06, 12 May 2008 (UTC)[reply]
Just to get the terminology straight, if a radio is capable of receiving "Single Sideband" (SSB), it can receive either the "Upper Sideband" (USB) or the "Lower Sideband" (LSB). It can also receive Morse code ("continuous wave"/CW) transmissions. And I think any radio that can receive SSB (as used by most amateur radio operators) can also receive ordinary amplitude modulation (as used by most commercial and state-run shortwave radio stations. But not all AM radios can receive SSB; that takes extra circuitry in the receiver.
Atlant (talk) 00:33, 13 May 2008 (UTC)[reply]
Trivia note: if you're driving in Canada and see a license plate starting with "VE", that's a ham radio operator, the license number is their call-sign, they get it for free. Ram their car, you can ask them all sorts of questions while you wait for the police ;) Franamax (talk) 03:05, 13 May 2008 (UTC)[reply]
For several decades, amateur radio "hams" have used ssb for their voice transmissions, because it transmits farther for a given power limit and per government regulatins. Without SSB your receiver would only make their transmissions sound like Donald Duck. Look for a receiver with SSB. A Beat frequency oscillator (BFO) is needed to pick up Morse code transmissions. A radio which picks up SSB should also pick up CW (Morse code). Without the BFO capability, a radio would only pick up shortwave transmissions like goverenment radio stations (news, propaganda, etc). I have used a radio frequency signal generator with a non-ssb receiver to pick up ssb and cw transmissions, but I do not recommend it. Edison (talk) 04:34, 13 May 2008 (UTC)[reply]

Ordinary cheap shortwave receivers will have a difficult time picking up amateur radio signals. The problem is that the normal broadcast stations have a much stronger signal which will cause the weak HAM radio signal to become swamped in intermodulation noise. You need a more expensive communications receivers to be able to enjoy listening to HAM radio. If you want a good deal, you should try to get hold of a second hand communications receiver. There are many exellent communications receivers that are no longer manufactured that would cost you $1000 or more if bought new. But the second hand price will be significantly lower. E.g. why not buy a second hand Kenwood R-5000? Count Iblis (talk) 13:42, 13 May 2008 (UTC)[reply]

I have picked up hams as well as international broadcasters on old and cheap sw receivers. The broadcasters tend to be in different bands from the hams. Also the ham might be in the next town and the internatinal broadcaster might be a continent away. But CW and SSB are tricky to receive without the aforementioned special circuitry. Radio Shack had a fairly cheap receiver a few years ago which included SSB and CW reception. Edison (talk) 03:36, 14 May 2008 (UTC)[reply]
Don't know if that would affect what you are looking to set up, but have a look at Scanner (radio)#Legal issues before you get yourself into trouble. Disclaimer: Not advice, just a friendly hint.Lisa4edit (talk) 05:33, 14 May 2008 (UTC)[reply]
Edison, I agree. However, the fact that the hams operate in different bands is not enough for the cheaper radios to prevent strong intermodulation effects. Even on my R-5000 I can notice very small intermodulation effects if I listen to air traffic communication on, say, 5658 KHz. This is because when you tune to that frequency the RF stage will let the frequencies in some block through that in case of the R-5000 includes the 49 meters band. And then it is inevitable to get third order intermodulation effects caused by powerful stations on 49 meters band. This problem is much more severe on my cheaper radio. What is especially annoying is that if you listen at night (on the cheaper radio) the intermodulation effects will drown out even strong signals on the 5600 khz band.
So, before you buy a radio, you should really think about what your primary focus will be. If it is listening to broadcast stations and to ham stations, the latter only occasianaly and it wouldn't be a big deal if you can't some stations, then you could go for a radio priced $100 or even cheaper. But if you would be dissapointed if you can't receive amateur stations very well you should consider spending just a few hundred dollars more to buy a good second hand communications receiver. Count Iblis (talk) 17:38, 14 May 2008 (UTC)[reply]

Sociocultural evolution

Sociocultural evolution, the progression:  Hunter-gatherer bands → Social rank → tribes → Social stratification → chiefdoms → Neolithic Revolution →→→ Civilization: Agrarian society (Pre-industrial society): Agrarian villages → Towns → Cities → City-states → Nation-states →→ Industrial Revolution → (Modern) Industrial society →→ (Postmodern) Post-industrial society → Informational Revolution → Information society → Digital Revolution →→ Globalization → World government?

Is that correct?

The Transhumanist 21:30, 12 May 2008 (UTC)[reply]


you missed off this one:

http://en.wikipedia.org/wiki/United_Federation_of_Planets —Preceding unsigned comment added by 78.144.146.123 (talk) 22:13, 12 May 2008 (UTC)[reply]

Is it correct? Only if you assume evolution of this sort is both linear (non-branching) and teleological (driven towards a specific end). Neither of which are reasonable assumptions (or supported by evidence) in either biological or sociocultural evolution. --98.217.8.46 (talk) 00:41, 13 May 2008 (UTC)[reply]

The thing which might make World government inevitable is the failure of so many to have achieved the first tier and fewer the next and so on. -- Taxa (talk) 01:41, 13 May 2008 (UTC)[reply]
You missed out religion, which has always played a major role in social development (kings often relied on a priest-class to confirm their divine origins). That influence has varied through time and directed sociocultural evolution into several divergent paths. (Note, you can always play Civ3 to check your theory out:) Franamax (talk) 03:01, 13 May 2008 (UTC)[reply]
That seems more or less a correct hierarchy of least to most complex societies. Keep in mind that societies, as 98.217.8.46 implied, may move "backwards" so to speak. The collapse of the Roman Empire was a step back from the nation-state to the city state for parts of Europe. They may also skip steps. The Native American tribes have pretty much been absorbed into the American nation-state.
Durant has a hierarchy of his own going:
  • family
  • clan: a group of related families occupying a common tract of land, having the same totem, and governed by the same customs or laws
  • tribe: a group of clans united under the same chief
  • state: based on geographical contiguity rather than kinship
  • ...
I'm sure he goes on, though I haven't gotten very far in The Story of Civilization. — Ƶ§œš¹ [aɪm ˈfɻɛ̃ⁿdˡi] 05:45, 13 May 2008 (UTC)[reply]
An excellent read along with The Story of Philosophy. 71.100.14.205 (talk) 11:02, 13 May 2008 (UTC)[reply]

Brown versus green (fuel/CO2 versus O2 production)

Suppose I have 50 acres covered with a 15 year old stand of pine trees (another 5 years of growth needed to become suitable for harvest as lumber) for which I'm offered a certain amount to harvest now. The amount of the offer sounds reasonable but a higher reward might be possible from firewood than lumber. Before I make a decision I need to know how much energy and carbon dioxide the stand can produce and how much oxygen will be lost in order to make the right green decision. Is there a template I can use to facilitate this analysis or is all this talk about green just meaningless theory? 71.100.14.205 (talk) 22:02, 12 May 2008 (UTC) [reply]

If you want to reduce CO2, then lumber is the way to go. If you burn the wood, all the CO2 gets released back into the atmosphere, if you keep it as wood, it's stored up. --Tango (talk) 10:47, 13 May 2008 (UTC)[reply]
Unfortunately economics have to be made part of the equation such as in maximizing the profit by using the energy availability as a positive and the co2 output as a negative. A template for determining the amount of timber per acre would probably help. 71.100.14.205 (talk) 11:27, 13 May 2008

(UTC)

The "economy" depends on you location. This link gives one example for US Mississippi [17]. This may not be representative or relevant to your location. Ecologically the more uses a product gets before it is decomposed the better. Your timber will eventually either rot or get burned, but can go through quite a few cycles of use in-between. 71.236.23.111 (talk) 20:04, 13 May 2008 (UTC)[reply]
If you want to reduce CO2, then lumber is the way to go. If you burn the wood, all the CO2 gets released back into the atmosphere, if you keep it as wood, it's stored up.
But if this used as a substitute for fossil fuel use then the answer is not clear cut. Would it save more than sequestration through lumber? -- Q Chris (talk) 07:12, 14 May 2008 (UTC)[reply]

In Australia at least, the pine would have a much greater dollar value as lumber (and pine bark potting mix) than firewood anyway.Polypipe Wrangler (talk) 20:31, 15 May 2008 (UTC)[reply]

Ergonomics of press-up exercise

Are press-ups an ergonomic exercise or do they cause undue strain on the wrists? ----Seans Potato Business 22:04, 12 May 2008 (UTC)[reply]

May 13

Can my macaw breed with his macaw?

Mine: Female hyacinth macaw

My friend's: Male red and green macaw

Supposing that the two birds get along and eventually pair up, will they be able to breed? --81.77.245.65 (talk) 00:21, 13 May 2008 (UTC)[reply]

I'm no expert, but it should work. Hyacinths are reported to be very difficult to breed, though. Considering that our page lists hyacinth macaws as endangered, wouldn't you rather want to see if a zoo or breeder in the area has a male hyacinth and breed yours with that one? Just a suggestion. -71.236.23.111 (talk) 01:44, 13 May 2008 (UTC)[reply]
I don't think so. They aren't even in the same genus. bibliomaniac15 04:22, 13 May 2008 (UTC)[reply]
Bird hybrid has a couple of examples. Somewhere it said that different genera don't breed in nature, but some mate successfully in captivity. It would just be a pity in this case because the species needs the contribution of every surviving member for genetic diversity. Hope you'll decide to get a suitable hyacinth gentleman for her. Lisa4edit (talk) 05:37, 13 May 2008 (UTC)[reply]
From what I've read on the internet, any large macaw can breed with any other large macaw and produce fertile hybrid offspring. As I understand it, it can be quite a common occurrence if you have an aviary containing several different species. The other day, I saw a picture of a third-generation 'mutt' hybrid which was (apparently) descended from four different species. I've seen a few photos of Hyacinth x Ara macaw hybrids too - they do look rather odd and (IMO) quite ugly. Imagine a Hyacinth Macaw-sized/shaped head and bill on a much smaller and thinner body, with random bald patches scattered around the face.
As others have alluded to, it's probably best to find another Hyacinth if you're planning on mating her. Intentionally breeding hybrids of endangered species is something that is somewhat frowned upon amongst macaw enthusiasts. --Kurt Shaped Box (talk) 14:27, 13 May 2008 (UTC)[reply]
Why are they considered separate species, then? Isn't being able to produce viable offspring together the definition of being the same species? There might be some blurred lines between species within a genus, but I would have thought species from different genera would be more distinct. --Tango (talk) 22:14, 13 May 2008 (UTC)[reply]
Not "viable", "fertile". But there are other factors. Tigers and lions breed in captivity. In nature they have different ranges, so they don't meet. Birds also seem to be a bit more flexible reproduction wise than taxonomists gave them credit for. Some of their hybrids are fertile and do reproduce. --Lisa4edit (talk) 05:20, 14 May 2008 (UTC)[reply]
What's the technical definition of "viable", then? --Tango (talk) 14:38, 14 May 2008 (UTC)[reply]
The dictionary says this [18] for fertile they use the "reproduce" meaning [[19]]. Have some mercy on the taxonomists. What are they going to do? They spent all this time sorting things into a system splitting the finest of hairs and then those animals just go ahead and mate in some cage and throw everything off kilter :-) 71.236.23.111 (talk) 17:05, 14 May 2008 (UTC)[reply]
Go easy on the hybrid macaws too. No-one ever told them that it was scientifically-impossible for them to reproduce. :) --Kurt Shaped Box (talk) 22:32, 14 May 2008 (UTC)[reply]
Thanks for all the answers. I wasn't definitely going to attempt to breed her with my friend's macaw. Just sounding out the possibilities. I feel sorry for her when she starts going broody at about this time every year and I see her go through all that nest making behaviour for nothing. I've considered obtaining a male Hyacinth for her but they seem to be very expensive these days. Since when did that happen? My bird was only slightly more expensive than a large 'Ara' macaw when I got her in the early 80s. --90.242.185.237 (talk) 21:51, 15 May 2008 (UTC)[reply]
If your hyacinth macaw gets suffientily lonely, she may even try to breed with a water hyacinth, but I would think the probabilty of this mating producing any offspring would be rather low. :-) StuRat (talk) 12:26, 16 May 2008 (UTC)[reply]

Animal stripes

<moved from misc desk>
Do baby tigers, leopards, and zebras get more stripes or spots as they grow older, or do the stripes and spots they are born with get bigger? Mr Beans Backside (talk) 19:07, 12 May 2008 (UTC)[reply]

Don't know about either leopards or tigers, but ozelots and domestic cats are born with a fuzzy version of their general fur pattern that gets more defined with age. Some changes happen, but mostly in the "belly" area (what you see when the cat lies on its back]. Lion cubs are born with spots that disappear as they mature. OR one of our domestic cats had white tufts behind her ears, that disappeared as she matured. Some cats that appear black (Melanism) at birth can later grow stripes or spots. Hope this helps. --71.236.23.111 (talk) 20:16, 13 May 2008 (UTC) For further reading Gene expression and Cat coat genetics could get you started.Lisa4edit (talk) 21:04, 13 May 2008 (UTC)[reply]

Push/press ups with equipment

Following the ergonomic question above, I was wondering why people recommend using equipment for pushups, say holding small dumbells to the floor. Does this technique work different muscle sets or is it a ploy to sell more equipment? Julia Rossi (talk) 02:13, 13 May 2008 (UTC)[reply]

Personally, hands flat on the floor stresses the wrists more than using little things with handles which allow the hands to be more curved. Edison (talk) 04:27, 13 May 2008 (UTC)[reply]
Thanks Edison. Julia Rossi (talk) 09:58, 13 May 2008 (UTC)[reply]
Now things have changed since we thought that knuckle press-ups were "better" because they hurt more! -- Q Chris (talk) 07:09, 14 May 2008 (UTC)[reply]
Pushups using dumbells or similar are better because you can get a deeper pushup, resulting in more muscles working harder. Luxosus (talk) 14:05, 14 May 2008 (UTC)[reply]

help in cleaning up a dog's skull

Hi what steps should I take to clean up a skull that I found. here are the specifics:

  1. Found half-buried in sand used for construction (the greyish one)
  2. Most of the skull is dark brown in color. An unburied part is cracked and whitish.
  3. The lower jaw is missing.
  4. My female co-worker was disgusted when I picked it up and kept it.
  5. It looks like a dog's skull, about 15-20 cm long.
  6. It still has teeth.
  7. It smells funky but I can't find any flesh clinging to it. No idea regarding the brain though

I want to make it sparkling white and tough to boot. I hope the ingridients that you would reccomend are easily found and cheap.--Lenticel (talk) 06:03, 13 May 2008 (UTC)[reply]

For a similar question in March 08[20] and preserving a rabbit's skeleton in Feb 08 here[21] with one of the answers as ants ants ants. On a documentary I saw a forensic scientist using detergent with its enzymes or whatever, breaking down pig's parts, but don't remember that it mentioned skeletons, only ways of getting rid of bodily crime evidence. Julia Rossi (talk) 06:16, 13 May 2008 (UTC)[reply]
You should win a prize for hitting on a topic for an article missing from wikipedia. Our articles mention "preparation" and "preparator", but there's no page. Taxidermy doesn't cover it. Pathologists' Assistant I thought was a very odd link from the German wikipedia, which does have and article. --Lisa4edit (talk) 06:24, 13 May 2008 (UTC)[reply]
Then there's google[22] cleaning your own animal bones by soaking in water with some detergent and more – like how to clean a skull (taking care not to overdo things) here[23]. This article has really clean white skulls. All common ingredients and cheap as so good luck, Julia Rossi (talk) 06:33, 13 May 2008 (UTC)[reply]
I like the last two sources. I'll have a sparkling skull in no time. Thanks!--Lenticel (talk) 07:05, 13 May 2008 (UTC)[reply]

hydrocarbons

what is the formula for finding how many isomers a hydrocarbon has? —Preceding unsigned comment added by 41.204.224.23 (talk) 10:20, 13 May 2008 (UTC)[reply]

Homework alert. Read the notice at the top of the page, we will not do your homework for you. Regards, CycloneNimrodTalk? 12:09, 13 May 2008 (UTC)[reply]
On the contrary, I think it's unlikely to be a homework question (if so, it's a trick question, because there is no formula you can just write down). For questions like this the On-Line Encyclopedia of Integer Sequences is an invaluable resource. The sequences you're looking for are OEISA000602 and OEISA000628. —Keenan Pepper 19:33, 13 May 2008 (UTC)[reply]

Current in capacitor and nature of magnetic fields

I have a difficulty understanding how current passes between an insulator in capacitors. Also what exactly is the nature of magnetic fields? What passes in flux lines? Is it a particle, a massless thing with momentum? Or something else entirely? Bastard Soap (talk) 10:48, 13 May 2008 (UTC)[reply]

Current doesn't really pass across the insulator of a capacitor. What happens is that electrons accumulate on one plate of the capacitor and "holes" (the absence of electrons that would normally be there when the material is neutrally charged) accumulate on the other. The resulting electric field results in a voltage difference between the plates. (You could think of this as analogous to blowing up a balloon; the air you blow isn't really going anywhere, but is building up until the pressure exerted by the atmosphere and the strain of the rubber equals the pressure at the mouth of the balloon, or until the balloon pops of course.) The amount of charge it takes to produce a given voltage difference is defined as the capacitance of the device:
If you connect a DC voltage source (e.g. a battery) directly to the terminals of a capacitor, current will flow and charge will build up until the voltage across the capacitor equals that of the voltage source, and then no current will flow. That is why capacitors are treated as an open during DC analysis of an electronic circuit. --Prestidigitator (talk) 16:44, 13 May 2008 (UTC)[reply]

But in practice it is not correct before the capacitor charges up right? Bastard Soap (talk) 16:58, 13 May 2008 (UTC)[reply]

If you mean the treatment of the capacitor as open, yes, it's not correct until then. I don't think the reason current "passes through" a capacitor is much different from the reason it passes through a conductor. The electromagnetic repulsion between the charge carriers can get through the insulator; it's only the charge carriers themselves that can't. If you force charges to accumulate on one side of a capacitor, they repel the charges on the other side, leading to a current there. The only difference is that, since the charges have nowhere to go, there's a limit to how far you can push them before you reach the voltage limit of your battery (or whatever) and no more current flows. In the case of AC there's no net motion of the charges and so no net buildup, and the capacitor is not much different from a segment of wire. As for the nature of magnetic fields, I'll leave that for someone else to answer. -- BenRG (talk) 18:01, 13 May 2008 (UTC)[reply]
I see capacitors as very different from mere wires for AC. The "non-wire" nature of capacitors in AC circuits is useful in oscillators, filters, motor-starting, power factor correction, and phase shifting circuits. Edison (talk) 03:32, 14 May 2008 (UTC)[reply]

Darwin

Was Charles Darwin a eugenicist or a social Darwinist? --Begantrue (talk) 22:01, 13 May 2008 (UTC)[reply]

Neither, as far as I know. They're not really opposites, either, so I'm not sure I understand your question... You would probably be better off on the humanities reference desk - this isn't really a science question. --Tango (talk) 22:11, 13 May 2008 (UTC)[reply]
Neither. He was a naturalist. Also, what's this troll smell? — Kieff | Talk 23:12, 13 May 2008 (UTC)[reply]
It's a legitimate question, one historians have been asking for some time now. It's a question about what beliefs he held and wrote about, not his occupation. He did write on both topics. --Captain Ref Desk (talk) 02:48, 14 May 2008 (UTC)[reply]
For a fairly complete approach, by a respected historian of heredity, you might look up this article: Diane B. Paul, "Darwin, social Darwinism and eugenics", in Jonathan Hodge and Gregory Radick, eds. the Cambridge Companion to Darwin (Cambridge University Press, 2003), 214–239.
In short: Darwin was on the fence in both cases. He valued his cousin Francis Galton's work primarily because Galton established, as far as Darwin was concerned, that intelligence was as inheritable as stature and skin color. He believed that it was possible, as Galton claimed, that society could be breeding itself into inferiority, but he was not at all sure that society could or should do anything about that. He considered Galton's view of a state that could help regulate breeding as "utopian". Neither he nor Galton believed at all in anything that looked or sounded like even the coercive form of eugenics practiced in the United States, much less under National Socialism.
Darwin's intellectual priority was convincing people that evolution was plausible. He did this by looking for things in animals that he saw in man, as a way of bridging the gap. He was not interested in how human societies should organize themselves, not like Galton and Spencer were.
That being said, he didn't totally disown these ideas either. But eugenics was brand new and not totally formulated; social Darwinism was not yet a coherent set of principles (and certainly not under that name). Did he believe evolution applied to society? Yes, but he and everyone else just called it Darwinism at that point.
But in all things, the most salient aspect of Darwin is his fence-sitting. Read the sections on society in Descent of Man—he goes back and forth, is eugenics sensible, is it moral, does it make sense, over and over again, back and forth. (Origin of Species is written in much the same fashion, he circles around and around.) In the conclusion of that book he gets as close as he ever did to saying that perhaps the state should be keeping track of whether people marry their own cousins (he tried to get his friend Lubbock to pass a law that would mandate an investigation of this—and he himself always felt that his own children's sickly demeanors came from the fact that he had married his own cousin) and that people should give more attention to the heredity of their spouses than they usually did. But it is not very forceful, it is not the cry of what we would today consider a real eugenicist or social Darwinist. He did not denounce the ideas very strongly, but he did not support them very strongly either. They were not major forces on his agenda. --Captain Ref Desk (talk) 02:48, 14 May 2008 (UTC)[reply]

kitty's diet

If I give Fluffie nothing but meat and water, is that a guarantee that she'll get enough taurine? Is there any bird or mammal meat that's low in taurine? —Tamfang (talk) 22:09, 13 May 2008 (UTC)[reply]

Unfortunately it isn't that simple. I'm no expert in this area but a google search turns up many exceptions to the general rule that meat=taurine. There are so many exceptions that I can only suggest that you speak with your vetinarian concerning your pet's diet. BTW, the general rule is that mammal meat and fish are good sources of taurine. Poultry is not included in that rule. In fact, This Yahoo answer states that poultry is low in taurine. This paper says ruminants are a poor source of taurine. This page supports the previous two, indicating that chicken and beef are low in taurine. (Subscription required) states that raw rabbit diets are low in taurine. The list of exceptions keeps getting longer the more links I check. The best bet is to ask your veterinarian. 152.16.59.190 (talk) 06:41, 14 May 2008 (UTC)[reply]
Apart from really asking your vet. You should at least put out a pot of grass for your cat. Although cats are carnivores, that doesn't mean they only eat meat in the sense of carcass that we consider such. A catch like a mouse or a bird contains plant matter and cats also eat insects if they get to go outside. That gives them a lot more variation in their diet than "meat only".Lisa4edit (talk) 07:29, 14 May 2008 (UTC)[reply]

Thanks all! —Tamfang (talk) 16:04, 14 May 2008 (UTC)[reply]

Thailand food production

What is one way that a country can increase the amount of food they can create without hurting the environment, while increasing their economy, making a valuable asset to their people, and making it a productive and sustainable part of their agriculture for future generations? Aka- Im doing a project on Thailand and I have to find An alternative or more efficient method of food production that would be benificial, economically sound, & environmentally sustainable for Thailand. —Preceding unsigned comment added by Sleppytime (talkcontribs) 23:57, 13 May 2008 (UTC)[reply]

Not hurting the environment is a difficult one, since pretty much everything carries a risk of some damage. Ignoring that requirement, there are all kinds of options: selectively bred varieties of crops, genetically modified crops, new pesticides, new fertilisers, new agricultural tools (better tractors, ploughs, harvesters, etc), introduction of new insects to fertilise crops, the list goes on. Of course, Thailand already has quite advanced agriculture, as far as I know. There's always room for improvement, but I doubt there are any obvious things that would have a major impact, since they would have done them already if there were. --Tango (talk) 21:05, 14 May 2008 (UTC)[reply]
There is no way to create more rice without changing the environment. Rice, like all plants, animals, fungi, bacteria, and otherwise, will naturally increase and decrease in population, and drastically is very common. Environmental change will naturally happen. Mac Davis (talk) 17:23, 15 May 2008 (UTC)[reply]

May 14

MnO2

Hello. If manganese(IV) oxide is ionic (not covalent), then why is it commonly called manganese dioxide? Calling MnO manganese monoxide is a little bit ridiculous. Thanks in advance. --Mayfare (talk) 00:55, 14 May 2008 (UTC)[reply]

So you can forget the roman numerals and everyone still knows what you're talking about? The term predates IUPAC, anyway. Someguy1221 (talk) 01:17, 14 May 2008 (UTC)[reply]
Well, the term dihydrogen monoxide is widely known. :D --JDitto (talk) 03:33, 14 May 2008 (UTC)[reply]

Natural vibration of solids

So I was thinking, if a solid already vibrates naturally, what happens if it's made to stop vibrating? Does the material become more durable or more brittle? Thanks. --JDitto (talk) 03:07, 14 May 2008 (UTC)[reply]

Vibration could be due to heat, impinging sound, impacts by particles. Removing the vibrations will be the same as removing heat, or lowering the temperature. The solid will contract, it may recrystallize in another form, it may become more brittle. If you reduce the temperature close to absolute zero you will minimize the vibrations, but you will still get quantum fluctuations or zero point energy. Graeme Bartlett (talk) 03:59, 14 May 2008 (UTC)[reply]
Other interesting properties will result if you can get the temperature extremely low, where the material is in the lowest quantum state. Friction can vanish in a supersolid, and with tighter and tighter upper bounds on temperature, and therefore momentum and movent of the sonstituent particles, the location of the solid becomes vague and a Bose-Einstein condensation occurs. Graeme Bartlett (talk) 04:14, 14 May 2008 (UTC)[reply]

PHYSICS- "FORCE"

Which force is responsible for the noise made by the dry hinge of the door? —Preceding unsigned comment added by 117.96.7.67 (talk) 05:35, 14 May 2008 (UTC)[reply]

Electrostatic (repulsion between the surfaces in the hinge) and gravity (attracts the door to the Earth) --Shniken1 (talk) 05:51, 14 May 2008 (UTC)[reply]
It's sound created by friction. Wisdom89 (T / C) 05:54, 14 May 2008 (UTC)[reply]
And as Shniken1 suggests, friction is one of the many manifestations of the electromagnetic force. Algebraist 08:48, 14 May 2008 (UTC)[reply]
More specifically, it's the stick-slip phenomenon that relates to the friction.
Atlant (talk) 13:00, 14 May 2008 (UTC)[reply]

technological pudding

What does it supposed to mean? --Omidinist (talk) 06:52, 14 May 2008 (UTC)[reply]

It would help if you could supply some context.--Shantavira|feed me 07:14, 14 May 2008 (UTC)[reply]
"Modern science has succeeded in vastly increasing our knowledge of nature,
from the microworld of subatomic particles to the vastness of intergalactic space.
The proof is in the technological pudding"
(Clifford D. Conner, A People's History of Science, p. 497). --Omidinist (talk) 10:58, 14 May 2008 (UTC)[reply]


It is a rephraseing of "the proof of the pudding is in the eating". In this case, technology is the proof. 206.252.74.48 (talk) 12:26, 14 May 2008 (UTC)[reply]

Great. Thanks. --Omidinist (talk) 13:07, 14 May 2008 (UTC)[reply]

Auntie, what's the matter?

Is matter-antimatter annihilation the only way to obtain 100% conversion to energy? Clarityfiend (talk) 07:14, 14 May 2008 (UTC)[reply]

Even this is a problem as proton connecting with antiproton also produces electrons and positrons. Graeme Bartlett (talk) 11:27, 14 May 2008 (UTC)[reply]
It is far and away the closest approximation, however. — Lomn 13:11, 14 May 2008 (UTC)[reply]
Accretion onto a black hole is probably the runner up, with efficiencies in the double-digit percent range. -- Coneslayer (talk) 13:18, 14 May 2008 (UTC)[reply]
I think Hawking radiation has the same conversion rate as matter/antimatter. — DanielLC 20:35, 14 May 2008 (UTC)[reply]

CO2 + energy = fuel. But how?

Taking CO2 out of the atmosphere (or oceans) would get an extra impulse if we could do something useful with it. Such as make fuel. Since petrol is transformed into CO2, releasing energy, can't one do the reverse, take CO2 out of the atmoshere (or oceans), add energy and make a fuel, preferably one that cars can run on? In other words, I want to reverse the arrow in the exothermic reaction equation in the combustion article (eg with x=8 to make octane). But is this as simple as burning? And in what form would the energy then have to be added? Heat? Or can it also be electricity? Of course, plants do something like this with light through photosynthesis, but that has a maximum efficiency of 6%. Artificial photosynthesis is just in its early stages I understand and the first stage produces hydrogen, but it is going to take decades until the hydrogen economy has a complete infrastructure. And the second stage produces glucose, just like plant photosynthesis, something that existing cars can only use as an additive (as is done in Brazil). Is it difficult to produce the longer molecules in petrol that cars can use as-is? And which alkane would be the most logical choice? Octane seems like the most logical choice for the car, but how well can cars run on other hydrocarbons that might be more easily produced? Are longer chains more energetic?
Also, since chemical energy storage is all about the exchange of electrons (yes?), doesn't it make more sense to use those directly, in other words, use electricity for reduction to combine the short CO2 into long carbon chains?
In short, can windmills and solar cells produce petrol? And shouldn't we have a carbon energy article? After all, that is the driving force of life on earth.DirkvdM (talk) 07:48, 14 May 2008 (UTC)[reply]

Cars with proper engines run fine on alkohol or biodiesel, both of which are fairly easy to produce from e.g. plant matter. Current engines may have problems because the alternative fuels have different lubricating properties and may act as solvents for gaskets. But solutions for these problems exist - see e.g. Ethanol fuel in Brazil, a large country that is using bioalcohol as fuel for 30 years. Synthesizing e.g. octane from CO2 and, say, water, is certainly easily done in the lab, but I don't think that there is an efficient industrial process for doing this. --Stephan Schulz (talk) 08:50, 14 May 2008 (UTC)[reply]
Actually, I don't think turning CO2 into octane can be done easily/efficiently through chemical processes even at the laboratory scale. I'm sure it can be done, but there are quite a large number of steps along the way, plus wasted energy and lost material. Dragons flight (talk) 09:11, 14 May 2008 (UTC)[reply]
I did not say efficiently ;-). And I mean "easily" not as in "it takes a few minutes" but as in "a competent chemist with a laboratory and a library will figure out how to do it without new research". --Stephan Schulz (talk) 09:33, 14 May 2008 (UTC)[reply]
Bear in mind that even thoug biofuel or some alternative like hydrogen may be necessary for things like boats, planes and cars travelling long distances, a large proportion of human travel can probably occur with direct use of electricity. Intracity/commuter travel via trains and plug-in electric cars, long distance transport of good within a country or even between countries with trains. Shipping (with ships) is probably one of the most significant things going to require some form of fuel, but shipping is AFAIK already very efficient (see food miles) and may get even more so with a return to wind assisted shipping[24]. Whether this fuel will be in the form of synthetic petrol, biofuel, hydrogen or something else is anyones guess but at the moment petroleum is still cheap enough that it's unlikely to be a problem for the few things that actually need fuel. As others have stated, biofuel is actually fine as a fuel for cars, the bigger problem is the cost. Nil Einne (talk) 13:14, 14 May 2008 (UTC)[reply]
Shipping may be efficient but due to its fuel (bunker oil) is one of the most polluting.[25][26] Iceland is disscussing converting their fishing fleet to hydrogen power, however. Rmhermen (talk) 14:54, 14 May 2008 (UTC)[reply]
Also renewable power sources are not without issues, the biggest one being storage. Since most renewable sources are not reliable and depend on the conditions it's ideal if you can store the power for use when power output is low. Batteries are too low density for this sort of thing. Pumped hydro (which is inefficient and needs somewhere to store all the water) or using the energ to produce a fuel (hydrogen for example) is probably the best option we have nowadays. Nuclear power plants of course have a somewhat similar problem in that their output can only be regulated slowly unlike fuel power stations which can generally be shut down or start-ed up at will depending on need. Non-pumped hydro can of course be used as one source to release power at will although it may mean some 'waste' (when your power output from other sources is so high and your dam is full you just have to it go) and there is always the risk if you have exceptionally low rainfall you hydro backup will run out. You can of course at a pump to a hydro power plant in some instances. The other issue is of course availability. The desert is a very good place to make solar power plants but many are not surprisingly sparsely populated and deserts are quite far from a lot of Western Europe for example. So either you need very long power lines which of course means very high losses (unless you make a room temperature super conductor) or you need to convert the energy to some sort of fuel (hydrogen perhaps). You can of course build solar and other renewable power plants in Western Europe but for solar anyway deserts are probably close to being the best location. Nil Einne (talk) 15:59, 14 May 2008 (UTC)[reply]
Public transport and electric/hydrogen cars might be a nice ideal, but it will remain a reality for a long time that private cars running on petrol are the preferred mode of transport and we need something that works now. And what is un-pumped hydro and how do you let a car run on it? And transport of hydrogen and electricity is indeed more of a problem than a liquid (at room temperature) fuel. So that's why I want a liquid fuel. DirkvdM (talk) 10:46, 15 May 2008 (UTC)[reply]
"Unpumped Hydro" refers to normal water-power, using natural run-off (though usually dammed). "Pumped Hydro" is only a somewhat inefficient but cheap way to store surplus power for load equalizing. Essentially, when your grid is delivering more than you need (because of all the base load power stations running at the optimum level and low demand), you pump water from a lower reservoir back up the mountain. See Pumped-storage hydroelectricity. --Stephan Schulz (talk) 11:46, 15 May 2008 (UTC)[reply]

So back to my (now adapted) question. Is there a way to make a liquid fuel that existing cars can run on with CO2 and an electric charge? How can you make this form hydrocarbon chains? DirkvdM (talk) 10:46, 15 May 2008 (UTC)[reply]

If octane burns with oxygen to yield heat with products of combustion carbon dioxide and water, how much more energy would be required to reverse the process, ideally? What would be entailed in such a synthesis? Above it was said ""a competent chemist with a laboratory and a library will figure out how to do it without new research"." but what might go on in a future reaction vessel which had water, carbon dioxide and electricity as raw materials and octane as a product? Edison (talk) 19:13, 15 May 2008 (UTC)[reply]
In New Scientist magazine earlier this year there was an article using CO2 this way. It started with using a ceramic which was heated and cooled in a cycle to remove one oxygen to form CO gas. experimental stuff. Polypipe Wrangler (talk) 20:49, 15 May 2008 (UTC)[reply]

Where to get the CO2 from

The above question led me to wonder where one could best get the CO2 from; So where are the CO2-concentrations highest? this source says the concentration in the oceans is 60 times higher than in the atmospere. I assume that is volume-based. But if that is averaged out for the atmosphere (to which height?) then that number doesn't help because I want to know how the concentrations compare at ground level (and surface level for the oceans, or is it more evenly distributed there?). Of course, there is also the question how easily the CO2 can be extracted from water or air, but that may be affected by the method by which it is transformed into fuel. DirkvdM (talk) 07:48, 14 May 2008 (UTC)[reply]

Well, currently the easiest way is to burn less fuel in the first place. The process of converting CO2 (probably plus water if you want petrol-like stuff) back into usable fuel is even in the theoretical best case less than 100% efficient - the second law of thermodynamics sees to that. In practice, the efficiency will be fairly abysmal, I suspect - heat engines are only at 40% or so with the currently feasible or even plausible future materials. So unless you have spare energy that is not from fossil fuels, its easier and more efficient to not produce CO2 in the first place. If you decide to extract CO2 anyways, because you have spare energy (Iceland comes to mind), there are two less hopeless ways of doing it. First, collect it where it is produced - at the tailpipe or the chimney. That way you have a decent concentration to start with. Secondly, the smart way it indeed to let plants do the collection and concentrate on the conversion of plant matter into fuel. Sure, photosynthesis may only be 6% efficient, but then the solar cells grow themselves for free. --Stephan Schulz (talk) 08:39, 14 May 2008 (UTC)[reply]
You really want CO2 in the sea. Assuming all factors are yielding positive for the local phytoplankton, including the ill-sought after iron, marine snow will be capable of bringing certain amounts down to several kilometers below sealevel, which in theory can help on the greenhouse effect. I digress on your topic, but thought this was a relevant and not uninteresting bit of information, as there's a certain level of "saturation" for CO2 in water, which phytoplankton has an impact on. 213.161.190.228 (talk) 11:16, 14 May 2008 (UTC)[reply]
If you want effortless biomass, so from a natural surrounding (forest or such), then efficiency is even less than 1%, I believe (that 6% is a theoretical maximum). And then you have to harvest that and convert it to a useable fuel, each with their own (in)efficiencies. In comparison, the efficiency of solar cells is over 10% (and rising).
Anyway, the question here is about the concentration in water or air. You've got a good point with the CO2 from tailpipe or chimney, but I'm thinking about a way to harvest wind energy at sea, so not near such CO2-sources. DirkvdM (talk) 11:27, 14 May 2008 (UTC)[reply]
You could put your turbine on the coast and then grow algae in a tank. As Stephan said your "production cost" would be minimal and you'd get at the CO2. If you use wastewater to feed the algae you might get a twofer. 71.236.23.111 (talk) 16:15, 14 May 2008 (UTC)[reply]
(EC) Don't forget you need a way to convery your electricity to some sort of fuel, so your 10% quickly goes out the window. If you're using CO2, it simply doesn't seem feasible to me since we not only have no way that I know of to convert it to a fuel in mass quantities that I'm aware of, you're also almost definitely going to have to concentrate it somehow which is not going to be cheap energy wise. Hydrogen might be a better bet but even it's still far from being practicable not to mention the infrastructure investments required. Bear in mind if you're using something like jatropha or palm oil you can use the oil as a fuel with minimal processing. Obviously the planting, growing and harvesting is not free but the biomass you have left behind is probably the biggest problem sicne you're losing a lot of what your producing which is why people are trying to find ways to convert it to something usable. Algae fuel is another alternative which removes the land issue and also should eb a lot easier to harvest etc, but we still don't have a good algae Nil Einne (talk) 16:20, 14 May 2008 (UTC)[reply]
You might be interested in this lecture. [27] Some of the base data is a bit out of date, but the general principles still apply for the most part. (I think you'll need one of the players to get the full lecture.) Lisa4edit (talk) 03:09, 15 May 2008 (UTC)[reply]

WMD

I recently read some facts on Weapon of mass destruction. I have some questions on WMD especailly about the destructive power.

  1. How many nuclear warheads (each 15 KT) will be able to completely destroy a large city like the New York City?
  2. Which is more dangerous - Biological weapons or nuclear weapons i.e. which have more destructive and killing efficiency?
  3. What will be the effect on ecosystem of the planet as a whole if 1000 nuclear warheads (each 15 KT) are blasted above ground? Otolemur crassicaudatus (talk) 09:55, 14 May 2008 (UTC)[reply]
The first part is odd: 15kT is a tiny nuke, like the WW2 ones. Do you mean 15 MT TNT equivalence? What do you mean by completely destroy? Just killing all the people or completely incinerate the deepest cellar? The second part is a bit odd too: either could completely render humanity extinct. Presumably biological agents would always leave some non-human residue but so what. See Nuclear Winter for the third part. --BozMo talk 11:37, 14 May 2008 (UTC)T[reply]
He probably means something that terrorists could get ahold of. A terrorist isn't going to have a 15Mt weapon. They might be able to get a 15kt weapon, though—you could make something like that if you had the right materials and resources. No small, non-state entity is going to build a hydrogen bomb, though. --Captain Ref Desk (talk) 13:36, 14 May 2008 (UTC)[reply]
  1. No idea.
  2. Nukes are rare and expensive. It also needs technical knowledge to launch (you can't launch one on a slingshot) but is more devastating and efficient than bioweapons as it can affect larger areas. Bioweapons on the other hand is cheap, easily mass-produced and can be concocted in your parent's backyard. When released, it would only affect a small area and must be manually and stealthily released (you can't load a canister of virus/bacteria in a missile warhead, it will be incinerated) but the mass production part will compensate that disadvantage. Vaccines and drugs can also be used to diminish the weapon's effect so minus points on killing efficiency.
  3. See Nuclear winter and the more poorly written Nuclear summer.--Lenticel (talk) 11:45, 14 May 2008 (UTC)[reply]
For the first question, I mean 15 KT TNT equivalence. I just want to know an estimation that approximately how much such nuclear overheads will be able to destroy cities like the New York City. Destruction means complete destruction of buildings, all major artificial structures including the entire city population. Otolemur crassicaudatus (talk) 11:50, 14 May 2008 (UTC)[reply]
Immediate total destruction will require an implausibly large number. Going for the last few survivors in deep cellars is extremely expensive. To destroy New York as a working community so that it will fall apart without massive outside help is only a few nukes, especially if this includes ground bursts with massive fallout. So what is your setting? A few terrorists exploding 4 bombs will do massive harm, but the city will be rebuilt. An all-out global nuclear war with a 4 bomb share for New York will probably finish the city, at least for generations, if not forever. Without the industrial base and organized society, rebuilding will not happen. --Stephan Schulz (talk) 12:07, 14 May 2008 (UTC)[reply]
I do understand the cities will be rebuilt. I have only confusion about how many weapons will be able to destroy a big city. Tokyo is the most populous city in the world with a population of 33,600,000. During the Atomic bombings of Hiroshima and Nagasaki, Hiroshima's population was 255,000 and 140,000 people died in Hiroshima within 1945. Hiroshima has an area of 905.01 km². Tokyo has an area of 2,187.08 km² i.e. approximately four times of Hiroshima. Little Boy dropped over Hiroshima had an yield equivalent to 13 kt TNT. So an estimation can be made from this that to completely destroy Tokyo, at least 4 Little-boy sized nuclear weapons will be needed. And to completely destroy the New York City also, at least 4 Little boy sized nuclear weapons (each having an yield equivalent to 13-15 kt TNT) will be needed. Is the calculation right? Otolemur crassicaudatus (talk) 12:41, 14 May 2008 (UTC)[reply]
  • (ec, and somewhat repetitive now): Again, this depends on your definition. Strongly-built structures in Hiroshima withstood the blast even near ground zero, and there were even survivors (someone in a bank vault, I think) close to ground zero. Also, much of Hiroshima was lightly-build. Modern skyscrapers probably can withstand the blast better. As a lower bound, and depending on the definition of "destruction", your estimate may be right. But a single 15MT explosion will have a much stronger effect on most targets... --Stephan Schulz (talk) 13:30, 14 May 2008 (UTC)[reply]
1. A 15kt nuclear weapon would have a radius of total destruction of about 1.81 km / 1.12 mi (high blast pressure + fireball + ionizing radiation), or around 10 km2/ 4 mi2. Just eyeballing it, you'd need around 10-12 or so if you were totally ignoring the effects of fire (which is hard to predict, but caused most of the damage and death at Hiroshima and Nagasaki). Or, if you had just 1Mt warhead, that would do the trick in taking out all of Manhattan and most of the surrounding boroughs.
2. It depends who has them and who is going to use them and how many are going to be used and so forth. Personally I think nuclear arms are easier to control that biological arms, but a well-placed nuclear weapon can easily be more destructive than biological weapon.
3. Where, presumably, are 1000 15kt weapons to be found? Now you're into the realm of talking about the arsenals of states, which are not 15kt fission weapons, but in 100s of kts in fission-fusion weapons, which have a very different fallout profile. --Captain Ref Desk (talk) 13:36, 14 May 2008 (UTC)[reply]
Check out list of nuclear tests to see just how little 1000 * 15kt is compared to the atmospheric testing that actually was done. Just the Tsar Bomba alone is several times higher yield than those 1000 bombs put together, although yield is not the most important factor in determining environmental effects. --Sean 14:24, 14 May 2008 (UTC)[reply]
The fact is that we do not know what the effects of modern biological weapons would be because they were never used. Lenticel's comment above is incorrect in that highly effective agents cannot simply be prepared in your backyard and biological agents were certainly planned to be deployed by missiles - even as recently as in Iraq. While toxin-type biological weapons (which require contact with the deployed agent) would probably not approach the death tolls of a nuclear blast, infectious-type weapons (which only require contact with an infected victim) might be another matter. Let's hope we never find out. Rmhermen (talk) 14:40, 14 May 2008 (UTC)[reply]

It's exciting to be able to fill your brain with so much information, but then it begins to itch. You want to share this knowledge to an equal intellengent person who might understand. But, as I heard befor -you can get so busy thinking up answers the question you might have though: should you share. You guys are talking Monsters. Your wilding the answers around on an open page like a kid with toy gun to some you don't even know and in todays world Monsters aren't the same as 20-30 years ago and neither are people. I know you can hungerly serch for answers, but some answers will give you nightmares for the rest of your life. Sharing these answers stuck in your Pandor's Box while may seam relaxing but your the one that hurt your brain. You need to be responsible for your Monsters. Keeper of WMD. —Preceding unsigned comment added by 69.44.236.100 (talk) 15:12, 14 May 2008 (UTC)[reply]

... what? Nimur (talk) 16:24, 14 May 2008 (UTC)[reply]
Um... If some 'mad terrorist' who wants to destroy the world is relying on the reference desk to find out the potential destructive power of nuclear and biological weapons, I think we're safe especially since the biggest problem is not knowing how destructive they can be, but actually producing them Nil Einne (talk) 16:27, 14 May 2008 (UTC)[reply]
It may also be worth pointing out that security through obscurity is not highly regarded as a defense mechanism. Open access to information encourages effective defense systems. Nimur (talk) 16:35, 14 May 2008 (UTC)[reply]
In any case, none of these questions would be useful in destroying anything. They're just about effects, and are easy to look up. You want to stop nuclear terrorism? Encourage your governments to do more to help secure nuclear materials in unstable countries. Encourage them to move towards more open relations with countries that stand a high chance of proliferation—the biggest antidote for proliferation is increased openness and exchange, it makes it both harder to hide and reduces the motivation. People asking questions on the internet has nothing to do with proliferation or terrorism. Knowing how many nuclear weapons it takes to destroy New York isn't going to help anybody out at actually doing it. --98.217.8.46 (talk) 02:08, 15 May 2008 (UTC)[reply]
For question 2, I'd say bioweapons have the potential to be more dangerous than nukes. If a bioweapon could be developed that is highly infectious, say being both airborne and waterborne, 100% fatal, and has a long incubation period, say a year, it could wipe out almost all of humanity. The long incubation period would give infected people more time to spread the disease before they show symptoms and also would delay work on a cure and/or isolation measures until everyone was already infected. StuRat (talk) 12:14, 16 May 2008 (UTC)[reply]

Masturbation

Hi,

If a woman masturbates continously, is it will be a problem on her menstrual period and is it will effect her body?—Preceding unsigned comment added by 202.124.160.212 (talk) 12:19, 14 May 2008 (UTC)[reply]

Yes, the splashing will soil the bed sheets.
Atlant (talk) 12:55, 14 May 2008 (UTC)[reply]
Replacing reply OP removed - please don't do that! I'm afraid we cannot provide medical advice. If you are concerned, you should consult a doctor. --Tango (talk) 14:28, 14 May 2008 (UTC)[reply]
What do you mean by 'continuously?' If you mean, without stopping at all, for several days, then she will suffer the normal effects from not sleeping, eating, or drinking. -FisherQueen (talk · contribs) 15:13, 14 May 2008 (UTC)[reply]
As FQ said, it depends what you mean by 'continously'. Presuming you simply mean regularly, according to [28] and [29] masturbation can be helpful in alleviating menstrual cramps. According to [30] 'solo' masturbation is not sufficiently associated with to maintainece of a fertile pattern of menstrual cycles, you need at least one sexual partner (male or female) for that (although coital intercourse is not necessary). My guess even if it isn't sufficient, a 'young' woman who masturbates regularly will still on average have higher fertility and a more regular menstrual cycle then someone who does not. According to [31] masturbation may or may not increase in the later luteal phase and may affect testosterone levels. It also appears woman has a higher desire for sex and an increase in sexual feelings in the midfollicular phase suggesting that there will likewise be a stronger desire to masturbate in this phase. Nil Einne (talk) 20:00, 14 May 2008 (UTC)[reply]

STEAM

WHEN WATER BOILS THERE IS STEAM. BUT, WHEN, WHAT TEMPERATURE IS STEAM CONSIDERED STEAM. IS IT ANYTHING PAST 212 DEG. OR STEAM PER USE REQUIRED? THANKS, DARRELL CHAPMAN —Preceding unsigned comment added by 69.44.236.100 (talk) 14:04, 14 May 2008 (UTC)[reply]

Please don't write in all capital letters, it is considered rude (and hard to read). Anyway, you may be interested in our articles on steam and superheating. While I cannot definitively answer what defines "steam" (perhaps a dictionary would be useful as well), I can conclusively state that "H2O beyond 100°C" is insufficient, even ignoring atmospheric pressure. — Lomn 14:14, 14 May 2008 (UTC)[reply]
Steam is tiny water droplets suspended in air, this is distinct from water vapour. --Tango (talk) 14:26, 14 May 2008 (UTC)[reply]
There are a lot of different uses of terminology by both laypersons and experts; I'd be cautious about drawing such a sharp distinction. In engineering, steam is the gas that you get from boiling water; it is – or ought to be – pure water, containing very little air. At 1 atmosphere of pressure, you can make it by heating liquid water past 100°C and waiting until it has all boiled into the gas phase.
Live steam is steam that's held at a temperature above 100°C. All of the water remains in the gas phase. If you pull heat out of steam – as by running it down a long, uninsulated conduit – then its temperature will remain at 100°C, but some of the gas will condense into droplets of liquid water. Engineers will often discuss steam quality, where poor-quality steam contains a lot of water droplets. Steam turbines (for example) hate poor quality steam, as the liquid water droplets are murder on rapidly moving turbine blades.
In lay usage, the term 'steam' is often used to describe the cloud that appears above the spout of a kettle. There, it's a mixture of proper steam, water droplets, and air. (If you look closely at the space right above the spout of a hard-boiling kettle, you can usually see a small region where the cloud hasn't yet formed; that ought to be essentially live steam containing no water droplets or air.) TenOfAllTrades(talk) 15:15, 14 May 2008 (UTC)[reply]
Interesting. That's a very different definition than the one I know. Is it unique to engineers? --Tango (talk) 15:55, 14 May 2008 (UTC)[reply]
It's just a semantic definition based on "purity" of the gaseous H20. Such a quantitative measure would be useful if you were doing something with the steam (as driving a turbine). Nimur (talk) 16:27, 14 May 2008 (UTC)[reply]
I believe the answer needed was 100 deg C and the area between the steam cloud and the end of the kettle. In plants where steam is generated if you walk through steam pipeways you have to carry a broom handle with you. You wave it in front of you as you walk through. if and when a flange gasket gets a pin hole (leak), a pincle thin shot of -invisible- steam will shoot out. If you pass that broom handle through that it WILL slice through that stick and or arm. Thanks. —Preceding unsigned comment added by 69.44.236.100 (talk) 18:32, 14 May 2008 (UTC)[reply]

Inductor on a bike wheel

Hi all,

I'm thinking of making a very simple circuit to power (even briefly) an LED on my bike wheel. I'm thinking that if I have a powerful magnet on my fork and a coil of wire attached to the LED on my wheel, I can induce a voltage as the wheel spins, lighting the LED. Question 1: Is that general idea ok? Question 2: How should the coil be consturcted to generate the most power? A tight cylinder with the opening towards the magnet? A torus? With/without a steel bar to wrap around?

Any thoughts much appreciated, thanks! — Sam 14:20, 14 May 2008 (UTC)

An easy way to answer all these questions would be to disassemble a commercial version of such a device and see how they did it. --Sean 14:26, 14 May 2008 (UTC)[reply]
(English version) —Keenan Pepper 15:15, 14 May 2008 (UTC)[reply]
Thanks, but I'm sure the clever engineers in this forum can work out the best way of coiling a wire without spending over $60 for commercial examples..! ;) — Sam 15:21, 14 May 2008 (UTC) —Preceding unsigned comment added by 63.138.152.238 (talk)
Coiling a wire is easy. Just carefully wrap it around a circular object - a thread spool, a pencil... if you want to go all out, you can purchase ferrite cores for a few cents each, and wrap it with a high-gauge ("thin") wire. Neodymium magnets are easy enough to get and are about as strong as you'll find. Your construction quality will affect how well you produce electric power; you may also want to think about mounting lots of magnets around the wheel to get a continuous flux. Alternatively, you could have one magnet which sweeps past the inductor once per wheel-revolution. This will generate a brief pulse of (high?) power; you could build a circuit which tries to spread that power over the full cycle until the next time the magnet comes back around. I doubt this will work very easily, since I think you will not get a whole lot of electric generation even with a good magnet. Nimur (talk) 16:32, 14 May 2008 (UTC)[reply]
If the point is to make the LED flash briefly as it passes a magnet fixed to the frame (or vice-versa), this should be a pretty easy circuit to engineer and a casual attempt at winding your own inductor will probbaly work. You should use two LEDs in antiparallel or place an ordinary silicon diode in antiparallel with the LED, though. Put a hundred turns on a steel bolt serving as a core, arrange the magnet to sweep by in close proximity, and see what happens. Try several magnet/bolt orientations. If nothing happens, add more turns to the coil until you get flashes.
Atlant (talk) 17:05, 14 May 2008 (UTC)[reply]
Theoretically The flux through an element of area perpendicular to the direction of magnetic field is given by the product of the magnetic field and the area element. you should get the best effect if you strung your wire along the wire rim. Practically I'd go with Atlant's idea. 71.236.23.111 (talk) 17:18, 14 May 2008 (UTC)[reply]
I noticed something that said that the voltage(?) would increase if the size of the circle were larger, but is this the case even if the loop is huge compared to the magnet? Would a wire wrapped around the rim of my wheel (I think this is what you are saying) really generate a current when wizzing by my little magnet attached to the fork?
Also, what is the best configuration for a given length of wire? Huge circle with fewer loops (say, around my wheel) or smaller circle with more loops? — Sam 17:45, 14 May 2008 (UTC)
Possibly you have misunderstood .111's idea (or maybe I have). 111 is not suggesting winding along the rim, rather, 111 is suggesting winding toroidally around the rim. A wire laid along the rim will not succeed in cutting any lines of magnetic flux from a fixed magnet attached to the frame. I think 111 has suggested themself that this is not very practical, you would need to wind over the face of the tyre which would rapidly destroy the wire on most road surfaces. Either that or wind underneath the tyre, but this would probably still result in the wire not lasting very long and would do nothing for tyre fit either. SpinningSpark 22:04, 14 May 2008 (UTC)[reply]
It should work fine, but you will be a sitting duck with no light when you stop, which would be very unsafe. The commercial version offers a backup so the light keeps flashing when you are stationary. I wonder if a coil (complete with iron core) from a doorbell/door buzzer would work as the pickup coil, with neodymiuym magnets attached to the spokes? Depending on how much electrical energy is supplied when the magnet passes the coil, you might feel an annoying pulsation in the pedal effort, as if the brakes were applied slightly. Normal LED bike lights run for a considerable time on very small batteries, so the energy per flash might be too slight to notice. But you never get something for nothing when extracting electrical power from human powered movement. Edison (talk) 22:37, 14 May 2008 (UTC)[reply]

Diabetes and the treatment of warts

(This is not a solicitation for medical advice. I’m asking the question out of intellectual curiosity. I have access to professional medical care, but I won’t bother my doctor with questions about medical conditions that I neither have nor have a reason to worry about.)

I’ve seen warnings on some over-the-counter medications for treating warts that the medications/treatments are not for people with diabetes. What’s the connection between diabetes and wart treatments? I assume that a doctor would have more options in treating a patient who suffers from both diabetes and warts. How may a doctor treat a warts patient who’s also diabetic? —Preceding unsigned comment added by 72.78.102.153 (talk) 16:21, 14 May 2008 (UTC)[reply]

People with diabetes mellitus are more likely to develop plantar warts which are caused by the human papilloma virus and these take longer to heal. I'm not sure on the physiology of why you can't use the treatments, however. Regards, CycloneNimrodTalk? 18:55, 14 May 2008 (UTC)[reply]
The warning is to avoid being accused of causing or aggravating a diabetic foot ulcer. The salicyclic acid in the wart preparations can damage normal skin and some people with diabetes have reduced sensation and impaired circulation and healing. If the warning is understood and the preparation is used with care, it is not otherwise risky to diabetic feet. —Preceding unsigned comment added by 159.14.240.230 (talk) 21:03, 14 May 2008 (UTC)[reply]

healthcare Infomatics

What is the meaning when information articles are characterized as "white page or white page article" ?Rlhandy (talk) 18:17, 14 May 2008 (UTC)rlhandy[reply]

Maybe it's a white paper? DMacks (talk) 19:20, 14 May 2008 (UTC)[reply]
My understanding of "white page" is that nothing has yet been decided or written and the writer is therefore free to make a construction from scratch. If the OP would give a link to where this was seen it would help to decide if this is the meaning. SpinningSpark 21:51, 14 May 2008 (UTC)[reply]

Avoidance of other people embarrassing themselves

Social anxiety can include avoiding social interaction out of a fear to embarrass oneself. But is there a technical term for (pathological) avoidance of situations where other people could potentially embarrass themselves? I've looked around this site, but I can't seem to find it. Social anxiety by proxy maybe? 82.210.125.161 (talk) 18:57, 14 May 2008 (UTC)[reply]

Agoraphobia is the closest thing I can come up with off the top of my head. Wisdom89 (T / C) 22:32, 14 May 2008 (UTC)[reply]
:-) Overanxious mother syndrome? I'd say you are on the right track with social anxiety. If avoidance is involved I'd go with "a type of social phobia". That's the closest I'd know, but I'm no expert. --71.236.23.111 (talk) 22:38, 14 May 2008 (UTC)[reply]
I would think that it would be related to a very high (overly high?) sense of empathy. Feeling anxious when other people are in terribly embarrassing situations (like many of us experience while watching embarrassing scenes in movies) is because of our ability to put ouselves into other people's shoes and feel like these things are happening to us, in the same way that watching someone in pain can be exceedingly painful. — Sam 13:19, 15 May 2008 (UTC) —Preceding unsigned comment added by 63.138.152.238 (talk)
Psychological projection might have something interesting. But still no definite term.Lisa4edit (talk) 13:58, 15 May 2008 (UTC)[reply]

Does stars have that divine light , the reason for creation? Which guides humanity and other creatures? if so, where are they channelized?

I was reading a book, which says if we can attain oneness, we can attune with the nature around.. as well as with the oness of other creatures.is this oness..or the divine light..or the golden light..suppossed to come from one channel ( we call Heaven)-& perceived by only those , of the likes - Jesus, Hindu Gods - Shiva, Durga, Allah?--who have same commn star..to guide & protect them? also does this energy frm the light of stars affect ur lives, which can inturn mark the change in century? Trekkersdelight (talk) 21:26, 14 May 2008 (UTC)[reply]

Doesn't really seem to be a science question. Try the Humanities Desk. SpinningSpark 21:46, 14 May 2008 (UTC)[reply]
The common star for all these is the sun, and this surely does affect our lives. But the basis of this book does not sound like science, so this is probably not the desk to ask this question or make your statement. Stars do not mark centuries, these are a human creation as part of the calendar. Graeme Bartlett (talk) 21:47, 14 May 2008 (UTC)[reply]
Stars are often "channeled" or "channelized" here --Bmk (talk) 01:48, 15 May 2008 (UTC)[reply]
If oneness is attained how could there be stars, light, channels, perception, protection, guidance, lives, centuries, etc? If these differences remain, it's not oneness. Pfly (talk) 06:35, 15 May 2008 (UTC)[reply]

Tapeworms as a weight-loss aid?

Does WP have an article on this, as a matter on interest? Occasionally, I have read about people intentionally cultivating their own internal parasite(s), the thinking being that healthy, well-fed worm = slimmer human. Don't know if it actually works or not (I suspect that the effect is minimal) but I would like to read more about it. --Kurt Shaped Box (talk) 22:27, 14 May 2008 (UTC)[reply]

The pathology section of the Tapeworm does mention weight-loss as a possible symptom, but there's not much information. I think if you read the other symptoms you'll agree that self-infecting with Tapeworms is a terrible, terrible idea. And I might mention that I think vitamin deficiency is more likely than weight-loss. Ugh. --Bmk (talk) 00:16, 15 May 2008 (UTC)[reply]
On the other hand, is keeping a pet tapeworm in your innards particularly more extreme/nasty in terms of unwanted effects than some of the other methods that people try in an attempt to control their weight - Olestra, amphetamines, Liposuction, Stomach stapling, various diet pills etc.? --Kurt Shaped Box (talk) 00:29, 15 May 2008 (UTC)[reply]
I would definitely prefer getting Liposuction over self-infecting myself with a tapeworm *spine shivers* 76.196.10.3 (talk) 01:22, 15 May 2008 (UTC)[reply]
That's a fair point KSB, but I still would rather have surgery or some other treatment than tapeworms, even if they worked well - they give me the creeps. --Bmk (talk) 01:45, 15 May 2008 (UTC)[reply]
Agreed. Plus you may end up actually gaining weight, because your body will try to compensate for the lack in nutrients by making you eat more. --71.236.23.111 (talk) 02:04, 15 May 2008 (UTC)[reply]

worms can also help against asthma :) Count Iblis (talk) 02:29, 15 May 2008 (UTC)[reply]

Just a rumour? Maria Callas' radical weight loss raised speculations, among them, tape worm.  ; ) Julia Rossi (talk) 11:03, 15 May 2008 (UTC)[reply]
That's a great article idea, we should have at least a well-done section about it. Mac Davis (talk) 17:17, 15 May 2008 (UTC)[reply]
I'm having a deja vu moment here, didn't I just ask a question about this recently ? I believe I mentioned that other seemingly disgusting creepy crawlies are sometimes used medically, like maggots to eat away gangrene and leeches to restore blood flow after finger reattachment surgery. Our tapeworm article says that, while many nasty effects can happen from tapeworms, they rarely do: "According to Intestinal Cestodes, authors Craig, and Ito 2007 suggest that the effects of this gut dwelling Cestodes are usually very minimal". This compares with the very common and disgusting side effects, such as anal leakage, associated with commercial weight loss formulae like Alli. StuRat (talk) 11:45, 16 May 2008 (UTC)[reply]
One thing I could imagine regularly occurring amongst tapeworm aficionados, which might lessen the effectiveness of the method is the "I've got a worm inside me, so I can get away with eating more now!" effect. --Kurt Shaped Box (talk) 12:18, 16 May 2008 (UTC)[reply]

Trees in paper

If you have 100 cubic feet of wood with whatever density most trees have, how many 8.5" x 11" pieces of paper can you get. Inversely, how much wood goes into making 100 sheets of paper? Not homework, by the way. Thanks, schyler (talk) 23:16, 14 May 2008 (UTC)[reply]

The first problem is determining how much of the tree's lignum can be converted to cellulose. Figures differ depending on the method applied and who states the figures for what purpose.
Figures on cellulose content I've found range from 30% [32] to 50-55 % of a tree’s volume [33]. Wood pulp describes various methods for turning wood into pulp from which cellulose can be produced. Each method offers a distinct rate of efficiency with trade offs in quality, energy use and water pollution. So there is no direct answer. Maybe someone else can narrow it down some. --71.236.23.111 (talk) 02:00, 15 May 2008 (UTC)[reply]
The number of sheets of paper you can make will depend highly on the thickness of the paper. The amount of wood needed for a 100 sheets of tissue-thin paper (say for a bible) will be different than that for 100 sheets of thick letterhead. - You may want to look at the Papermaking article. -- 128.104.112.147 (talk) 23:06, 15 May 2008 (UTC)[reply]

May 15

Identify the spider

Who am I?

I'm sure it's a pretty common species, and we probably already have zillions of photos of it on Commons... but still... what kind of spider is it?

Photo was taken in Indianapolis, Indiana in July of last year. ~ ONUnicorn(Talk|Contribs)problem solving 02:29, 15 May 2008 (UTC)[reply]


That appears to be a grass spider, genus Agelenopsis (all I did was look up "common spiders" and found it pretty quickly...) --DrVornado (talk) 02:34, 15 May 2008 (UTC)[reply]

Thanks. It does look like the spiders on that page... though the web it was on sure seemed sticky (see all the stuff stuck to it in the picture?) :) ~ ONUnicorn(Talk|Contribs)problem solving 12:37, 15 May 2008 (UTC)[reply]

Inventions & Inventors

1. Like to know the names of atleast 5 inventors who died testing their invention 2.Kindly provide a brief detail about the invention and how the person died

Thanks 118.92.106.168 (talk) 06:17, 15 May 2008 (UTC)[reply]

  • William Bullock died after getting caught in his rotary printing press.
  • Otto Lilienthal died from injuries after a gliding accident
  • Franz Reichelt died testing his parachute-coat.
  • Thomas Midgley, Jr. invented a system of strings and pulleys to lift him from bed after he contracted polio. He got entangled in the ropes of this device and died of strangulation.
  • Alexander Bogdanov experimented with the concept of blood transfusion. He died after blood of a student suffering from malaria and tuberculosis was given to him in a transfusion.
  • Marie Curie developed the radium isolation process. She later died from aplastic anemia, likely due to exposure to radiation. As our article says, "much of her work had been carried out in a shed with no safety measures. She had carried test tubes containing radioactive isotopes in her pocket and stored them in her desk drawer, remarking on the pretty blue-green light the substances gave off in the dark."
Rockpocket 06:45, 15 May 2008 (UTC)[reply]
Believe it or not we actually have an article on the subject: See List of inventors killed by their own inventions. --Cameron (t|p|c) 08:55, 15 May 2008 (UTC)[reply]
Wikipedia, you never cease to amaze me! DMacks (talk) 16:38, 15 May 2008 (UTC)[reply]

Sinking of London

London is sinking by 1-2 mm per year due to sea-level rise. If London is sinking by 1-2 mm per year, then a calculation can be made that by 2100, London will sink 100 mm and by 2200, it will sink upto 200 mm. My question is that if a city like London sinks 200 mm, then what will the effect? Will it become inhabitable? Otolemur crassicaudatus (talk) 09:02, 15 May 2008 (UTC)[reply]

The last major flood in central London was the 1928 Thames flood. Today central London has excellent flood defences - see Thames barrier. Other parts of England are not so fortunate - see 2007 United Kingdom floods. Gandalf61 (talk) 11:01, 15 May 2008 (UTC)[reply]
This may be completely irrelevant but there was recently a film on TV about the flooding of London. It was cleverly named 'Flood', this is the link if you're interested: Flood (film). --Cameron (t|p|c) 11:03, 15 May 2008 (UTC)[reply]
I am not talking about flood, I am talking about permanent sea level rise due to global warming. What will be the effect if London permanently sinks by 200mm? Otolemur crassicaudatus (talk) 11:30, 15 May 2008 (UTC)[reply]
The only logical assumption I could make is that the water level of the Thames and other rivers will also rise by 200mm? It's clearly not a huge issue for London anyway. The Thames Barrier has more than enough leeway for 2cm of water rise. Either of two things would be needed for a devastating effect: a) a significant flood which the barrier can't handle or b) global warming to continue rising the sea level until the level of the Thames is too high. Regards, CycloneNimrodTalk? 12:03, 15 May 2008 (UTC)[reply]
The Thames Barrier only protects against floods when it's closed. If sea levels rose, it would have to be closed more often, which could hurt the city's economy a bit, I expect. If they rose enough that the barrier had to be permanently closed, a new solution would have to be found. --Tango (talk) 12:59, 15 May 2008 (UTC)[reply]
I guess not a lot, except they may have to raise flood defences by a similar amount. 20cm isnt that much anyway. -- Q Chris (talk) 12:05, 15 May 2008 (UTC)[reply]
You can't make this calculation without some theoretical model of the future behavior of the London-Thames system. It's looks like you're linearly extrapolating the data without any theoretical motivation, which would make this similar to the prediction that the entire world's population will consist of Elvis impersonators by 2015. -- BenRG (talk) 13:11, 15 May 2008 (UTC)[reply]
BenRG is right about it not being linear. Firstly there are two processes combining: relative to a fixed point (eg. the centre of the Earth), there is an isostatic subsidence of the land due to the effects of ice during the last ice age, and the global rise in eustatic sea level due to the various effects of global warming. The subsidence may be approximately linear, although it's probably slowing down slightly. The rate of global sea level rise is controversial, but likely to be increasing. There's a more detailed explanation of the figures here, and some interesting stuff on the Thames barrier here. Eve Hall (talk) 17:56, 15 May 2008 (UTC)[reply]
As an aside, London is currently inhabitiable. That is to say, it is able to be inhabited. The question is whether or not London would be made uninhabitable due to its sinking. Ahh, English! — gogobera (talk) 17:21, 15 May 2008 (UTC)[reply]
Hi. Well, the 10 cm of sinking in 100 years might not be a problem, but sea level rise in 100 years might be, as it could be anywhere between 200 - 2000 mm. If it's closer to the latter, then more than likely the Thames Barrier would have to be permanently closed, or else 3 million Londoners would need to find new homes. Also, the sea level rise might speed up the sinking, by compressing the soil and eroding it. Venice is also sinking, and sea level rise would make it worse, but make coastal city-dwellers could learn from the Venicians and survive by paddling through the streets. Thanks. ~AH1(TCU) 17:42, 15 May 2008 (UTC)[reply]
Permanently closing the barrier isn't an option - where would all the rainwater from upriver go? If sea levels rise such that the barrier can't at least be opened at every low tide, some other solution would have to be found. What is the typical tidal variation at the barrier, anyone know? --Tango (talk) 13:33, 16 May 2008 (UTC)[reply]

Just to clarify, but if I'm reading the introduction to the article correctly, the "sinking" is not due to sea level rise, but combining with it to produce a "double whammy". That is, any sea level rise is a separate issue from the landmass sinking. I didn't have time to read the article, but shouldn't the islands still be rising from post-glacial rebound? Matt Deres (talk) 18:31, 15 May 2008 (UTC)[reply]

London isn't too bad off compared to other parts of the North Sea. Dragons flight (talk) 18:56, 15 May 2008 (UTC)[reply]

Cold Cathode Fluorescent Lamps used as Backlights in LCD computer monitors and televisions

With reference to the Wikipedia article on Cold Cathode:

http://en.wikipedia.org/wiki/Cold_cathode

I should be very grateful if either a contributer to the article or a user knowledgeable about the construction and technology of CCFLs used as LCD backlights in televisions and computer monitors could clarify whether these CCFLs actually do contain a radioactive material that emits beta particles to start the ionization of the gas. If this is the case, then would it be correct to assume that facing the back of the computer monitor where the backlight can be seen shining through the grill that one is being exposed to ionizing radiation - since the fast moving beta particles would penetrate the thin glass of the backlight? 78.147.162.37 (talk) 12:13, 15 May 2008 (UTC)[reply]

Julia Howard —Preceding unsigned comment added by 78.147.162.37 (talk) 12:01, 15 May 2008 (UTC)[reply]

I'm no expert, but I should think any possible risk is very, very low here. Please consider self-powered lighting. There, a much more powerful source of beta radiation (that is, a source of high-energy electrons) is used to directly cause the phosphor coating of the tube to glow. Yet the enclosing tube is still glass (albeit, possibly leaded glass) and there seem to be few worries about radiation exposure unless the glass tube is ruptured. In a CCFL, any possible radiation source is there just to facilitate initial ionization, much like in many neon lamps (where ionization sources are definitely used). The magnitude of the radiation from this source must be many orders of magnitude lower than with the self-powered-lighting.
Atlant (talk) 12:47, 15 May 2008 (UTC)[reply]

Heel on wooden floor

When we move on a wooden floor with heel shoe why marks or scratches are left on the floor? —Preceding unsigned comment added by MAQMAQ (talkcontribs) 12:08, 15 May 2008 (UTC)[reply]

I'm not sure I understand the question so let me ask a clarifying question: Are we talking about shoe heels in general or a stiletto heel? In the first case, marks can be left because the rubber material of the heel can rub-off and transfer to the floor. Alternatively, small abrasive particles such as sand that become embedded in the heel can abrade the floor. In the second case, the concentration of weight on the very small area of the stiletto heel can exceed the strength of the floor, causing crushing of the material of the floor.
Atlant (talk) 12:57, 15 May 2008 (UTC)[reply]

Discharging a capacitor on a metal sheet

Hi all,

Let's say I charge a capacitor. The capacitor now has one plate with more negative charges than positive, and one plate with more positive charges than negative. If I then touch one of the plates to a hunk of metal, will I discharge half the capacitor? I'm guessing yes, because the charges between the plate and the metal will balance out, and since the tiny capacitor won't make much of an effect on the hunk of metal, both the plate and the metal will end up with (almost) no net charge. Is that right?

Second question: What will happen if I touch the negative side of a battery to a hunk of metal? Anything at all? Thanks!

You still have a capacitor, but now it's formed between the one original plate of the capacitor and the other plate acting in combination with the hunk of metal. The charge that was on the plate of the capacitor that is now in contact with the hunk of metal sheet will re-distribute itself among the new, expanded plate so as to minimize the stored potential energy. If you then separate the capacitor plate from the added hunk of metal some of the charge will be left behind on the hunk of metal and the original capacitor will end up holding less differential charge than when you started.
Atlant (talk) 15:59, 15 May 2008 (UTC)[reply]
However, it seems like the electrostatic field between the positive and negative plates of the capacitor would tend to keep the charges on the plates rather than letting them disperse much onto the metal sheet. Attraction between those positive and negative charges and all that. --Prestidigitator (talk) 16:06, 15 May 2008 (UTC)[reply]
Something similar will happen with the battery: a bit of charge will leak off the battery to the hunk of metal and remain there when you remove the battery, but not much. In the case of the capacitor the charge on each side of the dielectric is fixed but the voltage will change slightly when you add the hunk of metal; in the case of the battery the voltage across the terminals is fixed but the net charge on each side will change slightly. -- BenRG (talk) 19:09, 15 May 2008 (UTC)[reply]

Mice and candle wax

Do mice eat candle wax because it is good for them or because there is nothing else and they are hungry? Because my bro said the mice in his house were eating the candle wax from his candles. What nutritional benefit would a mouse get from eating the candle wax from a candle? Terror toad (talk) 13:13, 15 May 2008 (UTC)[reply]

Depending on what the candles are made of, they can certainly be nutritious. I did a bit of survival training as a Scout and one of the things we were told was good to have with you is an old fashioned tallow candle. It can provide light, help light a fire and if you get desperate, you can eat it - it's just animal fat. Beeswax is also edible. I'm not sure paraffin wax would be too nice to eat, though, and that's the most common wax in modern candles according to candle. --Tango (talk) 14:10, 15 May 2008 (UTC)[reply]
I've wondered the same about soap, which mice will regularly consume from my parents' summer cottage. Can they really digest the saponified fatty acids? —Ilmari Karonen (talk) 15:54, 15 May 2008 (UTC)[reply]

spectral lines of helium

What is the formula for the spectral lines of helium? Em3ryguy (talk) 14:16, 15 May 2008 (UTC)[reply]

I don't think "formula" is quite the right term, as there's no f(x)=y sort of thing, but here's helium's spectrum. — Lomn 14:47, 15 May 2008 (UTC)[reply]
If you're talking about singly-ionized helium, you can use a version of the Rydberg formula. -- Coneslayer (talk) 15:36, 15 May 2008 (UTC)[reply]
No. The full spectrum. Em3ryguy (talk) 15:54, 15 May 2008 (UTC)[reply]
Well, you can look up all the spectral lines of hydrogen in the CRC handbook here (warning - large PDF file). Does that help? --Bmk (talk) 17:06, 15 May 2008 (UTC)[reply]
You may also find this page helpful. -- Coneslayer (talk) 16:01, 15 May 2008 (UTC)[reply]

Joules, Newton Meters, and Foot Pounds

I know in metric/SI units that Joules referes to mechanical work and newton meters refers to torque, even though they're bot N x m. But foot pounds are a little more ambigious. Is it acceptable to use foot pounds interchagably between mechanical work and torque? Thanks Deltacom1515 (talk) 16:14, 15 May 2008 (UTC)[reply]

No idea why that's not formatted correctly. Deltacom1515 (talk) 16:14, 15 May 2008 (UTC)[reply]

According to our article on torque and our article on foot-pound, the foot-pound is a standard unit for both torque and work (in the US customary units system). And btw, if you have a space at the beginning of a paragraph, wikipedia puts the whole paragraph in a weird boxy thing (I fixed it for you). --Bmk (talk) 16:19, 15 May 2008 (UTC)[reply]
And as foot-pound notes, some people use "pound-foot" as the unit of torque, reserving "foot-pound" for work. -- Coneslayer (talk) 16:21, 15 May 2008 (UTC)[reply]
Yes. Further, that's what I was taught forty years ago, so it's not exactly a new convention :-)
-- Danh, 70.59.79.51 (talk) 21:09, 15 May 2008 (UTC)[reply]

Wow, way to read. I completly missed that. Guess that answered my question. Deltacom1515 (talk) 16:33, 15 May 2008 (UTC)[reply]

Area of cathode in an MFC

It is said that the area of the cathode in an MFC helps improve the efficiency of the MFC. Is this true? Why is it so? —Preceding unsigned comment added by 59.92.47.158 (talk) 16:39, 15 May 2008 (UTC)[reply]

Particle deceleration

note: question moved from Wikipedia talk:WikiProject Physics

I was wondering, since it's possible to accelerate particles to near the speed of light, is it possible to decelerate particles, and if so, how? (nb. I failed physics, so I wouldn't know myself)Avnas Ishtaroth (talk) 12:49, 15 May 2008 (UTC)[reply]

Yes. The easy way is to put a nice thick lead wall in front of your particle beam, then stand well away from the Bremsstrahlung radiation. You can also (in a hand-wavy sort of sense) reverse your particle accelerator to extract work - the physical processes involved work both ways. See laser cooling and Bose-Einstein condensate for some how and why getting a group of particles moving really slowly in some particular reference frame. - Eldereft ~(s)talk~ 16:46, 15 May 2008 (UTC)[reply]
What particles? Kamiokande uses water. In this case you get Cherenkov radiation. --Lisa4edit (talk) 17:04, 15 May 2008 (UTC)[reply]
The question of "decelerating" a particle is really a question of giving your particle a negative acceleration. That is to say, it's velocity should decrease with time. Of course, the smallest velocity a particle can have is zero. But the question remains, zero velocity with respect to whom. For instance, say you have some charged particles traveling and accelerating to the right due to an electric field pointing to the right. If you turned off the field, the particles would simply travel to the right at some constant speed. (They're in a vacuum so there is no friction.) If you turn on an electric field in the opposite direction, they would "decelerate" until they stopped and then begin accelerating to the left.
Physicists generally talk about acceleration in different directions (to the left and right, etc), rather than "deceleration". Heh, even the deceleration article simply redirects to acceleration! — gogobera (talk) 17:40, 15 May 2008 (UTC)[reply]
I don't believe it's possible to decelerate all particles, however. Photons can't go slower than the speed of light and the theoretical tachyon always travels faster than the speed of light. StuRat (talk) 11:21, 16 May 2008 (UTC)[reply]
Well, photons travel at the speed of light in the medium in which they are travelling (see refractive index). For anything other than a vacuum, that is a speed less than what is colloquially called "the speed of light" (c). Again, follow Lisa4edit's advice to see Cherenkov radiation (I'll add it as a see-also to refractive index). DMacks (talk) 14:45, 16 May 2008 (UTC)[reply]

Dandelion invasion

Hi. Today when I hid the sun behind an object that just obscures the sun's disk, I saw what looked like a heavy snowstorm, only it was May, and the sky was a clear blue with only a few fractus clouds in the distance. Anyway, and this is not homework, I speculated that they were dandelion seeds, which was confirmed when I saw them close up. However, around the sun, several hundred seed packets could be seen, ranging from probably 1 - 50 metres above ground. The wind was strong and from the north, I live in southern Ontario. Anyway, there must have been one per every cubic metre or two, as the highest ones near the sun were maybe as small as 10 arcmins apart from each other. Where did they all come from? To my north there are about 20 hectares of land capable of producing seeding dandelions. This means there must have been over a million individual seeding dandelions producing seed packets! In early to late June, there may be as many as 500 dandelions per square metre in some places, I estimate, if they haven't been mowed yet. Are they an invasive species in North America? If the climate warms a few degrees, warm conditions become longer throught the year, and rainfall becomes sporadic but interspresed with heavy and brief downpours, will there be more invasive dandelions? If so, what will happen to the local ecosystems? Also, do flowering dandelions close and reopen to become seeding ones, and how long does this take? A look at any random part of the sky or landscape for a few seconds will reveal at least a few of these seed packets, and sometimes several dozen! Do people consider them weeds because they're an invasive species or because they choke other vegetation? Thanks. ~AH1(TCU) 17:34, 15 May 2008 (UTC)[reply]

Have a look at the dandelion article to answer some of your questions. A weed is not the same as an invasive species which in turn is not the same as an introduced species. If you get dandelions in your lawn you'll consider them a weed, even if they are useful to local wildlife and co-exist with indigenous species. An introduced species can adapt and become part of the local ecosystem, it may cause some temporary imbalances, or displace a local species, though. If it doesn't multiply in excess and is not harmful to local wildlife, ideally finding some animal that finds it palatable, an introduced species is no better or worse than an indigenous species. (Debatable if it replaces one that then becomes extinct.) An invasive species disrupts the local balance and multiplies in excess. (Common definition). Not all invasives are non-native, but it is easier for an introduced species to become one, because it may lack natural enemies. Temporary proliferation of a certain plant doesn't mean it is invasive. It may serve as a much needed food-source for animals. Dandelions are rather undemanding when it comes to growing conditions, so they can grow in a wide range of habitats. They are rather tasty though (even to humans - try them in a salad), so any mass of them will soon find something to nibble off the leaves. Hope this helps. --Lisa4edit (talk) 18:04, 15 May 2008 (UTC)[reply]

Does light attract itself gravitationally?

The other day I saw an interesting episode of The Universe which discussed the end of the universe through heat death, with the final dark age of the cosmos being one in which all black holes have finally evaporated and all that is left are stray random photons.

This got me wondering, though. Since light is a form of energy, and general relativity says that energy as well as matter bends space/time, does it follow that light can attract itself through gravitational force? After all, if a photon of light contains energy than it should, I think, very slightly bend space/time and thus attract energy and matter. Would it follow that two beams of light fired initially in parallel would slightly bend toward each other even in the absence of other energy or matter? And if so, would it be possible that in the universe's final dark age that stray photons could very, very slowly bend towards each other into wide orbits, producing large galactic size pockets of slightly higher than normal electromagnetic energy or heat? Dugwiki (talk) 17:41, 15 May 2008 (UTC)[reply]

  • does it follow that light can attract itself through gravitational force?
    Yes, light has a gravitational effect that can attract other light. Dragons flight (talk) 18:16, 15 May 2008 (UTC)[reply]
  • Would it follow that two beams of light fired initially in parallel would slightly bend toward each other even in the absence of other energy or matter?
    Not exactly. Gravitational fields propogate at the speed of light as well. Parallel photons would not interact because the gravitational field from one photon could never catch up to the other one. However, if you imagine these as beams of light with many photons, then yes they would interact. The gravitational field from early photons in one beam would effect later photons in the other. This effect would be ridiculously small. Dragons flight (talk) 18:16, 15 May 2008 (UTC)[reply]
  • would it be possible that in the universe's final dark age that stray photons could very, very slowly bend towards each other into wide orbits, producing large galactic size pockets of slightly higher than normal electromagnetic energy or heat?
    I think the answer is no, but I'm not entirely sure. No matter how large you imagine the universe to grow, I don't think you could every get a large enough density of photons (given initial conditions such as exist today) to create bound orbits for other photons. I think the energy density of light is too low and the velocity too high to ever allow for gravitationally bound conglomerations of photons in the future universe. Dragons flight (talk) 18:16, 15 May 2008 (UTC)[reply]
    That sounds right. For photons to orbit, you need something close to a black hole (I think something 8/27 times the density of a black hole would work - a very large neutron star, maybe). Radiation density on the scale of neutron stars seems highly improbably, if even possible. --Tango (talk) 18:43, 15 May 2008 (UTC)[reply]
    Remember that black holes don't have a single density (even an average one): the Schwarzschild radius scales with the mass, which in turn scales with the radius cubed at constant (average) density. So any substance whatsoever can form a black hole if it fills a large enough spherical region. For, say, the cosmic background, with a density of 4.64×10−34 g/(cm3), would form a black hole as a ball with a radius of two trillion light years. (It might be closer to 1.4 trillion; I'm not sure that I shouldn't give it a gravitational density of ; see my oft-repeated link on the subject.) Using instead the critical density of 9.2×10−30 g/(cm3) the radius is 14 billion light years; that this is approximately the age of the universe times the speed of light is not a coincidence. --Tardis (talk) 17:57, 16 May 2008 (UTC)[reply]
(Edit conflict) While photons don't have rest mass, they do have relativistic mass. However, it certainly isn't much mass (for example, I believe a 400 nm photon should have a mass of about 6×10−36 kg). The wave nature of photons causes beam divergence (see diffraction), and as far as I know any gravitational curving of the path due to other photons is negligible compared to that. --Prestidigitator (talk) 18:25, 15 May 2008 (UTC)[reply]

Suspensions

I am looking for examples of situations where there is a need to keep solid particles suspended in water. One example I know of is oil drilling - as the drill progresses the little bits of broken rock need to be carried away in a stream of water - certain additives are added to the water to help keep the rock bits in suspension. Can you think of any other situations? I am more interested in industrial or agricultural applications, less in medical or food applications. Thanks! ike9898 (talk) 17:42, 15 May 2008 (UTC)[reply]

Magnetorheological fluids, though I think those generally use oil, presumably for its higher density and viscosity. One of the ways of making artificial diamonds uses a slurry of diamond grit slowly deposited by a controlled temperature differential. Generally, a gritty liquid like you describe is called a slurry. - Eldereft ~(s)talk~ 20:55, 15 May 2008 (UTC)[reply]
Detergents have non-surfacant ingredient. This keeps the dirt from settling back on the clothes or other surfaces, so it can be flushed through the drain. --Lisa4edit (talk) 06:44, 16 May 2008 (UTC)[reply]

greenhouse gases

how much methane and CO2 do 6 billion people breate out or fart out each day? Paul kahlich (talk) 17:49, 15 May 2008 (UTC)[reply]

We emit 1.5-3.5 kg of CO2 per person per day (1-2 lbs of carbon per day per person). But that doesn't matter because all the carbon we emit comes, directly or indirectly, from plants which gathered their carbon out of the atmosphere in the first place. Hence respiration has no impact on the global carbon balance. Dragons flight (talk) 18:01, 15 May 2008 (UTC)[reply]
If that were true - why would we be concerned with emissions from cows? The problem is that while you're correct in saying that the CO2 production is balanced by the carbon we consume in eating plants - the methane we produce is not. Methane is a much worse greenhouse gas than CO2 (75 times worse!) and it's not reabsorbed by the next generation of plants that we grow to replace the ones we ate. Eventually, methane does decompose into CO2 - but it has a half-life of something like 100 years and since human populations are growing, we're inherently increasing the greenhouse problem. However - as bad as this is, it's nothing like as bad as with animals like cows who ruminate on their food - producing vastly more methane pound-for-pound than humans. 70.116.10.189 (talk) 04:29, 16 May 2008 (UTC)[reply]
Correction. See the methane article, second paragraph. The half life in the atmospere is 7 years, not 100. And because of the decay, the effect gets ever smaller, so one can never state the global warming potential (the effect compared to CO2) without specifying over which period. Most quoted is a number of 23 (stronger than CO2), which apparently is for a period of just over 100 years. Note that 105 years is 15 half-life periods (15 x 7), so by then the amount of methane is reduced to 1/(2^15) = 0,00003 times the original quantity. In other words, it's almost all gone and any global warming effect will be by the resultant CO2. Amrad (talk) 10:41, 16 May 2008 (UTC)[reply]
But temperatures have gone down in spite of rising CO2 levels... isn't global warming attributed to water vapor or sunspots?--WaltCip (talk) 12:16, 16 May 2008 (UTC)[reply]
Temperatures have gone down? Are you joking or do you read taboids of the worst kind? Maybe you read that at some localities temperatures have dropped, but globally, the rise in temperature is so immense that even after a few decades it's a certainty even for the scientific community (as opposed to individual scientists), which is usually extremely careful with its claims. And concerning the solar activity, yes, that plays a role, but it's negligible compared to the effects of the rise in CO2 levels. Btw, sun spots are just another effect of increased solar activity, certainly not a cause for warming. Amrad (talk) 13:46, 16 May 2008 (UTC)[reply]

density of paraffin wax

I'm looking for the density of paraffin wax in liquid phase. The wax we're using melts at 85°C (used in a car thermostat just in case you're wondering) but I'm not familiar with any good sites or databanks? Thanks in advance —Preceding unsigned comment added by 193.190.253.149 (talk) 18:58, 15 May 2008 (UTC)[reply]

Well, a database referenced in our paraffin article, found here, lists "paraffin oil" as 0.8 g/cm^3. Nevermind - turns out paraffin oil is another word for kerosene (as I found out in our paraffin oil. --Bmk (talk) 20:28, 15 May 2008 (UTC)[reply]
It seems that mineral oil is at least close to being liquid paraffin (the alkane lengths seem somewhat smaller than in paraffin wax, but that probably doesn't matter too much). (Its density is actually given only in the latter article and not its own; should fix that.) --Tardis (talk) 16:15, 16 May 2008 (UTC)[reply]

Trees vs Lamp posts

What are the relative strengths of lamp posts and trees (say, of equal width?) when struck, such as by a speeding car? Has there been experiments done on this?--Fangz (talk) 19:30, 15 May 2008 (UTC)[reply]

Well, many lamp posts and street signs are designed to collapse when hit by a vehicle (rather than kill the occupant), so that would definitely need to be taken into account. I saw a bus hit one once and it just toppled over without any resistance whatsoever. --98.217.8.46 (talk) 19:39, 15 May 2008 (UTC)[reply]
A lamp post that's solidly planted in the ground will be much stronger than an equivalently-sized tree. However, every lamp post I've seen has had a breakaway joint at the bottom that shears off easily when struck by a vehicle, but I've never seen a tree with a breakaway joint. --Carnildo (talk) 22:16, 15 May 2008 (UTC)[reply]
Apropos safety, shouldn't those falling lamp posts have a speaker that goes "timbeeeeeer", to warn pedestrians? Amrad (talk) 10:28, 16 May 2008 (UTC)[reply]

A few questions about neurotransmitters

1. Is it possible for the brain/nervous system to run out of transmitter chemicals? 2. If so, how and from what is the supply replenished? 3. If neurotransmitters are re-used, where are they stored when not in use? —Preceding unsigned comment added by 69.224.182.55 (talk) 20:37, 15 May 2008 (UTC)[reply]

Is the infomation in neurotransmitter not suffient for your needs? DMacks (talk) 20:57, 15 May 2008 (UTC)[reply]
[EC] Certainly not under normal circumstances - neutotransmitters are manufactured within neurons. I would recommend reading our article on the chemical synapse - I think it will clear things up. According to that article, most neurotransmitters are taken into either the presynaptic or postsynaptic cell for some degree of decomposition, and then either recycled or disposed of. --Bmk (talk) 20:59, 15 May 2008 (UTC)[reply]

Squirrel feces

Why don't you ever see Squirrel feces on the ground? You see Bird / duck feces, but never other large animals like raccoons. LLOTAAMI (talk) 20:56, 15 May 2008 (UTC)[reply]

I see squirrel feces all too often. Just put a bunch of birdseed on the ground, and wait. -- JSBillings 22:40, 15 May 2008 (UTC)[reply]
That works for raccoon doo-doos too. They really liked corn on the cob we had dangled from the gutter! (Don't try this if you have a cat! The raccoon may get hungry after it's done with the bird seed.)--70.91.165.182 (talk) 00:49, 16 May 2008 (UTC)[reply]

I have a science project that I am allowed to get help for some information

Hi, my name is Lindsay, a fifth grade student. I need help with this topic: LIST WATER'S FREEZING POINT AND MELTING POINT IN BOTH DEGREES, CELSIUS AND FAHRENHEIT.I really need help on it and I couldn't find any imformation on it. I'm still doing my rough draft that is due May 20,2008.PLEASE HELP ME!!!! —Preceding unsigned comment added by 71.243.52.218 (talk) 21:56, 15 May 2008 (UTC)[reply]

The Reference Desk has a firm policy that we will not do your homework for you. However, since this is a project about water, I suggest you start at our water article. The top of that article notes that for chemical and physical properties (such as freezing point), you'll want to refine that to water (molecule) (I'm listing both to illustrate the research process). If necessary, Google can be used to do the Celsius/Fahrenheit conversion: search "98.6 fahrenheit in celsius", for example, and Google will return 37 degrees celsius, which is the average human body temperature. The relevant formulae can be found at both Celsius and Fahrenheit, incidentally. — Lomn 22:02, 15 May 2008 (UTC)[reply]
If you don't already know it, the freezing and melting point of a substance are both approximately the same temperature. Also note that "the freezing/melting point of water" can mean different things. 100% pure water has one freezing/melting point, while the salt-water found in the oceans has a different one. If the teacher didn't specify, they probably mean 100% pure water. The pressure also affects the freezing/melting point, but I'd assume they mean normal air pressure at sea level, called "one atmosphere", unless otherwise specified. You should say all this in your results, though: "The freezing/melting point of 100% pure water at one atmosphere pressure is ...". There are two other temperature scales, named the Rankine scale and Kelvin scale, so including the freezing/melting point of water on those scales might impress the teacher. You might also want to include the freezing/melting point of ocean water on all 4 temperature scales. That should be enough to get an A, I would think. StuRat (talk) 11:04, 16 May 2008 (UTC)[reply]

Benzene resonance

Are benzene and other aromatic compounds are often represented as a resonance of 2 structures with double bonds. Is benzene really a superposition of 2 cyclohexatrienes, i. e. if you somehow detect an electron between carbon 1 and carbon 2 and then do a measurement (very quickly after the first measurement) is there a higher chance of finding an electron between carbon 3 and carbon 4 or carbon 5 and carbon 6 than in the other gaps between the carbon atoms? Thanks. 201.66.22.129 (talk) 22:31, 15 May 2008 (UTC)[reply]

Resonance is a single hybrid structure that is an average of the individual resonance structures, not a set of discrete structures that rapidly change one to another. If you could freeze the pi electrons in-place and observe where these electron particles are at a certain time there may be more between some carbons than others, but a series of such measurements would not have them jumping one position to the next around the ring at alternate times, and that's a poor description of how electrons actually are anyway. So the "real" picture is that there is some pi bond at all positions at all times, but not equivalent to a normal "double bond" at any position ever. DMacks (talk) 22:45, 15 May 2008 (UTC)[reply]
Thanks, I didn't say "very quickly" because I thought of alternating states but because I thought the measurement could disturb the molecule. I am more interested in knowing if the electrons are really entangled in such a way that a measurement would cause them to fall into one or the other configuration for a short time. 201.66.22.129 (talk) 23:32, 15 May 2008 (UTC)[reply]
No. The actual molecular orbital diagram for a conjugated system is completely non-classical: looks very little like "double bonds in various places" at all. If one tried to figure out where each pair was (either by freezing in place, or by somehow observing them), one wouldn't see any pair that was anything like an alkene. Actually, the electron pairs aren't even all equivalent to each other, so the whole concept of three alkene-like pairs chasing each other around (or merging into a single cloud) is pretty wrong. But rather than the electrons themselves being entangled or the "cause" of this mess, it's more that all the p orbitals are entangled (LCAO is a reasonable approximation) and then pi electron pairs simply (fill the lowest molecular orbitals). Each molecular orbital looks almost nothing like an alkene. WP really needs a MO diagram for an aromatic system but doesn't seem to have one. Here is what I mean: notice the shapes of the colored regions for each pair. DMacks (talk) 01:39, 16 May 2008 (UTC)[reply]

May 16

I was reading FANCD2 article.

They used word monoubiquinated. What dose it mean? "This protein is monoubiquinated" That is apparently taken from NCBI web site. I could have requested it in Wiktionary, but afraid it could take years before someone will create article there. Vitall (talk) 01:40, 16 May 2008 (UTC)[reply]

The protein had one ubiquinone unit attached to it. DMacks (talk) 01:52, 16 May 2008 (UTC)[reply]
Thank you! That's perfectly answered my question! Vitall (talk) 02:15, 16 May 2008 (UTC)[reply]
Be careful when reading, as proteins can also be ubiquitinated, which is something altogether different. -- 128.104.112.147 (talk) 15:57, 16 May 2008 (UTC)[reply]

Higher frequency mean more bandwidth

Shorter wavelength mean more bandwidth. Like data could be transfered at higher speed, more bits per second. There should be a law/axiom or something clearly stating this. And I do remember I read about that here, in Wikipedia. Could someone point me to an article, preferably here, but anywhere in the internet would also be good enough. I did tried to Google for like 20 min but fail:( Vitall (talk) 01:49, 16 May 2008 (UTC)[reply]

Something like frequency modulation might have the info you want.--Fangz (talk) 03:18, 16 May 2008 (UTC)[reply]
I think you're looking for the Shannon–Hartley theorem. — Lomn 04:02, 16 May 2008 (UTC)[reply]
Shannon–Hartley theorem state, that the more bandwidth we got, more data(actual bits and bytes) we can transfer in a given time. That is excellent point, but it don't reflect wavelength bandwidth relation. Or am I missing something? Is it actually safe to say, that if we use ten times shorter wavelength we can reliably transfer 10 times more data using same amount of power in a given time(same signal to noise ratio)? Sorry if I making question even more complicated, but this whole thing is confusing. Vitall (talk) 05:09, 16 May 2008 (UTC)[reply]
Um as far as I'm aware there is no direct dependency between wavelength and bandwidth, however the shorter the wavelength, the higher the frequency and at a higher frequency you can have more bandwidth. For example, if your device works between 100-200mhz you only have 100 mhz of bandwidth. If your device works between 11-12ghz, you have 1ghz of bandwidth a 10 fold difference Nil Einne (talk) 06:37, 16 May 2008 (UTC)[reply]
From the article-Surprisingly, bandwidth limitations alone do not impose a cap on maximum information rate. This is because it is still possible for the signal to take on an indefinitely large number of different voltage levels on each symbol pulse, with each slightly different level being assigned a different meaning or bit sequence. If we combine both noise and bandwidth limitations, however, we do find there is a limit to the amount of information that can be transferred by a signal of a bounded power, even when clever multi-level encoding techniques are used. Em3ryguy (talk) 14:25, 16 May 2008 (UTC)[reply]

Endangered Species- Polar Bear

How can we save the polar bears from becoming endangered? —Preceding unsigned comment added by 75.108.103.100 (talk) 02:00, 16 May 2008 (UTC)[reply]

Reverse the effects of global warming (which unfortunately deflects a nearly impossible question with another nearly impossible question). The main threat to the polar bear is loss of habitat due to melting of sea ice. --Bmk (talk) 02:17, 16 May 2008 (UTC)[reply]
If we pray hard enough the Divine Intervention might save them. Vitall (talk) 02:36, 16 May 2008 (UTC)[reply]
I can't think of any course of action less likely to work. That kind of "wish things turn out" attitude is precisely the problem. Matt Deres (talk) 02:48, 16 May 2008 (UTC)[reply]
Well... Making sure all of the sea ice melts as quickly as possible might be even less likely to work than prayer, but apart from that... -- Captain Disdain (talk) 08:52, 16 May 2008 (UTC)[reply]
One radical solution might be to introduce polar bears to Antarctica, which will likely contain ice and snow long after it has all melted at the North Pole. This is because there is so much more ice in Antarctica to start with. Of course, this may have disastrous implications for other species in Antarctica now, such as penguins. StuRat (talk) 10:38, 16 May 2008 (UTC)[reply]

PAUA OPALS

WHAT ARE PAUA OPALS? —Preceding unsigned comment added by 66.191.157.101 (talk) 04:14, 16 May 2008 (UTC)[reply]

It's probably cut pieces of Paua or abalone shell. Graeme Bartlett (talk) 05:14, 16 May 2008 (UTC)[reply]

flame test

colour of tap water in a flame test

Did you read flame test? It will be coloured by the dominating metal, most likely sodium as it gives a much stronger emission than calcium or potassium. Graeme Bartlett (talk) 07:02, 16 May 2008 (UTC)[reply]

Airport X-rays

Do I need to protect electronics (computer, camera, …) from being irradiated by X-rays at the baggage inspection at an airport? Mabye it's necessary to differentiate between two cases: hand luggage and checked-in luggage. —130.237.2.72 (talk) 09:55, 16 May 2008 (UTC)[reply]

Consumer electronics are almost invariably not bothered by airport X-rays (I say almost because I'm sure as soon as I throw in an absolute, someone posts the counterexample). If some affected device exists, it will certainly be well-noted in the instructions. As previous similar topics have noted, the equipment used for carry-ons is less powerful than that used on checked luggage, so when in doubt, carry. Note also that the security of carry-on luggage is superior, and that some items (lithium ion batteries, for one) may be prohibited from checked luggage anyway. If something is really delicate, it's probably worth checking with your local post office to see if they x-ray shipments -- surely some non-x-ray transportation arrangement exists, though you're not going to find it at the airport. — Lomn 13:03, 16 May 2008 (UTC)[reply]
The airport's website will probably have a list of items that you shouldn't put in your baggage - I would expect that to include things that will be damaged in addition to the obvious security rules. --Tango (talk) 13:39, 16 May 2008 (UTC)[reply]
The electronocs of your camera will not be affected. If you are using film, however, you could be a little more cautious. Generally, x-rays are safe for film speeds up to 800 asa. For 800 and above they generally allow you to request screening by hand (though, in my experience, they do not always honor such requests). Even for film below 800 asa, however, you should try an avoid having un-processed film go through the machines several times. — Sam 14:16, 16 May 2008 (UTC) —Preceding unsigned comment added by 63.138.152.238 (talk)

Bush Stone Curlew

Hi What colour do the eyes of a bush stone curlew turn when it dies? —Preceding unsigned comment added by 121.217.227.24 (talk) 10:27, 16 May 2008 (UTC)[reply]

Exercise pill

After exercising, muscles which were flexed repeatedly, grow, of course. I believe this happens because the muscle cells reproduce. I would think this would require some chemical messengers, whether RNA, hormones, or other catalysts or proteins, to tell the muscle cells in the area to reproduce. Do we know what these chemical messengers are ? It doesn't seem to be testosterone or steroids, since increasing those levels alone doesn't cause muscles to grow, the exercise is still needed. I also suspect that the chemical messengers in question don't travel via the circulatory system, since exercising the right arm has no effect on the muscles of the left arm. Are they completely contained within the muscle cells, or do these chemicals diffuse through the cell walls to adjacent cells ? In any case, it would seem to be theoretically possible to provide those chemical messengers to the muscle cells, possibly using a modified virus to deliver them, in order ot get muscle growth without exercise. This would have several benefits:

1) Patients who are unable to exercise, due to a coma, spinal injury, etc., could maintain their muscles until healed.

2) Astronauts could maintain their muscles, which otherwise would atrophy due to weightlessness.

3) People with degenerative muscular diseases, like muscular dystrophy, could possibly be helped.

4) People too lazy to exercise could take a pill to get muscles.

So, does this sound theoretically possible ? StuRat (talk) 10:31, 16 May 2008 (UTC)[reply]

I *think* (or at least someone at the gym told me) that muscle growth occurs as a result of mechanical tearing to the muscle tissue (and the healing thereof) following exercise. I'm not sure as to how much influence chemicals have on the process, beyond those secreted as part of the body's natural healing response. Artificially stimulating cell growth sounds like a 'very bad idea' to me, considering that the heart is also a muscle (one which doesn't need, or like to be messed with) and that not all cell growth is strictly desirable. --Kurt Shaped Box (talk) 12:44, 16 May 2008 (UTC)[reply]

Rolling

I asked this question a little while back, and when I did I suspected that the answer would be no, and that seemed to be the consensus amongst those answering. The question was whether a ball placed on a frictionless inclined plane would roll. I though that it wouldn't roll because it is the force of friction causing the torque, and so without friction there cannot be torque. However, I came across another method of dealing with the problem which has instilled some doubts. In this case, we will treat the axis of rotation not as the center of the object but as the point of contact between the inclined plane and the ball. The idea is that the ball will try to rotate about this axis, but would be unable to because of the normal force, which would redirect it to roll down the inclined plane. In this scenario, the friction force will not add torque because it is applying a force on the axis of rotation, so radius is zero. In fact, it would be gravity which causes the torque. Both methods predict the same acceleration of the ball. So, can a ball rotate on a frictionless inclined plane? —Preceding unsigned comment added by 74.15.138.130 (talk) 12:38, 16 May 2008 (UTC)[reply]

I think the answer we came up with before is correct. When you think of the ball as rotating about the point of contact that's because that point is stationary. There is no reason for it to be stationary if there is no friction to make it so. The ball can rotate if you start it rotating before you let go, but it won't actually roll (since the ball is accelerating down the slope it would need to rotate ever faster to roll, so you can't even start it rotating just the right amount for it to "roll" by coincidence - although you could if you gave it a push on a level plane). --Tango (talk) 13:45, 16 May 2008 (UTC)[reply]
If you move the origin of the coordinates to anywhere except the center of mass of the ball, you lose the nice symmetry and you should deal very carefully to make sure you have calculated all torques. The answer should be the same regardless of where you place the origin, but the natural (simplest equations) are obtained with the standard origin at the center. Nimur (talk) 15:00, 16 May 2008 (UTC)[reply]
Raise your ramp until it is vertical. The normal force is now zero since it is perpendicular to the direction of weight, and friction is zero whether the ramp is "frictionless" or not. In fact, this situation is equivalent to there being no ramp at all. But what happened to your sum of torques around the point of contact? Your logic would have the ball rotating about any point we choose on the ball (except for any point along the line between the top and bottom points). Obviously that cannot be. So what's the problem? ;-) --Prestidigitator (talk) 16:18, 16 May 2008 (UTC)[reply]
Where you set your pivot point is arbitrary. The ball does "rotate" with respect to the new pivot, but that rotation is actually a linear motion with respect to the center. Summing torques only tells you about rotation, or non-radial motion, with respect to the pivot that you select. If you want to know if the ball "rolls", you must take the pivot at the center of the ball, because that is what defines "rolling". SamuelRiv (talk) 16:51, 16 May 2008 (UTC)[reply]
Right. I may have worded my hint a little poorly, but that's the idea. Also many problems dealing with ramps and balls/cylinders assume the point of contact will remain stationary because of friction, which gives you a little more of a basis for dealing with "rolling", but only as long as the assumption holds. Of course, it is often a pretty safe assumption until the word "frictionless" is thrown in. --Prestidigitator (talk) 17:41, 16 May 2008 (UTC)[reply]

Eating freshly harvested grains

Generally we boil dried rice and cook food made from dried wheat flour. Are there any people who eat freshly harvested rice, wheat or rye? How do the "wet" rice and wheat taste?

I wonder if I can put fresh raw fish on freshly harvested raw rice. I may start a revolution in Japanese cuisine! -- Toytoy (talk) 12:39, 16 May 2008 (UTC)[reply]

From rice: 'Raw wild or brown rice may also be consumed by raw foodist or fruitarians if soaked and sprouted (usually 1 week to 30 days).' Algebraist 12:41, 16 May 2008 (UTC)[reply]


What if the Ice Age never happened?

What would the earth be like? Most importantly, how would human beings be like? --Vincebosma (talk) 13:43, 16 May 2008 (UTC)[reply]

There have been many ice ages in Earth's history. Do you want to cancel all of them, or just the most recent? Algebraist 13:48, 16 May 2008 (UTC)[reply]


cancel all of them --Vincebosma (talk) 13:48, 16 May 2008 (UTC)[reply]

(edit conflict) It's worth pointing out that there is no "the ice age", there have, in fact, been several (see ice age#Major ice ages). The first difference to spring to mind would be the lack of any U-shaped valleys. I'm not sure what differences it would have on life, but there would certainly be some - various species have been forced to migrate or evolve (or both) in order to survive an ice age, or have simply died out. --Tango (talk) 13:52, 16 May 2008 (UTC)[reply]
If you cancel all ice ages, then the history of life on earth would be radically different for at least the last few hundreds of millions of years. I doubt anything sensible can be said about what humans would be like in such a circumstance; it's not clear that there would be anything even vaguely humanlike around at all. Algebraist 14:10, 16 May 2008 (UTC)[reply]


OK, how about just cancelling the most recent one? --Vincebosma (talk) 14:24, 16 May 2008 (UTC)[reply]

Well here's one observation: Native Americans came to North American via the Bering Land Bridge created by low sea levels during the last glaciation. Remove that and North American history certainly changes. In addition, if the overkill hypothesis for the Pleistocene extinction of North American animals is correct, you'd also substantially change the distribution of animals in North America. Dragons flight (talk) 14:33, 16 May 2008 (UTC)[reply]
You'd also destroy environments created by glacial activity. For example, the entire environment of the Ozarks region of Missouri exists as the result of glacial activity. Without glaciers, the mineral deposits that were pushed southward wouldn't be on the north side of the ozarks, most of the lakes wouldn't be there, and the mountains would be more mountainy and less hilly. That would affect the wildlife that lives there. Do that on a global scale and you affect all wildlife pretty much everywhere. -- kainaw 15:41, 16 May 2008 (UTC)[reply]
There is steadily mounting evidence that the Bering Land Bridge migration had a much smaller influence in populating the Americas than previously believed. One of the most recent [[34]]. But since the age of sites still falls within the last glacial, the ice sheet might have helped quite a bit. Comparing [[35]] and [[36]] may be useful in seeing how the mineral deposits Kainaw mentioned help agriculture. Lisa4edit (talk) 16:48, 16 May 2008 (UTC)[reply]

Indefinite Viral Carriers

I have been reading about common contagious viruses in humans. Some are commonly transmitted through fecal contact (ie: Coxsackie A virus). Is there a classification of viruses that are indefinitely reproduced in the intestines of humans? I'm looking into reading about treatments to try and eradicate indefinite viral infections in the intestines. If the virus dies out on its own, there obviously won't be a treatment. That is why I'm interested in finding the classes of viruses that don't die out. -- kainaw 15:37, 16 May 2008 (UTC)[reply]

Don't all viruses replicate indefinately? Otherwise they would die out? Fribbler (talk) 16:32, 16 May 2008 (UTC)[reply]
Yes. They replicate indefinitely, but the body fights off the virus and eventually wins out, eradicating it from the body. Some viruses are not beaten - just beaten into submission. I'm interested in viruses located in the human intestines that remain there throughout life but cause contagious diseases. Specifically, I am looking for classification names to help me in searching medical publications. -- kainaw 17:29, 16 May 2008 (UTC)[reply]

CO2

What is the weight ratio of CO2 released to fuel burned? LLOTAAMI (talk) 16:11, 16 May 2008 (UTC)[reply]

Depends on the fuel. DMacks (talk) 16:19, 16 May 2008 (UTC)[reply]
You get 3.7 kg of CO2 per kg of C in the fuel. A fuel like gasoline is about 85% C by mass. Dragons flight (talk) 16:22, 16 May 2008 (UTC)[reply]
(Edit conflict) I believe the best-case scenario would be complete combustion of pure hydrocarbons, which would produce a number of CO2 molecules equal to the number of carbons in the fuel combusted. So you might want to start by trying to find out the typical composition (and probably density since we usually measure fuel in terms of volume not mass) of the fuel you are interested in. If combustion is not complete (which you can bet on) you'll get less CO2 but lots of stuff that is worse. So in any case you should be able to come up with a decent upper-bound, best case figure. --Prestidigitator (talk) 16:40, 16 May 2008 (UTC)[reply]

Effect of salt on blood ADH concentration

Why would an injection of salt solution into a vein or eating large amounts of salt result in the production of Anti diuretic hormone by the pituitary gland? I've often heard it said that salt dehydrates the body. Any help would be great. —Preceding unsigned comment added by 92.21.248.149 (talk) 16:20, 16 May 2008 (UTC)[reply]

To retain water thus attempting to maintain an osmotic balance. Less urine is produced but it is more concentrated, eliminating salt. see Vasopressin. Fribbler (talk) 16:26, 16 May 2008 (UTC)[reply]
ADH secretion is stimulated by osmoreceptors in the hypothalamus, which respond, as the name suggests, to changes in plasma osmolarity which would change if blood volume decreased/increased or sodium chloride concentration were elevated or depleted. (well the ions). Wisdom89 (T / C) 18:15, 16 May 2008 (UTC)[reply]

Speed of Thought

I was thinking (pardon the pun) the other day, and I wondered, what is the speed of thought? How long does it take, once you have enough information, to make a decision? Are all areas of the brain involved or is there a sort of "thinking center" where this takes place? Thank you. Jen17op (talk) 16:24, 16 May 2008 (UTC)[reply]

There are whole branches of science devoted to studying such things. Have you tried starting at the thought article? I'm sure there are lots of artificial intelligence related articles that will provide useful information as well. --Prestidigitator (talk) 16:47, 16 May 2008 (UTC)[reply]
I haven't read it (yet), but I would expect that Malcolm Gladwell's recent book Blink would be relevant to your question. -- Coneslayer (talk) 16:51, 16 May 2008 (UTC)[reply]
Another way to go is start at Brain and have a look at Electroencephalography Functional magnetic resonance imaging Neuron. Don't have the time now to dig up pages, but there have been quite a number of studies correlating increased brain activity in certain areas to corresponding induced thoughts. Having a scout around in the science magazine archives should get you some more information. Lisa4edit (talk) 17:17, 16 May 2008 (UTC)[reply]

biology

what are prions??? what are the possible mechanisms through which they cause diseases??and what are the current techniques being employed to understand them and cure them???